You are on page 1of 227

• Corporate Office : 45, 2nd Floor, Maharishi Dayanand Marg,

Corner Market,
Malviya Nagar, New Delhi-110017
Tel. : 011-49842349 / 49842350

Typeset by Disha DTP Team

DISHA PUBLICATION
ALL RIGHTS RESERVED

© Copyright Publisher
No part of this publication may be reproduced in any form without
prior permission of the publisher and author. The author and the
publisher do not take any legal responsibility for any errors or
misrepresentations that might have crept in. We have tried and made
our best efforts to provide accurate up-to-date information in this
book.

For further information about the books from DISHA,


Log on to www.dishapublication.com or email to
info@dishapublication.com
SSC MULTITASKING SOLVED PAPER 2018 2018-1-
14
SSC MULTITASKING SOLVED PAPER 2017 2017-1-
12
SSC MULTITASKING SOLVED PAPER 2016 2016-1-
14
SSC MULTITASKING SOLVED PAPER 2014 2014-1-
16
SSC MULTITASKING SOLVED PAPER 2013 2013-1-
16
SECTION-A : GENERAL ENGLISH
1. Select the synonym of the given word.
Awkward
(a) Likable
(b) Clumsy
(c) Worthy
(d) Scared
2. Select the correctly spelt word.
(a) Independent
(b) Independant
(c) Indipendent
(d) Indepindent
3. Identify the segment in the sentence which contains the grammatical
error from the given options.
The city of Banaras is widely known from its ghats and temples.
(a) ghats and temples
(b) the city of Banaras
(c) from its
(d) is widely known
4. Select the most appropriate word to fill in the blank.
________ they are twins, they are worlds apart in their attitude to life.
(a) Although
(b) Despite
(c) Whereas
(d) However
5. Select the most appropriate option to substitute the underlined segment
in the given sentence. If there is no need to substitute it, select ‘No
substitution required’.
They won’t commit themselves until they see which way the wind is
blowing.
(a) their till
(b) No substitution required
(c) themself until
(d) them until
6. Select the most appropriate word to fill in the blank and make a
meaningful sentence.
He ________ me up on his way to work.
(a) picked
(b) raised
(c) lifted
(d) dropped
7. Select the most appropriate word to fill in the blank.
The company ________ its first all electric car in the Indian market
soon.
(a) is launching
(b) launched
(c) launch
(d) has launched
8. Select the most appropriate option to substitute the underlined segment
in the given sentence. If there is no need to substitute it, select ‘No
substitution required’.
The government is set strict limits on public spending this year.
(a) had been set
(b) have set
(c) No substitution required
(d) has set
9. Select the most appropriate word to fill in the blank and make a
meaningful sentence.
Conservative parents usually do not give their _______ for uncommon
career choices of their children.
(a) consent
(b) agreement
(c) allowance
(d) dissent
10. Select the most appropriate option to substitute the underlined segment
in the given sentence. If there is no need to substitute it, select ‘No
substitution required’.
The new product will be arrive on supermarket shelves early next year.
(a) No substitution required
(b) will arrive on
(c) will arriving on
(d) will arrived on
11. Select the antonym of the given word.
Persuade
(a) Convince
(b) Endorse
(c) Dissuade
(d) Fume
DIRECTIONS: (Qs. 12-16): In the following passage some words have
been deleted. Fill in the blanks with the help of the alternatives given. Select
the most appropriate option for each blank.
Bengaluru is becoming an exciting destination for higher education for
students (12) ______ all over the world. International students (13) ______
close to 20% of the student population (14) ______ the city and this number
is (15) ______ increasing. The city has almost 900 colleges-the maximum in
any city (16) _____ the country.
12. Select the most appropriate option to fill in blank No. 1.
(a) in
(b) for
(c) from
(d) by
13. Select the most appropriate option to fill in blank No. 2.
(a) constitute
(b) consist
(c) construct
(d) collect
14. Select the most appropriate option to fill in blank No. 3.
(a) at
(b) in
(c) to
(d) on
15. Select the most appropriate option to fill in blank No. 4.
(a) readily
(b) barely
(c) rarely
(d) steadily
16. Select the most appropriate option to fill in blank No. 5.
(a) about
(b) along
(c) across
(d) through
17. Select the word which means the same as the group of words given.
A young child just beginning to walk.
(a) Toddler
(b) Cub
(c) Joey
(d) Kid
18. Select the meaning of the given idiom.
Apples and oranges.
(a) Same things
(b) Different things
(c) Fresh
(d) Nutritious
19. Select the antonym of the given word.
(a) Novel
(b) Tiny
(c) Tough
(d) Gross
20. Identify the segment in the sentence which contains the grammatical
error from the given options.
He started life as a teacher before turn to journalism.
(a) He started life
(b) as a teacher
(c) to journalism
(d) before turn
21. Select the meaning of the given idiom.
At the drop of a hat.
(a) Facing difficulties
(b) Completely
(c) Immediately
(d) Losing a hat
22. Select the word which means the same as the group of words given.
Resistant to a particular infection
(a) Innocent
(b) Immune
(c) Immortal
(d) Innocuous
23. Select the correctly spelt word.
(a) Definitly
(b) Definitely
(c) Definetely
(d) Definately
24. Identify the segment in the sentence which contains the grammatical
error from the given options.
Jaipur, which is renown for its architecture, was founded about 300
years ago.
(a) about 300 years ago
(b) was founded
(c) for its architecture
(d) Jaipur, which is renown
25. Select the synonym of the given word.
Dedicate
(a) Decide
(b) Design
(c) Denounce
(d) Devote
SECTION-B : GENERAL INTELLIGENCE AND REASONING
26. Select the option that is related to the third term in the same way as the
second term is related to the first term.
Jute : Sack : Wood : ?
(a) Furniture
(b) Brown
(c) Plastic
(d) Hard
27. Six buses B1, B2, B3, B4, B5 and B6 are parked around a circular path
(not necessarily in the same order). B3 is third to the right of B6. B4 is
to the immediate left of B1. B2 is second to the left of B6.
How many buses are between B5 and B1?
(a) 0
(b) 1
(c) 3
(d) 2
28. In the following question, select the number which can be placed at the
sign of question mark (?) from the given alternatives.
59639
3 11 7 26
854?
(a) 48
(b) 44
(c) 36
(d) 34
29. In a certain code language, ‘WINDOW’ is written as ‘TLAKFT’. What
is the code for ‘BLADES’ in that code language?
(a) QCBYJZ
(b) PBBXIY
(c) PBAXIY
(d) QBBYJZ
30. In a certain code language, ‘ROAM’ is written as ‘44’, ‘HIMP’ is
written as ‘43’. What is the code for ‘BONE’ in that code language?
(a) 33
(b) 34
(c) 39
(d) 36
31. A piece of paper is folded and punched as shown below in the question
figures. From the given answer figures, indicate how it will appear
when opened?

(a)

(b)

(c)

(d)

32. Which answer figure will complete the pattern in the question figure?

(a)

(b)

(c)
(d)

33. Select the option that is related to the third Letter Cluster in the same
way as the second Letter Cluster is related to the first Letter Cluster.
ROAD : TREI : : MINT : ?
(a) OMRY
(b) PMSZ
(c) PNSZ
(d) OLRY
34. Select the letter-pair that can replace the question mark (?) in the
following series.
RD, XJ, DP, JV, ?
(a) QC
(b) PB
(c) PC (d) QB
35. After interchanging which two numbers, the value of given equation
will be ‘4’?
6+3÷9×7–5
(a) 7 and 6
(b) 3 and 5
(c) 5 and 6
(d) 9 and 5
36. In a certain code language, 'PENCIL' is written as 'RIPGKP'. What is
the code for 'FABRIC' in that code language?
(a) HFDVKG
(b) HEDVKG
(c) IEFWLH
(d) IFEWLH
37. Select the number that can replace the question mark (?) in the
following series.
600, 120, 30, 10, 5, ?
(a) 4
(b) 5
(c) 3
(d) 2
38. If a mirror is placed on the line AB, then which of the answer figures is
the right image of the given figure?

(a)

(b)

(c)

(d)

39. From the given answer figures, select the one in which the question
figure is hidden/embedded.

(a)
(b)

(c)

(d)

40. Identify the Venn diagram that best represents the relationship between
the given classes.
Producer, Director, Actor

(a)

(b)

(c)

(d)

41. In a certain code language, ‘PAL’ is written as ‘29’, ‘CAR’ is written as


‘22’. What is the code for ‘ZON’ in that code language?
(a) 43
(b) 45
(c) 55
(d) 51
42. Select the option that is related to the third term in the same way as the
second term is related to the first term.
Film : Producer : : Poem : ?
(a) Director
(b) Actor
(c) Poet
(d) Cobbler
43. Select the option that is related to the third number in the same way as
the second number is related to the first number.
596 : 965 : : 824 : ?
(a) 248
(b) 408
(c) 418
(d) 258
44. In the following question, select the figure which can be placed at the
sign of question mark (?) from the given alternatives.

(a)

(b)

(c)

(d)

45. In the following question below are given some statements followed by
some conclusions based on those statements. Taking the given
statements to be true even if they seem to be at variance from
commonly known facts. Read all the conclusions and then decide which
of the given conclusion logically follows the given statements.
Statements:
I. Some pen are sharpener.
II. All pen are pencil.
Conclusions:
I. Some pencil are pen.
II. All sharpener are pen.
(a) Both conclusion I and II follows.
(b) Only conclusion I follows.
(c) Neither conclusion follow.
(d) Only conclusion II follows.
46. Five chairs J, H, P, S and M are placed in a row facing towards east
(Not necessarily in the same order). P is second to the left of S. J is
second to the right of H. M is to the immediate right of S.
Which of the following is the correct position of J?
(a) Third to the left of M
(b) Second to the right of H
(c) Exactly between S and H
(d) To the immediate right of S
47. Five toys A, B, C, D and E are kept one above the other (not necessarily
in the same order). A is four places above C. D is between B and E. E is
three places below A.
Three of the given four options follows a same logic based on their
arrangement. Which of the following does not follow that logic?
(a) B D
(b) D E
(c) E C
(d) A C
48. In the following question below are given some statements followed by
some conclusions based on those statements. Taking the given
statements to be true even if they seem to be at variance from
commonly known facts. Read all the conclusions and then decide which
of the given conclusion logically follows the given statements.
Statements:
I. All L are B.
II. No T is L.
III. Some R are T.
Conclusions:
I. Some T are R.
II. No B is T.
III. No L is T.
(a) Only conclusion I follows.
(b) Only conclusion II follows.
(c) Both conclusion I and III follows.
(d) Both conclusion II and III follows.
49. By interchanging the given two signs which of the following equation
will be incorrect?
÷ and +
(a) 12 ÷ 9 × 31 + 3 = 105
(b) 7 × 16 + 4 ÷ 5 = 33
(c) 9 ÷ 11 + 11 × 2 = 9
(d) 6 × 11 + 2 ÷ 5 = 38
50. In the following question, select the missing number from the given
series.
17, 31, 47, 65, ?
(a) 87
(b) 89
(c) 85
(d) 83
SECTION-C : NUMERICAL APTITUDE
51. What is the average of first 15 odd numbers among the natural
numbers?
(a) 18
(b) 15
(c) 16
(d) 17
52. What is the circumference of the largest circle which can be inscribed in
a square of side 14 cm?

(a) 66 cm
(b) 88 cm
(c) 22 cm
(d) 44 cm
53. The marked price of an article is ` 3040. If the discount offered on this
article is 20%, then what will be the selling price?
(a) ` 2412
(b) ` 3262
(c) ` 2432
(d) ` 3132
54. The present age of a Manoj is twice the sum of the ages of his two
children. After 20 years, the age of Manoj will become equal to the sum
of the ages of his two children. What is the present age of Manoj?
(a) 40 years
(b) 30 years
(c) 36 years
(d) 35 years
55. A train starts from A at 6 AM and reaches B at 11 AM on the same day.
Another train starts from B at 8 AM and reaches A at 3 PM on the same
day. At what time the two trains will have crossed each other?
(a) 9 : 45 AM
(b) 8 : 45 AM
(c) 10 : 30 AM
(d) 7 : 45 AM
56. The Bar graph given below presents the number of shoes manufactured
by a company on the different days of a week.
What is the total number of shoes manufactured by the company on all
seven days together?
(a) 5930
(b) 6030
(c) 6130
(d) 6230
57. A sum of ` 10000 is invested in three schemes of simple interest. The
annual interest rates are respectively, 4%, 6% and 10%. ` 4000 were
invested in the first scheme. If the total interest earned after five years is
` 2800, then how much money was invested in the third scheme?
(a) ` 1500
(b) ` 5000
(c) ` 1000
(d) ` 3000
58. N and K together can complete a work in 240 days. K and G together
can complete the same work in 72 days and N and G together can
complete the same work in 80 days. In how many days K alone can
complete the same work?
(a) 280 days
(b) 240 days
(c) 360 days
(d) 180 days
59. A sum of ` 1200 is invested at compound interest (compounded half
yearly). If the rate of interest is 10% per annum, then what will be the
amount after 18 months?
(a) ` 1389.15
(b) ` 1185.45
(c) ` 1563.25
(d) ` 1295.35
60. The Bar graph given below presents the number of shoes manufactured
by a company on the different days of a week.
The number of shoes manufactured on D5 is how much more than the
average number of shoes manufactured per day?
(a) 72
(b) 64
(c) 76
(d) 68
61. The edge of a cube is 8 cm. What is the total surface area of the cube?
(a) 128 cm2
(b) 256 cm2
(c) 384 cm2
(d) 484 cm2
62. What is the median of the given data?
41, 43, 46, 50, 85, 61, 76, 55, 58, 95
(a) 61
(b) 58
(c) 57
(d) 55
63. If A is the smallest three digit number divisible by both 6 and 7 and B is
the largest four digit number divisible by both 6 and 7, then what is the
value of B – A?
(a) 9912
(b) 9870
(c) 9996
(d) 9954
64. Raman and Sanjay started a business by investing ` 63000 and ` 42000
respectively. If the total profit at the end of year is ` 9000, then what is
the share of Raman?
(a) ` 5400
(b) ` 4500
(c) ` 4200
(d) ` 3600
65. The ratio of the selling price to the cost price in a transaction is 4 : 5. If
the selling price is ` 80, then how much is the loss?
(a) ` 16
(b) ` 15
(c) ` 20
(d) ` 30
66. What is the value of:
(9 ÷ 30)2 × 2.4 + 0.3 o f 12 × (1 – 0.3)2 + 9 × (0.3)2 = ?
(a) 3.43
(b) 3.69
(c) 2.79
(d) 2.17
67. Vijay alone can complete a work in 50 days. How much part of the
work will be completed in ten days?
(a)

(b)

(c)

(d)
68. The profit earned on selling an article at ` 720 is half of the loss
incurred on selling the same article at ` 360. What is the cost price of
the article.
(a) ` 540
(b) ` 600
(c) ` 480
(d) ` 420
69. The Bar graph given below presents the number of shoes manufactured
by a company on the different days of a week.

The number of shoes manufactured on D1 is what percentage of shoes


manufactured on D7?
(a) 80.45
(b) 60
(c) 80
(d) 70.25
70. The ratio of the age of a father and his son is 3 : 1. If the product of
their ages is 432, then what is the sum of their ages?
(a) 36 years
(b) 48 years
(c) 60 years
(d) 54 years
71. What is the value of:
2 of 3 ÷ 3 × 2 + {4 × 3 – (5 × 2 + 3)} = ?
(a) 3
(b) – 24
(c) 6
(d) – 21
72. Rahul’s salary is 40% less than Rakesh’s salary. Deepak’s salary is 80%
more than Rahul’s salary. If Deepak’s salary is ` 34560, then what is
the salary of Rakesh?
(a) ` 32000
(b) ` 24000
(c) ` 28000
(d) ` 26000
73. A man goes from C to D at 40 km/h and he returns from D to C at x
km/h. If the average speed of the man for the whole journey is 60 km/h,
then what is the value of x?
(a) 100
(b) 120
(c) 110
(d) 80
74. The average of four numbers is 20. If the average of the first two
numbers is 15, then what is the average of the last two numbers?
(a) 22
(b) 18
(c) 25
(d) 20
75. The length of one of the diagonals of a rhombus is 48 cm. If the side of
the rhombus is 26 cm, then what is the area of the rhombus?
(a) 540 cm2
(b) 420 cm2
(c) 360 cm2
(d) 480 cm2
SECTION-D : GENERAL AWARENESS
76. Asian Infrastructure Investment Bank headquarter is located in which
city?
(a) Philippines
(b) Gurugram
(c) Shanghai
(d) Beijing
77. What is Cryptogamae?
(a) Plants having covered seeds
(b) Flowering plants
(c) Non-flowering plants
(d) Plants having uncovered seeds
78. The ratio of the length to the height (width) of the national Flag shall
be?
(a) 3 : 1
(b) 4 : 1
(c) 4 : 2
(d) 3 : 2
79. Who was the first women Chief Minister of an Indian state?
(a) Indira Gandhi
(b) Sucheta Kriplani
(c) Sarojini Naidu
(d) Mamta Banerjee
80. According to Census 2011, which of the following states has the highest
density of population in India?
(a) Uttar Pradesh
(b) West Bengal
(c) Bihar
(d) Maharashtra
81. India's foreign reserves does NOT consist of which of the following?
(a) Gold
(b) Carbon Bonds
(c) Reserve Tranche Position (RTP)
(d) Special Drawing Rights (SDR)
82. National waterway No. 2 is located on which river?
(a) Brahmaputra
(b) Ganga
(c) Cauvery
(d) Mahanadi
83. Which of the following Jyotirlingas is in Maharashtra?
(a) Mahakaleshwar
(b) Baidyanath
(c) Grishneshwar
(d) Malikarjuna
84. Which of the following sportspersons has received the ‘Rajiv Gandhi
Khel Ratna’ Award?
(a) Gita Phogat
(b) Saina Nehwal
(c) Deepika Kumari
(d) Kapil Dev
85. Who was the organizer of revolt of 1857 in Bihar?
(a) Bahadur Shah
(b) Nana Sahib
(c) Maulvi Ahmadullah
(d) Kunwar Singh
86. Deepika Kumari is associated with which Sport?
(a) Archery
(b) Squash
(c) Swimming
(d) Badminton
87. “Healthy States, Progressive India” Report has been published by which
of the following institution?
(a) Ministry of Health and Family Welfare
(b) World Health Organization (WHO)
(c) United Nations
(d) Niti Aayog
88. The Sixth Schedule of the Indian Constitution does NOT exercise its
control to the tribal areas of which of the following state?
(a) Manipur
(b) Tripura
(c) Assam
(d) Meghalaya
89. Botanical Survey of India (BSI) Headquarter is located in which city?
(a) Assam
(b) New Delhi
(c) Kolkata
(d) Bhopal
90. 1 pound = ________ ounces
(a) 16
(b) 18
(c) 12
(d) 20
91. Which of the following is NOT a feature of Eukaryotic cell?
(a) Nucleus well organized
(b) Mitochondria present
(c) Chloroplast absent in plant cells
(d) Nuclear membrane present
92. The World famous Borobudur Temple is situated in:
(a) Vietnam
(b) Indonesia
(c) Cambodia
(d) Japan
93. GST is NOT applicable on which of the following goods or services?
(a) Automobiles
(b) Salon services
(c) Restaurant services
(d) Alcohol
94. Which of the following Indian rulers was defeated by Mahmud Ghazni
in his first attack in the year 1000 AD?
(a) Chandra Pala
(b) Anand Pala
(c) Jaya Pala
(d) Sukh Pala
95. “Neel Darpan” was written by Dinbandhu Mitra to portray the
oppression faced by peasants to grow indigo in:
(a) Madras
(b) Assam
(c) Gujarat
(d) Bengal
96. Minamata Convention is signed to tackle the issue of:
(a) Nuclear radiations
(b) Ozone gas
(c) CFC and HFC
(d) Mercury
97. Inter-State Council was set up by the recommendation of which
committee?
(a) Sarkaria committee
(b) Y V Reddy committee
(c) TSR Subramanyam committee
(d) Chandrababu Naidu committee
98. Mawsynram is located on which hills of Meghalaya?
(a) Garo
(b) Jaintia
(c) Diphu
(d) Khasi
99. Who among the following takes part in election of the President of
India?
(a) Elected Members of both the houses of Parliament and Legislative
Assemblies of the states.
(b) Elected and nominated members of both the houses of Parliament.
(c) Elected members of both the houses of Parliament only
(d) Elected and nominated members of both the houses of Parliament
and Legislative Assemblies of the states.
100. National Youth Festival is celebrated on the birth anniversary of which
of the following personalities?
(a) Milkha Singh
(b) Chandrashekhar Azad
(c) Bhagat Singh
(d) Swami Vivekanand
1. (b) The word 'awkward' means ‘difficult to deal with’ the words
'likable' and worthy' are opposite in meaning 'Scared' means
'fearful' which is not related. Hence 'clumsy' is the correct answer.
2. (a) 'Independent' is the correctly spelt word.
3. (c) 'from its' should be replaced with 'for'
4. (a) Here two contrasting situations are mentioned so conjunction
'although' is corect choice.
5. (b) Here, 'thenselves' is the reflexive pronoun of 'they' other options do
not fit correct grammatically. Hence 'No substitution is required'.
6. (b) 'picked' is the correct option as 'raised' and 'lifted' are used to
improve level of someting, 'lifted' to take something up by hand
and 'dropped up' is not possible.
7. (a) launched and 'has launched' are used when the action has taken
place and 'launch' shows regularty but the action is going to happen
so ‘is launching’ is the correct answer other options do not fit
correct.
8. (d) 'has set' is the correct option as 'the government' is taken as
singular noun 'had been set' and 'is set' are used in passive voice.
9. (a) 'Consent' is the correct option 'agreement' and 'allowance'
(informer) are used formally and dissent does not make proper
sense.
10. (b) 'Will arrive on' is the correct answer as other options (will be arrive
on, will arriving on, will arrived on) are not grammatically correct.
11. (c) Here, the words 'convince' and 'endorse' are similar to 'persuade'
(fume - amount of vapour/smoke) is not related so, 'dissuade' is the
correct option.
12. (c) 'From' is the correct answer other 'prepositions' do not fit correct.
13. (a) 'Constituted' means 'to be one of the parts that form something' is
the correct option.
'Construct' means 'to build or make something'
'Consist' means be composed or made up of
'Collect' means 'to gather something'
14. (b) 'In' is the correct preposition before the place name (city)
15. (d) The word 'steadily' means 'in a regular way, continuously' is the
correct option.
16. (c) Preposition 'across' is the correct option.
17. (a) 'Toddler' is the correct option.
'Cub' is the young one of lion, fox, wolves and other carnivorous a
mammals.
'Joey' is the young one of kangaroo or 'marsupials'
'Kid' is the infants or small child.
18. (b) 'Apples and oranges' have no match, so, the idiom 'apples and
oranges' means 'different thing'
19. (c) The word 'Fragile' means 'that which breaks easily get damaged.
So, the word (Tough means 'hard or difficult') is the correct option.
20. (d) 'Before turn' should be replaced with 'before turning'
21. (c) The idiom 'at the drop of a hat' means 'immediately'.
So, the option no. (c) is the correct option.
22. (b) Here, the word 'innocent' means 'not done wrong' 'immortal' means
'never ending' and 'innocuous' means 'not harmful'
So, the option no. (b) (immune) is the correct word.
23. (a) 'Definitely' is the correctly spelt word.
24. (a) Jaipur, which is renown should be replaced with 'Jaipur, which is
renowned'
25. (a) The word 'dedicate' means 'devote'.
Hence, the option no. (d) is the right option.
26. (a) Jute is used to make sacks like that wood is used to make furniture.
27. (d) Given Six Buses – B1, B2, B3, B4, B5, B6
Arrangement – Circular
Steps:
(a) B3 is third to the Right of B6
(b) B2 is second to the left of B6

(c) B4 is to the immediate left of B1


Only the Bus B5 is left.
Hence the final arrangement is

Hence 2 buses are between B5 and B1, as counted clockwise or


anticlockwise direction.
28. (c) 5 9 6 = 5 × 9 – 6 = 39
3 11 7 = 3 × 11 – 7 = 26
8 5 4 = 8 × 5 – 4 = 36

29. (c)

Similarly,

30. (a) Look at the numbered alphabet and write down the no. position
corresponding to the letters of the word 'ROAM'
R O A M
18 15 1 13
The fact that the code for the ROAM is 44, gives you a clue that
the code is Probably obtained by performing all airthmatical
operations of the numbers of each letter.
Let us see
18 + 15 + 1 + 13 = 47
Further we have to find the final code which is given in the
question is '44'.
To get the final code we have to substract 3 from the addition of
the position of the each letter i.e. 47 – 3 = 44.
Similarly for, HIMP as 43
Thus, the code for 'BONE' is
B O N E
2 + 15 + 14 + 5 = 36
So, the final code is 36 – 3 = 33
31. (d) 32. (a)
33. (d) Here, every letter of the word ROAD follow the pattern is like +2,
+3, +4, +5.

Similarly, every letter in the word 'MINT' follow the following


pattern given in the above

34. (b) Here in the letter pair 5 letter are omitted in alphabetical sequence.
The following diagram gives you the more clear picture:

Hence PB can replace the question mark.


35. (a) 6 + 3 ÷ 9 × 7 – 5
Here we have to interchange the no. 7 and 6 as
⇒ 7+3÷9×6–5
⇒ 7+ ×6–5

⇒ 7+2–5
9–5=4
Hence proved. The equation will be equal to 4.
36. (b) Here you can see that 1 and 3 letters omitted alternatively in given
alphabetic sequence. The following diagram gives you the more
clear picture.

Clearly
'P' (skip 1 letter) 'R'
'E' (skip 3 letter) 'I'
'N' (skip 1 letter) 'P'
'C, (skip 3 letter) 'G'
"I" (skip 1 letter)"K"
'L' (skip 3 letter) 'P'
Similarly,
FABRIC can be coded. Let us see

∴ Code for FABRIC is HEDVKG.


37. (b) 600, 120, 30, 10, 5 ?
The pattern is follow like as

5 × 2 = 10
10 × 3 = 30
30 × 4 = 120
120 × 5 = 600
38. (c) 39. (c)

40. (c)

41. (c) The code given for PAL is 29.


This is the sum of the position of the letter in the alphabetic series.
P + A + L = 29
16 + 1 + 12 = 29
Hence for
Z + O + N = 55
26 + 15 + 14 = 55
42. (c) Film is produced by producer and poem is written by poet.
43. (a) Here the position of numbers are interchage only.
So, for

44. (b) Here the pattern followed as the one arrow added on back,
alternatively and the other arrow added on the Head alternatively.
Hence the final arrangement

45. (b) Venn diagram for the given statement is

Conclusion (I): Some pencil are pen is definitely true because we


have given that– All pen are pencil it means some part of pencil are
also pen.
Conclusion (II): All sharpener are pen is definately false because
we have given only some pen are sharpener it means some
sharpener are also pen but not the all sharpener are pen.
46. (b)
(1) P is second to the left of S.
(2) M is to the immediate right of S.
(3) J is second to the right of H

Hence the correct position of J is second to the right of H.


47. (d) (1) A is four place above C
(2) E is three places below A
(3) D is between B and E

Arrangement is

Here, all the pair follow the pattern of immediate neighbor of each
other except AC.
48. (c) Venn diagram is like

Conclusions:-
(i) Some T are R is true
(ii) No B is T is T is false
(iii) No L is T is true
Hence both I and III follows.
49. (c) From the option no. (c),
Equation 9 ÷ 11 + 11 × 2 = 9 is incorrect as we interchanging the
given two signs ÷ & + ,
Now, we have to solve the equation
⇒ 9 + 11 ÷ 11 × 2 = 9
⇒9+1×2=9 ⇒ 9 + 2 = 9 ⇒ 11 ≠ 9
Hence the option (c) is incorrect.
50. (c) 17, 31, 47, 65, ?
The difference b/w two numbers are from right to left
31 – 17 = 14
47 – 31 = 16
65 – 47 = 18
Hence the pattern is

Hence 65 + 20 =
51. (b) Here the first 15 odd natural number are
1 , 3, 5, 7, 9, 11, 13, 15, 17, 19, 21, 23, 25, 27, 29
Sum of first 15 odd numbers are
1 + 3 + 5 ....... + 29 = 225
Hence the average is = = 15

52. (d)

Hence the radius of circle is = 7 cm


Circumference of the circle is = 2πr
=2× × 7 = 44 cm

53. (c) Marked price of an article is = 3040


Discount = 20% = =

So, Hence the selling price = 4 unit.


Marked price is 5 unit.
5 unit → 3040
1 unit → 608
Hence the selling price is = 4 × 608 = ` 2432.
54. (a) Let present age of Manoj is x years.
and their two sons present age are y and z years.
According to the question,
x = 2(y + z) ...(i)
Their ages after 20 years.
x + 20 =(y + 20) + (z + 20)
x =y + z + 20
From equation (1),
x= + 20 ; x – = 20

= 20 ⇒ x = 40

Hence, present age of father = 40 years.


55. (a) Le the distance between A to B is d km.
Time of travel of first train = 11 – 6 = 5 hr.
Time of travel of second train
= 3 p.m. – 8 a.m. = 7 hr.
then Speed of first train = km/hr.

Speed of second train = km/hr.

Now, again we suppose that two train meet after t hr. from 8 a.m.
then, =d

=1

=1

12t = 35 – 14
t= = 1 hr. 45 min.

Hence, time when two train meets


=8 + (1 hr 45 min) = 9:45 am.
56. (d) The total number of shoes nanufactured by the company on all
seven days together is
Days = D1 + D2 + D3 + D4 + D5 + D6 + D7
= 720 + 628 + 740 + 942 + 966 + 1034 + 1200
= 6230
57. (c) Let the amount invested in third scheme is x then amount invested
in second scheme is 1000 – 4000 – x = (6000 – x)
Now, S.I. =

2800 =

2800 =800 + 1800 –

200 = ⇒ x = 1000

Hence, money invested in third scheme = ` 1000.


58. (c) (N + K)'s 1 days' work =

(K + G)'s 1 days' work =

(N + G)'s 1 days' work =

On adding, 2(N + K + G)'s 1 days work


=

= =

∴ (N + K + G)'s 1 days work =

∴ K's 1 day work = –


= = =

Hence, K alone can complete whole work in 360 days.


59. (a) P = 1200
Time = 18 month for half yearly time becomes twice
= 18 × 2 = 36 month = 3 year
Rate = 10% = 5% = for half yearly
Let the principal = 125

Total amount → 125 + 6.25 + 6.25 + 6.25 + 0.3125 + 0.3125 +


0.3125 + 0.015625
⇒ 144.703125 unit
125 unit → 1200 (given)
144.703125 → × 144.703125

Amount = ` 1389.15
60. (c) The number of shoes manufactured on D5 = 966
Average of shoes manufactured per day
= = 890
966 – 890 = 76
61. (c) Edge of cube = 8 cm
The total surface are a of cube is = 6a2
a = 8 cm
S.A. = 6 × 8 × 8 = 64 × 6 = 384 cm2
62. (b) To find the mediam
(1) Arrange the numbers in increasing order.
(2) Here we have even no. of term. So, we have to add the two middle
terms and divide by 2.
Data arrange in increasing order
41 43 46 50 68 76 85 95

Hence the median is


63. (b) L.C.M. of 6 and 7 = 42
Smallest 3 digits number divisible by 6 and 7 is the same that is
divisible by 42
and that number is A = 42 × 3 = 126
Largest 4 digits number that is divisible by 6
and 7 is the some that is divisible by 42
and that number is B = 238 × 42 = 9996
Now, B – A. = 9996 – 126 = 9870
64. (a) Raman's capital = `63000
Sanjay 's capital = `42000
Total profit = `9000
as we know,
Capital × Time = Profit
So, Ratio of their investments
Raman Sanjay
Capital 63000 : 42000
Time 1 year : 1 year
Profit 63000 : 42000
:
Hence the Raman's share is
⇒ ⇒ 1800 × 3 ⇒ `5400
65. (c) Ratio of the selling price to the cost price is = 4 : 5
Selling price is ` 80
Ratio,

Hence the loss is 1 unit


4 unit → 80
1 unit → 20
Hence the loss is 1 × 20 = ` 20
66. (c) (9 ÷ 30)2 × 2.4 + 0.3 of 12 × (1 – 0.3)2 + 9 × (0.3)2 = ?
⇒ (0.3)2 × 2.4 + 3.6 × (0.7)2 + 9 × (0.09)
⇒ (0.09) × 2.4 + 1.764 + 9 × (0.09)
⇒ 1.98 + 0.81
⇒ 2.79
67. (a) Vijay can do a work alone in 50 days
Work = Efficiency × Time
Hence 1 unit work completed in → 50 days
or 50 days → 1 work
10 days → =

Hence part of the work will be completed in 10 days.

68. (b)
69. (b) The number of shoes manufactured on D1 is = 720
Hence number of shoes manufactured on D7 is = 1200
The number of shoes manufactured on D1 is what percentage of
shoes manufactured on D7
the percentage is
⇒ ⇒ ⇒ 60%

70. (b) The ratio of their ages


Father : Son
3x : 1x
Product of their ages is 432
i.e. 3x2 = 432
x2 = 144
x = ±12
Age connot be negative so we have to opt + 12.
Hence, the age of father = 3 × 12 = 36
Age of son = x = 12
Sum of their ages = 12 + 36 = 48 years.
71. (a) 2 of 3 ÷ 3 × 2 + {4 × 3 – (5 × 2 + 3)}
⇒ 2 × 1 × 2 + {12 – (13)}
⇒ 2 × 2 + {– 1}
⇒4–1
⇒3
72. (a) Let the salary of Rakesh be 100 unit
We have given Rahul's salary is 40%
Less than Rakesh's salary and Deepak's salary is 80% more than
Rahul's salary.
So the ratio of the salaries are
Rakesh Deepak Rahul
100 108 60
(80% more than (40% less than
Rahul's Rakesh
= 48 100 – 40 = 60

60 + 48 = 108
Ratio's of salaries are
Rakesh Deepak Rahul

25 : 27 : 15
We have Deepak's salary = `34560
27 unit → `34560
1 unit → `1280
Hence the salary of Rakesh is
= 1280 × 25 = ` 32000
73. (b)
Average speed of the man =

We have given the average speed of the man = 60 km/h


= 60

⇒ = 60

⇒ = 60

⇒ 80x = 2400 + 60x


80x – 60x = 2400

2x = 240
x = 120
74. (c) Let the 4 no's be x1, x2, x3 and x4

Average = 20
x1 + x2 + x3 + x4 = 80
Average of first two numbers are 15
= 15

x1 + x2 = 30 ...(1)
x1 + x2 + x3 + x4 = 80
From the eq. (1) we have put the value of
x1 + x2 = 30
x3 + x4 = 80 – 30
x3 + x4 = 50
Hence the average of last two no. is
= = 25
75. (d) Given, Diagonal of a rhombus is 48 cm

Side of rhombus is 26 cm
Area of rhombus = × d1 × d2

The diagonal in a rhombus are perpendicular and bisect of other


diagonal.
Given length side 26 cm
One diagonal = 48 cm = DB
Now, OB = OD = 24 cm
Now consider triangle AOB this is a right angled triangle and we
know AB = 26 cm and OB = 24 cm
Now using the pythagorus theorem we can find out length of OA
as
OA2 = AB2 – OB2
= 676 – 576
OA2 = 100
OA = 10 cm
So the length of diagonal AC is 2 × 10 = 20 cm.
Area of Rhombus is × 20 × 48 = 480 cm2.
76. (d) The Asian Infrastructure Investment Bank (AIIB) is a multilateral
development bank with a mission to improve social and economic
outcomes in Asia. Its headquarter is in Beijing. It started
operations in January 2016 and has now grown to 100 approved
members worldwide.
77. (c) The word Cryptogams means hidden reproduction. Cryptogams
are flowerless and seedless plants. These plants bear no flowers
and fruits. Cryptogams include all non-seed bearing plants.
78. (d) The ratio of the length to the height (width) of the National Flag
shall be 4:3:2.
79. (b) Sucheta Kripalani ( 25 June 1908 – 1 December 1974) was
an Indian freedom fighter and politician. She was India’s first
woman Chief Minister, serving as the head of the Uttar
Pradesh government from 1963 to 1967. She was born in Ambala,
into a Brahmo family.
80. (c) The records of population density 2011 of India state that Bihar is
the most thickly populated state (1106 persons/sq km.) followed by
west bengal-1028 and Kerala 860.
81. (b) India’s foreign reserves does not consist of carbon bond, but
consists of gold , Reserve Tranche Position (RTP) and Special
Drawing Rights (SDR).
82. (a) National Waterway No. 2 extends from Sadiya to Dhubri up to
891 km in river Brahmaputra. It was selected in September 1988.
Devang and Lohit rivers meet Brahmaputra near Sadiya and before
Dhubri the famous river Manas of Assam joins it from the North.
The world famous Mujoli river island is located in this waterway.
83. (c) The Grisneshwar Temple is one of the shrines dedicated to Lord
Shiva . The temple is an important pilgrimage site in
the Shaiva tradition of Hinduism. This pilgrimage site is located in
Ellora . It is about 30 kilometres north-west of the city
of Aurangabad (Maharashtra).
84. (b) In 2010, Saina Nehwal got Rajiv Gandhi Khel Ratna Award in
2010 for Badminton. Nehwal has achieved several milestones in
badminton for India. She is the only Indian to have won at least
one medal in every BWF major individual event, namely
the Olympics, the BWF World Championships, and the BWF
World Junior Championships. She is the first Indian badminton
player to have won an Olympic medal.
85. (d) Veer Kunwar Singh (1777– 26 April 1858) was a leader during
the Indian Rebellion of 1857. He belonged to a Zamindar family
of the Parmar Rajputs of Jagdispur, currently a part of Bhojpur
district, Bihar. At the age of 80, he led a select band of armed
soldiers against the troops under the command of the British East
India Company. He was the chief organizer of the fight against the
British in Bihar.
86. (a) Deepika Kumari is an Indian athlete who competes in the event
of archery, is currently ranked World No. 5, and is a former world
number one. She won a gold medal in the 2010 Commonwealth
games in the women’s individual recurve event. She also won a
gold medal in the same competition in the women’s team recurve
event along with Dola Banerjee and Bombayala Devi.
87. (d) “Healthy States, Progressive India” is a comprehensive Health
Index report released by the NITI Aayog. The report ranks states
and Union territories innovatively on their year-on-year
incremental change in health outcomes, as well as, their overall
performance with respect to each other. The report has been
developed by NITI Aayog, with technical assistance from the
World Bank, and in consultation with the Ministry of Health and
Family Welfare.
88. (a) The sixth schedule of the Indian constitution does not exercise its
control to the tribal areas of Manipur . The Manipur (Hill Areas)
District Council Act, 1971 was passed by the Parliament providing
for constitution of District Councils for the Hill Areas of the State
of Manipur for the smooth functioning and better administration.
Hill Areas of Manipur to protect the rights of the Hill people of
different ethnic groups/tribal people settling in the State of
Manipur in lieu of enforcement of Sixth Schedule in the State of
Manipur as the same is not applicable under the Constitution of
India.
89. (c) The headquarter of the Botanical Survey of India is located in
Kolkata .In a rapidly developing country like India, it is necessary
to survey and document the floristic wealth of the country.The
Botanical Survey of India is the apex research organization under
the Ministry of Environment, Forests & Climate Change with the
mandate of Survey, Collection, Documentation (including the
traditional knowledge associated with plants), acting as the
custodian of authentic collections in well planned herbaria and ex-
situ conservation of wild plant diversity of the country.
90. (a) 1 lb (pound)= 16 oz(ounces)
91. (c) Eukaryotes are organisms whose cells have nucleus
enclosedwithin membranes, nlike prokaryotes ,which have
no embrane-bound organelles. Eukaryotes belongtothe domain
Eukaryota . Eukaryotic cells also contain other membrane-bound
organelles such as mitochondria and the Golgi apparatus, and in
addition, some cells of plants and algae contain chloroplasts.
92. (b) The Borobudur Temple Compounds is one of the greatest Buddhist
monuments in the world. It was built in the 8th and 9th centuries
AD during the reign of the Syailendra Dynasty. The monument
is located in the Kedu Valley, in the southern part of Central Java,
Indonesia.
93. (d) Alcohol for human consumption does not fall under the purview of
GST in India at present. The taxes imposed to Alcohol for human
consumption are continued as per the structure before GST
implementation.
94. (c) The invasions of Mahmud Ghazni took place in 1000
AD. Mahmud of Ghazni for first time attacked modern
Afghanistan and Pakistan in 1000 AD. He defeated Hindu ruler
Jaya Pala, who committed suicide himself later and was succeeded
by his son Anandpala.
95. (d) Nil Darpan is a Bengali play written by Dinabandhu Mitra in
1858–1859. The play was better known as the Indigo Revolt of
February–March 1859 in Bengal, when farmers refused to sow
indigo in their fields to protest against exploitative farming under
the British Raj.
96. (d) A UN conference in Japan adopted a treaty to stop the production
and use of Mercury, a deadly nerve toxin. Under the agreement, a
range of products, including thermometers, barometers and
electrical appliances will be phased out. The treaty has been named
after Minamata city that witnessed one of the worst incidents of
industrial poisoning by mercury, which killed hundreds and
disabled thousands of people in the 1950s and ’60s.
97. (a) The Inter-State Council is a non-permanent constitutional body set
up by a presidential order on the basis of provisions in Article 263
of the Constitution of India. The body was formed by a Presidential
Order dated 28 May, 1990 on recommendation of Sarkaria
Commission. The Council is formed to discuss or investigate
policies, subjects of common interest, and disputes among states.
98. (d)
99. (d) The president is indirectly elected by an electoral college
comprising the Parliament (both houses- Lok sabha and Rajya
sabha) and the legislative assemblies of states and territories, who
themselves are all directly elected.
100. (d) National Youth Day is celebrated on 12 January being the birthday
of Swami Vivekananda. In 1984 the Government of India declared
this day as National Youth Day and since from 1985 the event
is celebrated in India every year. It was quoted that the
philosophy of Swamiji and the ideals for which he lived and
worked could be a great source of inspiration for the Indian Youth.
PART-A : GENERAL INTELLIGENCE AND REASONING
1. In the following question, select the number which can be placed at the
sign of question mark (?) from the given alternatives.

(a) 36
(b) 62
(c) 80
(d) 84
2. How many triangles are there in the given figure?

(a) 5
(b) 6
(c) 7
(d) 8
3. A word is represented by only one set of numbers as given in any one
of the alternatives. The sets of numbers given in the alternatives are
represented by two classes of alphabets as shown in the given two
matrices. The columns and rows of Matrix-I are numbered from 0 to 4
and that of Matrix-II are numbered from 5 to 9. A letter from these
matrices can be represented first by its row and next by its column, for
example, 'Z' can be represented by 20, 97 etc. and 'Y' can be
represented by 42, 79 etc. Similarly, you have to identify the set for the
word "FRONT".
Matrix-I Matrix-II

(a) 00, 11, 32, 04, 12


(b) 95, 67, 32, 55, 75
(c) 00, 31, 85, 03, 76
(d) 95, 11, 79, 55, 23
4. From the given answer figures, select the one in which the question
figure is hidden/embedded.

(a)

(b)

(c)

(d)

5. Which answer figure will complete the pattern in the question figure?
(a)

(b)

(c)

(d)

6. If a mirror is placed on the line AB, then which of the answer figures is
the right image of the given figure?

(a)

(b)
(c)

(d)

7. A piece of paper is folded and punched as shown below in the question


figures. From the given answer figures, indicate how it will appear
when opened?

(a)

(b)

(c)

(d)

8. If 'x' means '+', '+' means 'x', '–' means '÷' and '÷' means '–', then which
of the following equation is correct?
(a) 12 – 3 × 4 = 12
(b) 5 + 2 – 4 = 13/2
(c) 49 × 7 – 14 = 98/2
(d) 44 + 4 – 2 = 88
9. If 3 # 9 @ 4 = 3 and 4 # 4 @ 4 = 2, then 6 # 4 @ 5 = ?
(a) 3
(b) 2
(c) 4
(d) 1
10. Arrange the given words in the sequence in which they occur in the
dictionary.
1. Viper
2. Vapour
3. Victory
4. Vacuum
5. Visit
(a) 42135
(b) 42315
(c) 24315
(d) 24135
11. In the following question, select the missing number from the given
alternatives.
?, 28, 65, 126, 217, 344
(a) 7
(b) 4
(c) 9
(d) 5
12. A series is given with one term missing. Select the correct alternative
from the given ones that will complete the series.
J, O, T, Y, D, ?
(a) I
(b) J
(c) H
(d) K
13. In the following question, select the odd word from the given
alternatives.
(a) Metre
(b) Kilometre
(c) Centimetre
(d) Litre
14. In the following question, select the odd number pair from the given
alternatives.
(a) 24 : 47
(b) 16 : 31
(c) 28 : 55
(d) 32 : 65
15. In the following question, select the odd letters from the given
alternatives.
(a) ACEG
(b) MOQS
(c) FHIK
(d) PRTV
16. Identify the diagram that best represents the relationship among the
given classes.
Cricketer, Male, Actor

(a)

(b)

(c)

(d)

17. In the following question, select the related word from the given
alternatives.
Love : Hate : : Deep : ?
(a) Long
(b) Bright
(c) Shallow
(d) High
18. In the following question, select the related number from the given
alternatives.
18636 : 3106 : : 2508 : ?
(a) 418
(b) 406
(c) 394
(d) 430
19. In the following question, select the related letters from the given
alternatives.
LP : QU : : VX : ?
(a) BD
(b) AD
(c) ZB
(d) AC
20. In the following question, select the word which cannot be formed
using the letters of the given word.
OPTICIAN
(a) PAINT
(b) PAIN
(c) COIN
(d) TAIL
21. P is father of Q, but Q is not his son. S is wife of P. R is son of S. How
is Q related to S?
(a) Brother
(b) Daughter
(c) Father
(d) Cannot be determined
22. Megha walks 10 km towards North. She turns right and walks 15 km.
She turns right and walks 20 km. She turns right and walks 15 km. How
far (in km) is she from her starting point?
(a) 15
(b) 5
(c) 10
(d) 20
23. In the following question below are given some statements followed by
some conclusions. Taking the given statements to be true even if they
seem to be at variance from commonly known facts, read all the
conclusions and then decide which of the given conclusion logically
follows the given statements.
Statements:
I. All pens are cups.
II. All cups are chairs.
Conclusions:
I. All pens are chairs.
II. Some cups are pens.
(a) Only conclusion (I) follows.
(b) Only conclusion (II) follows.
(c) Both conclusion follow.
(d) Neither conclusion (I) nor conclusion (II) follows.
24. In a certain code language, "BOOK" is written as "CQRO". How is
"ROAD" written in that code language?
(a) SQDH
(b) SQCH
(c) SRDH
(d) SREH
25. In a certain code language, "HELLO" is written as "97151620" and
"WORLD" is written as "241721169". How is "FRUIT" written in that
code language?
(a) 720241325
(b) 720231325
(c) 720241324
(d) 719231325
PART-B : NUMERICAL APTITUDE
26. Which of the following is true?
(a)
(b)

(c)

(d)

27. What is the simplified value of ?

(a)
(b)
(c)
(d)
28. If A = 23 × 34 and B = 25 × 32, then what is the value of A × B?
(a) 28 × 36
(b) 215 × 38
(c) 28 × 38
(d) 215 × 36
29. Which of the following relation (s) is/are true?
I.
II.
III.
(a) Only I
(b) Only II
(c) Neither I nor II
(d) Either I or II

30. What is the simplified value of ?

(a) 4
(b) 1.4
(c) 1.3
(d) 2.7
31. How many two digit numbers are divisible by 9?
(a) 9
(b) 8
(c) 10
(d) 1
32. P alone can complete a work in 10 days and Q alone can complete the
same work in 20 days. If P and Q work together, then in how many days
the same work will be completed?
(a) days

(b) days

(c) days

(d) days

33. A and B together can complete a work in 8 days. B alone can complete
the work in 24 days. In how many days A alone can complete the same
work?
(a) 16 days
(b) 14 days
(c) 12 days
(d) 13 days
34. What is the area (in cm2) of a square having perimeter 84 cm?
(a) 361
(b) 529
(c) 484
(d) 441
35. Two successive discounts of 30% and 70% are given. What will be the
net discount (in percentage)?
(a) 63
(b) 79
(c) 100
(d) 87
36. After giving a discount of 40% on an article, there is still a profit of
25%. If no discount is given, then what will be the profit percentage?
(a) 109.33%
(b) 25%
(c) 107.67%
(d) 108.33%
37. Marks of A and B are in the ratio 5 : 7 respectively. If the marks of A is
25, then find the marks of B?
(a) 60
(b) 35
(c) 28
(d) 45
38. If A : B = 4 : 5 and B : C = 20 : 11, then find A : B : C?
(a) 4 : 20 : 7
(b) 16 : 25 : 22
(c) 16 : 20 : 11
(d) 8 : 10 : 7
39. Average of 43, 57, 68, 32, 97 and x is 63. What is the value of x?
(a) 83
(b) 77
(c) 81
(d) 75
40. If the ratio of selling price and cost price is 3 : 4 respectively, then what
is the loss percentage?
(a) 10%
(b) 33.33%
(c) 20%
(d) 25%
41. An article is sold at 30% loss. If the selling price is increased by 50%,
then what is the profit percentage?
(a) 5%
(b) 8%
(c) 12%
(d) 20%
42. What is the value of 20% of 30% of 7200?
(a) 480
(b) 432
(c) 412
(d) 360
43. 12% of what number is equal to 30% of 960?
(a) 2400
(b) 1720
(c) 1440
(d) 2880
44. If a car travels a distance with 20% less speed, then it will reach 15
minutes late. What is the usual time (in minutes) taken by the car to
travel the same distance?
(a) 80
(b) 90
(c) 75
(d) 60
45. A goes from Point X to Point Y at a speed of 60 km/hr and comes back
with a speed of 80 km/hr. What is the average speed (in km/hr) of A for
going and coming back?
(a) 66.66
(b) 68.57
(c) 67.33
(d) 69.43
46. A certain amount double in 5 years, when invested at simple interest. In
how many years will it become 8 times?
(a) 35 years
(b) 40 years
(c) 30 years
(d) 45 years
DIRECTIONS (Qs. 47-50): The line chart given below shows the
expenditure on clothes (in 000') for the given years.
47. Expenditure in year 2016 is how much percent more than the
expenditure in year 2015?
(a) 19.07%
(b) 23.57%
(c) 21.16%
(d) 17.64%
48. What is the average expenditure (in 000') per year from the year 2014 to
2016?
(a) 3120
(b) 3080
(c) 3000
(d) 3430
49. Expenditure in the year 2014 is what percent of the expenditure in year
2013?
(a) 118.71%
(b) 18.71%
(c) 115.76%
(d) 15.76%
50. Expenditure in the year 2012 is how much more (in 000') than the
expenditure in year 2013?
(a) 878
(b) 158
(c) 228
(d) 335
PART-C : GENERAL ENGLISH
51. In the following question, some part of the sentence may have errors.
Find out which part of the sentence has an error and select the
appropriate option. If a sentence is free from error, select 'No Error'.
This book stall is very well stocked (1)/ you can almost get (2)/ every
book here. (3)/ No Error (4)
(a) 1
(b) 2
(c) 3
(d) 4
52. In the following question, some part of the sentence may have errors.
Find out which part of the sentence has an error and select the
appropriate option. If a sentence is free from error, select 'No Error'.
The old woman continued living a hard life (1)/ but never she asked (2)/
for any help from neighbours. (3)/ No Error (4)
(a) 1
(b) 2
(c) 3
(d) 4
53. In the following question, the sentence given with blank to be filled in
with an appropriate word. Select the correct alternative out of the four
and indicate it by selecting the appropriate option.
He runs faster than ______.
(a) I
(b) me
(c) my
(d) his
54. In the following question, the sentence given with blank to be filled in
with an appropriate word. Select the correct alternative out of the four
and indicate it by selecting the appropriate option.
There is no difference between ______ and me.
(a) yourself
(b) you
(c) yours
(d) your
55. In the following question, out of the given four alternatives, select the
one which best expresses the meaning of the given word.
BLEAK
(a) Comfortable
(b) Depressing
(c) Pleasant
(d) Bright
56. In the following question, out of the given four
alternatives, select the one which best expresses the
meaning of the given word.
CHAOS
(a) Order
(b) Quiet
(c) Mix-up
(d) Normality
57. In the following question, out of the given four alternatives, select the
one which best expresses the meaning of the given word.
CHORUS
(a) Soloist
(b) Signing group
(c) Artist
(d) Pundit
58. In the following question, out of the given four alternatives, select the
one which is opposite in meaning of the given word.
SPLIT
(a) Clear
(b) Position
(c) Join
(d) Distribute
59. In the following question, out of the given four alternatives, select the
one which is opposite in meaning of the given word.
CHERISH
(a) Abandon
(b) Admire
(c) Nourish
(d) Shelter
60. In the following question, out of the given four alternatives, select the
one which is opposite in meaning of the given word.
PROCEEDING
(a) Action
(b) Passiveness
(c) Movement
(d) Procedure
61. In the following question, out of the four given alternatives, select the
alternative which best expresses the meaning of the Idiom/Phrase.
Make a fortune
(a) To become rich
(b) Boasting
(c) Unavoidable
(d) Reviewing leisurely
62. In the following question, out of the four given alternatives, select the
alternative which best expresses the meaning of the Idiom/Phrase.
Cry down
(a) To depreciate
(b) Jealousy
(c) Spending diligently
(d) Think creatively
63. Improve the bracketed part of the sentence.
The jury were divided in (its) decision.
(a) their
(b) his
(c) her
(d) No improvement
64. Improve the bracketed part of the sentence.
Everybody was there except (me).
(a) I
(b) my
(c) mine
(d) No improvement
65. In the following question, out of the four given alternatives, select the
alternative which is the best substitute of the phrase.
Flesh Eater
(a) Herbivore
(b) Carnivore
(c) Omnivore
(d) Piscivore
66. In the following question, out of the four given alternatives, select the
alternative which is the best substitute of the phrase.
One who prescribes medicine
(a) Optician
(b) Politician
(c) Beautician
(d) Physician
67. In the following question, a word has been written in four different
ways out of which only one is correctly spelt. Select the correctly spelt
word.
(a) Sufering
(b) Suffering
(c) Suffiring
(d) Sufireng
68. In the following question, a word has been written in four different
ways out of which only one is correctly spelt. Select the correctly spelt
word.
(a) Cuorageous
(b) Couragious
(c) Couragoeus
(d) Courageous
69. Rearrange the parts of the sentence in correct order.
Modern e-mail
P: intenet or other
Q: computer networks
R: operates across the
(a) RQP
(b) PQR
(c) QRP
(d) RPQ
70. Rearrange the parts of the sentence in correct order.
The most frightening
P: is that it is likely to
Q: aspect of malnutrition
R: cause permanent brain damage
(a) RPQ
(b) PQR
(c) QPR
(d) RQR
DIRECTIONS (Qs. 71-75): In the following passage some of the words
have been left out. Read the passage carefully and select the correct answer
for the given blank out of the four alternatives.
A mother is the most ______ person in the life of everyone about which we
cannot ______ completely in the words. However, some of the ______
moments with our mother can be described. A mother is ______ most
beautiful and caring person in our lives. She always cares every moment for
our every need without her any ______ intention. In the morning, she calls us
very softly to get a rise from the bed and during the night she tells us lovely
stories to make us sleep with a beautiful dream.
71. A mother is the most ______ person
(a) precarious
(b) poetic
(c) precious
(d) dependent
72. which we cannot ______completely
(a) describe
(b) write
(c) speak
(d) listen
73. However some of the ______ moments
(a) valueless
(b) valuable
(c) checked
(d) filtered
74. A mother is ______ most beautiful
(a) the
(b) a
(c) an
(d) my
75. Need without her any ______ intention.
(a) public
(b) official
(c) customized
(d) personal
PART-D : GENERAL AWARENESS
76. Which among the following is called a medium of exchange?
(a) Money
(b) Crops
(c) Gold
(d) Land
77. In which kind of wants, both parties have agree to sell and buy each
other commodities?
(a) Single coincidence
(b) Double coincidence
(c) Triple coincidence
(d) None of these
78. Which among the following is/are organ/organs of Indian Government?
I. The Legislature
II. The Judiciary
(a) Only I
(b) Only II
(c) Both I and II
(d) Neither I nor II
79. How many fundamental duties does part IV-A of Indian constitution
specifies?
(a) 5
(b) 7
(c) 11
(d) 31
80. Who was the most famous Kushan ruler?
(a) Ashvaghosha
(b) Kanishka
(c) Heraios
(d) Ban Yong
81. Which Mughal emperor issued a 'Farman' granting the East India
Company the right to trade duty free?
(a) Shah Jahan
(b) Aurangzeb
(c) Bahadur Shah Zafar
(d) Shah Alam
82. 90 degrees south latitude marks what?
(a) South Pole
(b) North Pole
(c) Equator
(d) Prime Meridian
83. What is the area of the Earth's surface covered with water called?
(a) Lithosphere
(b) Atmosphere
(c) Biosphere
(d) Hydrosphere
84. Which of the metallic ions is essential for blood clotting?
(a) Na+
(b) Ca++
(c) K+
(d) Fe++
85. The total number of bones in the body of an adult human is ______.
(a) 205
(b) 206
(c) 216
(d) 636
86. Largest gland in human body is ______.
(a) Liver
(b) Pancreas
(c) Pituitary
(d) Thyroid
87. A force may change ______ of an object.
(a) speed
(b) direction
(c) shape
(d) All options are correct
88. Good conductors have many loosely bound ______.
(a) neutrons
(b) protons
(c) positron
(d) electrons
89. What is the full form of URL?
(a) Universal Resource Locator
(b) Universal Resource Location
(c) Uniform Resource Locator
(d) Uniform Resource Location
90. Isotopes differ in ______.
(a) No. of electrons
(b) No. of protons
(c) No. of neutrons
(d) Chemical reactivity
91. Chlorine atom and chloride ions ______.
(a) Have an equal number of protons
(b) Have an equal number of electrons
(c) Unequal number neutrons
(d) react spontaneously with water
92. Air pollution has harmful effect on which of the following?
(a) Living
(b) Non living
(c) Both living and non living
(d) Neither living nor not living
93. What is Pradhan Mantri Suraksha Bima Yojana?
(a) A health insurance scheme
(b) A financial inclusion scheme
(c) A life insurance scheme
(d) An accident insurance scheme
94. Who invented the 'world wide web'?
(a) Tim Berners-Lee
(b) John O'Sullivan
(c) Jan Koum
(d) Steve Jobs
95. Sachin Tendulkar played his last test match in which stadium?
(a) Wankhede Stadium
(b) DY Patil Stadium
(c) Brabourne Stadium
(d) None of these
96. The annual cultural festivals held at Surajkund, Faridabad is an
International _____ Mela.
(a) Books
(b) Crafts
(c) Science
(d) Cattle
97. Who among the following is a winner of 2016 Rajeev Gandhi Khel
Ratna Award?
(a) Sania Mirza
(b) Sakshi Malik
(c) Deepa Kumari
(d) Dutee Chand
98. Who is the author of the book named "Unbreakable"?
(a) Mary Kom
(b) Ahmed Faraz
(c) Vijay Dasda
(d) None of these
99. In 1974, India became the _____ non-arab state to recognize Palestine
Liberation Organisation as sole and legitimate representative of the
people of palestine.
(a) First
(b) Second
(c) Third
(d) Fourth
100. Which of the following neighbouring country does not share land
border with India?
(a) Maldives
(b) Bangladesh
(c) Myanmar
(d) Pakistan
1. (b)

2. (c) There are 7 triangles in all namely, ABC, ACD, ABP, BPQ, BQC,
ABQ and BPC

3. (a) From the given matrices, we can find that

So correct option is 00, 11, 32, 04, 12


4. (d) Clearly, option (d) is the correct figure.
5. (a) Figure given in option (a) is the correct missing figure.
6. (c) Option (c) is the correct image of the given figure.
7. (a) Option (a) is correct.
8. (d) Option (a) 12 – 3 × 4 → 12 ÷ 3 + 4 = 8 ≠ 12
Option (b) 5 + 2 – 4 → 5 × 2 ÷ 4 =

Option (c) 49 × 7 – 14 → 49 + 7 ÷ 14
=
Option (d) 44 + 4 – 2 → 44 × 4 ÷ 2 = 88 = 88
So, option (4) is correct.
9. (b) 3 # 9 @ 4 → (3 + 9) ÷ 4 = 3
4 # 4 @ 4 → (4 + 4) ÷ 4 = 2
So, 6 # 4 @ 5 → (6 + 4) ÷ 5 = 2
10. (b) The correct order of words is :

11. (c)

So, missing number = 23 + 1 = 9


12. (a)

13. (d) Metre, Kilometre and Centimetre are units of length. Litre is the
unit of volume.
So, option (d) is correct.
14. (d) 24 : 47 → 24 × 2 – 1 = 47
16 : 31 → 16 × 2 – 1 = 31
28 : 55 → 28 × 2 – 1 = 55
32 : 65 → 32 × 2 + 1 = 65
15. ( c)

16. (c) Cricketer and Actor are two disjoint sets. But both of them can be
male.
So, option (c) correctly represents the given set.
17. (c) Love and Hate are opposite words. Similarly, deep and shallow are
opposites.

18. (a)

19. (d)

20. (d) TAIL cannot be formed from the given word as L is not in the
word OPTICIAN.
21. (b) P is father of Q, but Q is not his son means Q is daughter of P. S is
wife of P, so P and S are couple, so Q is daughter of S.
22. (c) Let O be the starting point and F be the final point. So, she is 10
km from starting point.
23. (c)

So, both conclusions follow.

24. (a)

Similarly,

25. (a)

26. (a) LCM of 4, 7, 8 = 56


So,

So,

27. (a)

28. (a) A = 23 × 34
B = 25 × 32
A × B = (23 × 25) × (34 × 32 )
= 28 × 36
29. (b) = 1.41
and = 1.44 So,
30. (a) (2.7)3 + (1.3)3 = [(2.7) + (1.3)] [(2.7)2 – 2.7 × 1.3 + (1.3)2]

⇒ = (2.7 + 1.3) = 4

31. (c) Two digit numbers that are divisible by 9 are :


18, 27, 36, 45, 54, 63, 72, 81, 90, 99 (Total =10)
32. (b) P's 1 day work =

Q's 1 day work =

P's and Q's one day work


=

So, they will complete the work in days i.e. days.

33. (c) Let A alone can complete the work in x days


⇒ x = 12 days
34. (d) Perimeter of the square = 84 cm
side of the square = = 21 cm

Area of the square = (21 cm)2 = 441 cm2


35. (b) Let M.P. be ` 100
Then, S.P. after 1st discount = 100 – 30% of 100
= 70
S.P. after 2nd discount = 70 – 70% of 70
= 70 – 49 = 21
So, net discount = (100 – 21) % = 79%
36. (d) Let M.P. of the article be ` 100. Then
S.P. = ` (100 – 40) = ` 60
Profit = 25%
C.P. = = ` 48

If no discount is given, then S.P. = ` 100


Profit = ` 100 – ` 48 = ` 52
Profit % = = = 108.33%

37. (b) Marks of B = = 35

38. (c) A : B = 4 : 5 ⇒ A : B = 16 : 20
B : C = 20 : 11
∴ A : B : C = 16 : 20 : 11
39. (c) = 63

⇒ 297 + x = 378 ⇒ x = 81
40. (d) Let SP = 3x and CP = 4x
Loss = 4x – 3x = x
Loss % = = 25%
41. (a) Let CP of the article is ` 100
Then SP = ` 100 – ` 30 = ` 70
If selling price is increased by 50%, then
New SP = ` (70 + 50% of 70) = ` 105
Profit = ` 5 = 5%
42. (b) 30% of 7200 = = 2160

20% of 2160 = = 432

43. (a) Let 12% of x = 30% of 960


⇒ ⇒x=

⇒ x = 2400
44. (d) Let distance be x and speed be y
Then, time =

Reduced speed = y – 20% of y =

Increased time =

It is given that = 15 ⇒

= 15
⇒ = 15 ⇒ = 60

45. (b) Let distance between point X and point Y be x km.


Then, time taken to travel from X to Y (t1) = h
Time taken to travel from Y to X (t2) =

Total time taken =

Average speed =

= = 68.57 km/hr
46. (a) Here, I = P, R = R, T = 5 years
⇒P= ⇒ R = 20%

If amount becomes 8 times, then interest would be 7P


⇒ 7P = ⇒7= ⇒ T = 35 years.

47. (b) Expenditure in 2015 = 3075


Expenditure in 2016 = 3800
Increase in expenditure = 3800 – 3075 = 725
% Increase =

= = 23.57%

48. (c) Average expenditure from 2014 to 2016


=

= = 3000

49. (a) Expenditure in 2014 = 2125


Expenditure in 2013 = 1790
Required % = =
= 118.71%
50. (c) Expenditure in 2013 = 1790
Expenditure in 2012 = 2018
Difference = 2018 – 1790 = 228
51. (b) Use 'get' before 'almost' to make the sentence structurally correct.
52. (b) Use 'she' before 'never' to make the sentence structurally correct.
53. (a)
54. (b)
55. (b ) Bleak means Charmless, dreary etc. Therefore, 'depressing' is its
correct synonym. Rest options are its antonym.
56. (c) Chaos means complete disorder and confusion. Therefore, 'mix-up'
is its correct synonym. Rest options are its antonym.
57. (b) Chorus is a large organized group of singers, especially one which
performs with an orchestra or opera company.
58. (c) Split means break or cause to break. Therefore, 'join' is its correct
antonym.
59. (a) Cherish means to protect and care for (someone) lovingly.
Therefore, 'abandon' which means cease to support or look after
(someone) is its correct antonym. Rest options are its synonym.
60. (b)
61. (a) Make a fortune is a phrase which means to acquire great wealth by
one's own efforts.
62. (a) Cry down is an idiom which means to disparage, depreciate.
63. (a) Use 'their' instead of 'its' due to the use of 'were' in the sentence.
64. (d)
65. (b)
66. (d)
67. (b)
68. (d)
69. (d) RPQ
70. (c) QPR
71. (c) Precious
72. (a) describe
73. (b) Valuable
74. (a) the
75. (d) Personal
76. (a) Money is any object that is generally accepted as payment for
goods and services and the repayment of debt. Money functions as
a medium of exchange, a unit of account, and a store of value.
77. (b) The double coincidence of wants mean that both the parties have
to agree to sell and buy each other's commodity i.e. what a person
desires to sell is exactly what the other person wishes to buy.
78. (c) Government of India (GoI) is constituted of three organs namely
Executive, Legislature and Judiciary to administer the country and
its people efficiently.
79. (c) Article 51 A of Part IVA of Indian Constitution deals with
Fundamental Duties. Fundamental duties were added by 42nd and
86th Constitutional Amendment acts. As of now there are 11
Fundamental duties. Citizens are morally obligated by the
Constitution to perform these duties. However, these are non-
justifiable, i.e. without any legal sanction in case of their violation
or non-compliance.
80. (b) Kanishka, c.AD 120, king of Gandhara was the most powerful and
renowned ruler of the Kushan dynasty, one of the five tribes of
theYüeh-chih who had divided Bactria among them.
81. (b) By 1696, the Company began building a fort around the Hugli
settlement. It also bribed Mughal officials into giving the Company
Zamindari rights over three villages, one of which was Calcutta. It
also convinced the Mughal emperor Aurangzeb to issue a
farman, an official order, granting the Company the right to trade
duty-free.
82. (a) 90 degrees south latitude is the South Pole, on the continent of
Antarctica.
83. (d) A hydrosphere is the total amount of water on the planet. The
hydrosphere includes water that is on the surface of the planet,
underground, and in the air. A planet's hydrosphere can be liquid,
vapour, or ice.
84. (b) The chief element involved in clotting of blood is Calcium (Ca++),
which by itself is the 4th clotting factor. It helps in activating many
other clotting factors. Furthermore, it helps in conversion of
prothrombin to thrombin, which is essential for clot formation.
85. (b) Human body is composed of around 270 bones at birth - this
decreases to around 206 bones by adulthood after some bones get
fused together.
86. (a) Liver is the largest gland in human body. It is also the largest
(internal) organ in our body and can weigh up to 1.5 kg for a
human adult. That is, about 1/50th of the body weight is because of
liver.
87. (d) A force is a push, pull, or dragging of an object that affects its
motion. The action from a force can cause an object to accelerate,
to decelerate, to stop or to change direction. An external force may
cause a change in the internal arrangement of molecules making up
the object and depending upon the magnitude of the force, the size
or shape of the object may change.
88. (d) A Good Conductor implies that the outer electrons of the atoms are
loosely bound and free to move through the material.
89. (c) A uniform resource locator (URL) is the address of a resource on
the Internet.
90. (c) Isotopes is atoms with the same number of protons, but differing
numbers of neutrons. In other words, they have different atomic
weights. Isotopes are different forms of a single element.
91. (a) Chlorine is a naturally occurring element with a symbol Cl and
atomic number 17 whereas Chloride is an ion of Chlorine. In
Chlorine atom, the number of protons (17) is equal to the number
of electrons (17). In chloride ion, there are 17 protons but 18
electrons. Thus, the number of protons is same.
92. (c) Air Pollution affects not only living things but non-living things as
well. For example- Taj Mahal is getting affected due to air
pollution.
93. (d) Pradhan Mantri Suraksha Bima Yojana is a government-backed
accident insurance scheme in India. It was formally launched by
Prime Minister Narendra Modi on 9 May, 2015. It is available to
people between 18 and 70 years of age with bank accounts. It has
an annual premium of Rs 12 exclusive of taxes.
94. (a) English scientist Tim Berners-Lee invented the World Wide Web
in 1989.
95. (a) Sachin Tendulkar retired from all forms of cricket on 16
November 2013 after playing his 200th Test match, against the
West Indies in Mumbai's Wankhede Stadium.
96. (b) Every year from 1-15 February, (in 2018, a colourful traditional
craft festival of India, Surajkund Mela is held in the precincts of
Surajkund. This fair was first started in 1987.
97. (b) Four athletes, P V Sindhu, Dipa Karmakar, Jitu Rai and Sakshi
Malik received the Rajiv Gandhi Khel Ratna Award from the
President in 2016.
98. (a) Unbreakable: An Autobiography is the autobiography of the
unbreakable Mary Kom who is India's best known boxer, five
times World Champion and an Olympic medallist.
99. (a) India was the first non-Arab country to contemporaneously
recognize the Palestine Liberation Organisation's authority as "the
sole legitimate representative of the Palestinian people." A PLO
office was set up in the Indian capital in 1975, with full diplomatic
relations established in March 1980.
100. (a) Neighbouring countries with which India shares its borders are
Myanmar, Bangladesh, China, Bhutan, Nepal, Afghanistan, and
Pakistan.
PART-A : GENERAL INTELLIGENCE AND REASONING
1. From the given alternative words, select the word which cannot be
formed using the letters of the given word.
VENTURE
(a) RENT
(b) TRUE
(c) TURN
(d) RATE
2. If '+' means '×', '–' means '÷', '×' means '–' and '÷' means '+' then the
value of the given equation is:
9+8÷8–4×6=?
(a) 11
(b) 65
(c) 68
(d) 36
DIRECTIONS (Qs. 3 and 4): A series is given with one term missing. Out
of the four alternatives, choose the alternatives that will complete the series.
3. ACE, GIK, ? SUW, YAC
(a) MOQ
(b) MPQ
(c) MOP
(d) MNP
4. 2, 5, 10, 17, 26, ?
(a) 37
(b) 35
(c) 38
(d) 33
5. Find the missing number from the given responses.
4 8 2
3 2 2
5 8 ?
60 128 68
(a) 13
(b) 17
(c) 15
(d) 19
6. A man facing south turns to his left and walk 10m, then he turns to his
right and walk 15m, again he turns to his left and walk 5m and then he
turns to his left and walk 15m. In which direction he is facing now?
(a) South
(b) West
(c) North
(d) East
7. Find the number of triangles in the adjoining figure.

(a) 16
(b) 20
(c) 18
(d) 14
DIRECTIONS (Qs. 8 to 11): Select the related word/letters/number from
the given alternatives.
8. 8 : 512 :: 9 : ?
(a) 728
(b) 729
(c) 781
(d) 792
9. ADIP : DGLS :: BEJQ : ?
(a) FINU
(b) EJQU
(c) EHMT
(d) CGLS
10. AUTHOR : PEN :: DOCTOR : ?
(a) HOSPITAL
(b) DISPENSARY
(c) STETHOSCOPE
(d) WARD
11. POLITE : ETILOP
(a) DRAOB : BROAD
(b) SINGLE : ELGNIS
(c) CHART : TRACH
(d) WOMEN : WOMAN
12. One or two statements are given followed by two Conclusions/
Assumptions, I and II. You have to consider the statement to be true,
even if it seems to be at variance from commonly known facts. You are
to decide which of the given conclusions/assumptions can definitely be
drawn from the given statement. Indicate your answer.
Statement: No man is a donkey. Rahul is a man.
Conclusions:
I. Rahul is not a donkey.
II. All men are not Rahul
(a) Only Conclusion I follow
(b) Neither Conclusion I nor Conclusion II follow.
(c) Only Conclusion II follow
(d) Either Conclusion I or Conclusion II follow.
13. Some equations are solved on the basis of certain system, Find out the
correct answer for the unsolved equation on that basis.
9 × 6 × 2 – 269, 8 × 6 × 5 = 568, 5 × 4 × 1 = ?
(a) 201
(b) 145
(c) 415
(d) 451
14. From the given answer figures, select the one in which the question
figure is hidden/embedded.

15. If a mirror is placed on the line MN, then which of the answer figures is
the right image of the given figure?

DIRECTIONS (Qs. 16 to 18): Find the odd number/letters/words from the


given alternatives.
16. (a) KILOMETERS (b) KILOGRAMS
(c) TONNES
(d) QUINTALS
17. (a) JKNQ
(b) DEGJ
(c) YZBE
(d) QRTW
18. (a) 154
(b) 119
(c) 85
(d) 51
19. A piece of paper is folded and cut as shown below in the question
figures. From the given answer figures, indicate how it will appear
when opened.

20. Which one of the given responses would be a meaningful order of the
following?
1. Family 2. Community 3. Member
4. Locality 5. Country
(a) 3, 1, 2, 4, 5
(b) 3, 1, 2, 5, 4
(c) 3, 1, 4, 2, 5
(d) 3, 1, 4, 5, 2
21. The age of father is twice that of the elder son, After ten years, the age
of father will be three times that of the younger son. If the difference of
ages of the two sons is 15 years, the age of the father is?
(a) 70 years
(b) 55 years
(c) 60 years
(d) 50 years
22. If NAME is coded as MZLD, how will CLAIM be coded?
(a) BKZII
(b) BKYHL
(c) BKZHL
(d) BKZHI
23. Which one of the following diagrams represents the relationship
among:

24. A word is represented by only one set of numbers as given in any one
of the alternatives. The sets of numbers given in the alternatives are
represented by two classes of alphabets as in two matrices given below.
The columns and rows of Matrix I are numbered from 0 to 4 and that of
Matrix II are numbered from 5 to 9. A letter from these matrices can be
represented first by its row and next by its column, e.g., 'A' can be
represented by 00,44,etc., and 'I' can be represented by 56, 79 etc.
Similarly, you have to identify the set for the word ZEST.

(a) 89, 13, 03, 01


(b) 98, 13, 30, 10
(c) 89, 31, 30, 01
(d) 98, 13, 33, 04
25. Which answer figure will complete the pattern in the question figure?
PART-B : NUMERICAL APTITUDE
26. P invests ₹ 9100, for 3 months, Q invests ₹ 6825 for 2 months and R ₹
8190 for 5 months in a business, If the total profit amounts to ₹ 4158,
how much profit should Q get?
(a) ₹ 346.50
(b) ₹ 693
(c) ₹ 682.50
(d) ₹ 1386
27. A contractor has the target of completing a work in 40 days. He
employed 20 persons who completed (1/4) of the work in 10 days and
left. The number of persons he has to employ to finish the remaining
part as per target is:
(a) 20
(b) 40
(c) 30
(d) 10
28. The average of three numbers of which greatest is 16 is 12. If the
smallest is half of the greatest, the remaining number is:
(a) 12
(b) 14
(c) 8
(d) 10
29. A person covers a certain distance in 6 hours, if travels at 40 km/hour.
If he has to cover the same distance in 4 hours, then his speed must be:
(a) 50 Km/hour
(b) 80 Km/hour
(c) 70 Km/hour
(d) 60 Km/hour
30. By selling a coat for ₹ 630, a shopkeeper gains 5%. Find the cost price
of the coat?
(a) ₹ 650
(b) ₹ 625
(c) ₹ 600
(d) ₹ 700
31. Two varieties of sugar are mixed together in a certain ratio. The cost of
the mixture per Kg is ₹ 0.50 less than that of the superior and ₹ 0.75
more than the inferior variety. The ratio in which the superior and
inferior varieties of sugar have been mixed is:
(a) 2 : 3
(b) 5 : 1
(c) 5 : 2
(d) 3 : 2
32. P, Q and R can do a piece of work in 60 days, 100 days and 80 days
respectively. They together work to finish the work and receive ₹ 2256.
Then P will get?
(a) ₹ 960
(b) ₹ 576
(c) ₹ 564
(d) ₹ 752
33. A box contains one rupee, fifty-paise and twenty-five paise coins. The
total number of coins are 378. The ratio of values of the above coins is
13 : 11 : 7. The number of twenty-five paise coins was:
(a) 132
(b) 78
(c) 210
(d) 168
34. The Selling price of a radio was ₹ 255 when 15% discount was allowed.
Then the marked price of the radio was?
(a) ₹ 400
(b) ₹ 300
(c) ₹ 350
(d) ₹ 275
35. The unit's digit of the number 6256 – 4256 is:
(a) 1
(b) 4
(c) 0
(d) 7
36. Ram's income is ₹ 100 more than that of Shyam. If the average income
of Ram and Shyam is ₹ 850, then Ram's income is:
(a) ₹ 900
(b) ₹ 800
(c) ₹ 850
(d) ₹ 475
37. In a college, 1/5th of the girls and 1/8th of the boys took part in a social
camp. The total number of students in the college took part in the camp
is:

(a)

(b)

(c)

(d)

38. Each side of a square is increased by 10%. The percentage increase of


its area is:
(a) 20
(b) 25
(c) 21
(d) 12.5
DIRECTIONS (Qs. 39 and 40): The pie-diagram shows the expenditure
incurred on the printing of a book by a publisher, under various heads. Study
the pie-diagram and answer the questions.

39. If the expenditure incurred on the printing under head A is ₹ 5000/-.


Then the sum of expenditure incurred under head B and D is:
(a) ₹ 5780
(b) ₹ 7850
(c) ₹ 8750
(d) ₹ 8570
40. The two expenditures, which together will form an angle of 108° at the
centre are:
(a) A & D
(b) D & E
(c) A & E
(d) B & E
41. A two wheeler depreciates at 20% of its value every year. If present
value of the same be ₹ 90000, its depreciated value would be ₹ 36864
after:
(a) 3 years
(b) 6 years
(c) 4 years
(d) 5 years
42. A sum was doubled with 12½% rate of simple interest, per annum.
Then time taken for that sum is:
(a) 10 years
(b) 8 years

(c)

(d)

43. The price of a bicycle is marked by a trader at ₹ 1000. He sold the


bicycle allowing successive discounts of 20%, 10% and 5%. Thus the
trader gained 14% then the cost price of bicycle (in rupees) is:
(a) 790
(b) 600
(c) 510
(d) 560
44. The price of electricity has been increased by 25%. If a person wants to
keep the expenditure same, then the percentage reduction in use of
electricity should be:
(a) 19
(b) 20
(c) 21
(d) 18
45. A dealer marks his goods 20% above cost price. He then allows some
discount on it and makes a profit of 8%. The rate of discount is:
(a) 6%
(b) 10%
(c) 4%
(d) 12%
46. The radius of a sphere is doubled. The percentage of increase in its
surface area is?
(a) 100
(b) 400
(c) 75
(d) 300
47. The ratio of the areas of two squares one having double its diagonal
than the other is?
(a) 3 : 2
(b) 2 : 1
(c) 4 : 1
(d) 3 : 1
48. A man travels 600 Km by train at 80 Km/hour, 600 Km by ship at 30
Km/hour, 500 Km by aeroplane at 400Km/hour and 300 Km by car at
60Km/hour. What is the average speed (Km/hour) for the entire
distance?
(a) 63

(b)

(c)

(d)

49. A man retired from his service at the age of 60. He served for 3/5th years
of his retirement age. He joined his job at the age of:
(a) 20 years
(b) 18 years
(c) 24 years
(d) 36 years
50. A man purchased 120 reams of paper at ₹ 80 per ream. He spent ₹ 280
on transportation, paid octroi at the rate of 40 paise per ream and paid ₹
72 to a porter. In order to gain 8%, he must sell each ream of paper for:
(a) ₹ 90
(b) ₹ 85
(c) ₹ 89
(d) ₹ 87.5

PART-C : GENERAL ENGLISH

DIRECTIONS (Qs. 51 to 60): Some part of the sentences may have errors
and some are correct. Find out which part of the sentence has an error and
blacken the circle corresponding to the appropriate correct option. If a
sentence is free from error, blacken the circle corresponding to "No Error"
option in the Answer Sheet.
51. This sponge is not able to absorbing water.
(a) This sponge is
(b) No error
(c) Not able to
(d) Absorbing water.
52. I am thinking I will go to the marked later this afternoon.
(a) I am thinking
(b) No error
(c) Later this afternoon
(d) I will go to the market
53. I am too much pleased to know that you have topped the list.
(a) to know that
(b) you have topped the list
(c) No error
(d) I am too much pleased
54. Have you been doing what has been asks of you?
(a) Have you
(b) been doing
(c) what has been asks of you
(d) No error
55. We rejoiced at his being promoted.
(a) being promoted
(b) No error
(c) We rejoiced
(d) at his
56. Kindly submit your documents to the clerk.
(a) to the clerk
(b) No error
(c) your documents
(d) Kindly submit
57. It is a most beautiful painting in the gallery.
(a) It is a most
(b) No error
(c) beautiful Painting
(d) in the gallery.
58. Love for ours environments is enough to protect nature.
(a) No error
(b) ours environment
(c) is enough to protect nature
(d) Love for
59. He is not what you would call an honest man, doesn't he?
(a) He is not what
(b) you would call an honest man
(c) doesn't he?
(d) No error
60. He has not been attending English classes for one week.
(a) He has not been attending.
(b) English classes
(c) for one week
(d) No error
DIRECTIONS (Qs. 61 to 70): Sentences given with blank to be filled in with
appropriate word(s). Four alternatives are suggested for each question.
Choose the most appropriate alternative out of the four and indicate it by
blackening the corresponding circle in the Answer Sheet.
61. Since she is a teacher of language, one would not expect her to be guilty
of a ——.
(a) solecism
(b) bornbast
(c) schism
(d) stanchion
62. Anna is not popular. She has —— friends.
(a) a little
(b) a few
(c) few
(d) little
63. Sally parked and got —— the car quickly.
(a) out of
(b) on
(c) in
(d) over
64. —— she can't drive, Anita has bought a car.
(a) Since
(b) Even though
(c) Whether or not
(d) Even if
65. You will have to face some practical problems when you start —— this
plan.
(a) prosecuting
(b) proscribing
(c) prescribing
(d) executing
66. They were worried about a new cult which —— many followers.
(a) must be gaining
(b) were gaining
(c) was gaining
(d) is gaining
67. The actor died when his car turned ——
(a) turtle
(b) tortoise
(c) across
(d) tiger
68. A man is know —— the company he keeps.
(a) of
(b) by
(c) for
(d) in
69. He must —— the loans.
(a) repeal
(b) repay
(c) repeat
(d) receipt
70. The convict was —— on a Monday.
(a) hanging
(b) hanged
(c) hang
(d) hung
DIRECTIONS (Qs. 71 to 73): Out of the four alternatives, choose the one
which best expresses the meaning opposite to the given word.
71. Authentic
(a) fake
(b) honest
(c) frank
(d) futile
72. Methodical
(a) orderly
(b) Hurry
(c) random
(d) crazy
73. Polite
(a) rude
(b) rough
(c) tough
(d) rash
DIRECTIONS (Qs. 74 to 78): Out of the four alternatives, choose the one
which can most appropriately substitute the given word(s) or sentence.
74. In a primitive or uncivilized state.
(a) rural
(b) savage
(c) village
(d) olden
75. Happening every year.
(a) annually
(b) routine
(c) timely
(d) season
76. One who believes in God.
(a) pilgrim
(b) believer
(c) theist
(d) worshipper
77. All in a mess.
(a) disturbing
(b) disarranged
(c) negligence
(d) distracting
78. A number of people listening to a concert of lecture.
(a) group
(b) audience
(c) tribe
(d) spectator
DIRECTIONS (Qs. 79 to 83): A sentence of a part of the sentence is
underlined. Below are given alternatives to the underlined part which may
improve the sentence. Choose the correct alternative. In case no
improvement is needed choose "No Improvement".
79. How long I shall stay is doubtful.
(a) I can stay
(b) I will stay
(c) I want to stay
(d) No improvement
80. Even at the peak of her success, Kalpana Chawla did not forget her
home or her duty towards other human, this is evident from the fact that
she distributed education of girls in India.
(a) she contribute to education of girls.
(b) she contributed girls for education.
(c) No improvement.
(d) she contributed to the education of girls.
81. Every person was not fitted for everything.
(a) is not fit at
(b) No improvement
(c) was not fit at
(d) is not fit for
82. The doctor scribbled his prescription which I could not make out.
(a) make over
(b) make up
(c) make for
(d) No improvement
83. Neither parties deserves our help.
(a) No improvement
(b) parties deserved
(c) party deserves
(d) parties deserve
DIRECTIONS (Qs. 84 to 86): Out of the four alternatives, choose the one
which best expresses the meaning of the given word.
84. Stationery.
(a) Writing accessories
(b) Computer accessories
(c) Moving
(d) Fixed
85. Abhorrent.
(a) Irregularity
(b) Repugnant
(c) Admirable
(d) Uncommon
86. Innocent.
(a) Guilt
(b) Bias
(c) Offence
(d) Guiltless
DIRECTIONS (Qs. 87 to 92): Four words are given, out of which only one
world is correctly spelt, Find the correctly spelt word.
87. (a) municiple
(b) muneciple
(c) munecipal
(d) municipal
88. (a) journal
(b) jornal
(c) jornale
(d) jurnale
89. (a) license
(b) liscense
(c) lisense
(d) licanse
90. (a) harrass
(b) harass
(c) harras
(d) haraas
91. (a) saperate
(b) separete
(c) separate
(d) seperate
92. (a) exhilarate
(b) exhelerate
(c) exhilerate
(d) exelerate
DIRECTIONS (Qs. 93 to 95): Four alternatives are given for the
idiom/Phrase underlined in the sentence. Choose the alternatives which best
expresses the meaning of the idiom/Phrase.
93. The presentation by the group at the meeting passes muster.
(a) achieves to impress
(b) meets required standards
(c) does not meet the required standards.
(d) fails to impress.
94. All and sundry were asked to leave the village immediately.
(a) some people
(b) all the people
(c) all the adults and elderly
(d) all the adults
95. Joe can be a loose canon sometimes.
(a) one who acts without thinking.
(b) one who speaks without thinking.
(c) one who acts crazy.
(d) one who is unpredictable
DIRECTIONS (Qs. 96 to 100): Read the following passage carefully and
choose the most appropriate answer to each question out of the four
alternatives.
Passage: At times there was something inhuman about Mr. Rogers, the
Headmaster. His formidable chin fitted out in the most formidable way and
he seemed to be always frowning at the world in general. At the assembly he
was like an inspecting officer in the army; he surveyed the gathered innocents
with his crinkled eyes, and then began his list of morning rebukes. Any
hapless offender, for the most venial offence, would be commanded to appear
before his presence in front of the gathered ranks, and would then be
subjected to a tirade of abuse that would leave him trembling or even in tears.
Mr. Rogers was ruthless and cruel in public, "A dehydrated old sadist" was
how Mr. Jones, the chemistry master described him. The extraordinary thing
was that in private he could be magnanimous and gentle. Any boy who was
really in trouble would receive a sympathetic hearing and went out feeling
that the burden had been lightened.
96. What best describes Mr Rogers at the school assembly?
(a) ruthless and cruel
(b) a sensitive officer
(c) a formidable teacher
(d) a genial person
97. The staff considered Mr Rogers to be ——
(a) an intimidating person
(b) a person to be idolized
(c) a person without any scruples
(d) a gentle and understanding person
98. "in a most intimidating way" means.
(a) in a very fowl way
(b) in a very helpless way
(c) in an extremely intimate manner
(d) in an extremely frightening manner
99. Another word for 'magnanimous' is:
(a) generous
(b) high handed
(c) magnificent
(d) arrogant
100. What happened to boys who visited him in private?
(a) They came out trembling and in tears.
(b) They looked even more worried than before.
(c) They echoed the sentiments of the staff members.
(d) They seemed relieved of their troubles.

PART-C : GENERAL AWARENESS


101. In earth atmosphere, which of the following continuously decreases
with height?
(a) Wind velocity
(b) Temperature
(c) Pressure
(d) Humidity
102. Which state in India has the largest coastline?
(a) Andhra Pradesh
(b) Gujrat
(c) Tamil Nadu
(d) Maharashtra
103. The crop Development Programme of the Government of India covers
which of the following groups of commercial crops?
(a) Tea, Cotton and Rubber
(b) Jute, Tea and Coffee
(c) Cotton, Jute and Sugarcane
(d) Tea, Coffee and Spices
104. Under the Constitution of India who are the ultimate sovereign?
(a) president of India
(b) Indian People
(c) Prime Minister of India
(d) All elected leaders of India
105. Which of the following does not belong to physical environment?
(a) Lithosphere
(b) Atmosphere
(c) Hemisphere
(d) Hydrosphere
106. An example of a solution of liquid in solid is:
(a) jelly
(b) rubber
(c) foam
(d) smoke
107. Which one of the following is not correctly matched?
(a) Iron-ore - Kudremukh
(b) Copper - Khetri
(c) Manganese - Koraput
(d) Coal - Singreni
108. Which was the instrument that was played by Ustad Bismillah Khan?
(a) Sitar
(b) Shehnai
(c) Santoor
(d) Flute
109. Loss of water in plants in the form of liquid is know as:
(a) Osmosis
(b) Imbibition
(c) Transpiration
(d) Guttation
110. Which memory is both static and non-volatile?
(a) RAM
(b) CACHE
(c) ROM
(d) BIOS
111. The Modi Government at the Centre has formed NITI AAYOG by
replacing which one of the following?
(a) Finance Commission
(b) Planning Commission
(c) Agricultural Price
(d) Fiscal Commission
112. Poverty in India is measured through one of the following criteria:
(a) number of rooms of the residence
(b) intake of the calories
(c) education level of the family
(d) number of family members
113. The Attorney General of India is appointed by the:
(a) Prime Minister
(b) Law Minister
(c) President of India
(d) Home Minister
114. Which of the following can Not act as a bleaching agent?
(a) Nitrous oxide
(b) Sulpher dioxide
(c) Chlorine
(d) Hydrogen peroxide
115. Which table in an operating system contains information about all the
open files?
(a) open file table
(b) open-seek table
(c) open table
(d) open-location table
116. The aim of the Antyodaya Programme is
(a) Uplifting minorities
(b) Eliminating urban
(c) Improving the standards of scheduled castes
(d) Helping the poorest among the poor
117. Which of the following states has recently launched the Annapurna
Rasoi Yojna for providing quality meals to weaker sections at
subsidised rates?
(a) Uttar Pradesh
(b) West Bengal
(c) Madhya Pradesh
(d) Rajasthan
118. For Commercial Banks lending the 'Priority Sector' denotes:
(a) Self employed
(b) Small industrialists
(c) Agriculture and small scale industries
(d) Small farmers
119. The toxic metal associated with the Minamata episode is:
(a) cadmium
(b) lead
(c) mercury
(d) arsenic
120. The release of which of the following into ponds and wells helps in
controlling the mosquitoes?
(a) Crab
(b) Dogfish
(c) Snail
(d) Gambusia fish
121. Sun Yat Sen was the leader of:
(a) Taiping Rebellion
(b) Boxer Revolt
(c) C P C Party
(d) K M T Party
122. A biome is
(a) A group of plants growing in a particular area
(b) A delimited area
(c) A complex of communities characterised by distinctive climate &
vegetation
(d) A collection of rare plants & animals
123. The lens of the eye is behind the:
(a) Pupil
(b) Vitreous human
(c) Retina
(d) Optic nerve
124. Which of the following income will not be included in national income
calculation?
(a) Income of a joker in a circus company
(b) Income of a commercial artist
(c) Commission income of a share broker
(d) Individual's income from sale of shares
125. Which one of the following states is known as 'Garden of Spices'?
(a) Kerala
(b) Karnataka
(c) Gujrat
(d) Tamil Nadu
126. Pillared halls where the Buddhist monks worshipped were known as:
(a) Chaityas
(b) Mathas
(c) Viharas
(d) Stupas
127. Which of the following is a semiconductor:
(a) silver
(b) copper
(c) glass
(d) Silicon
128. Milkiness observed on passing carbon dioxide through lime water is
due to the formation of:
(a) Calcium carbonate
(b) Calcium chloride
(c) Calcium hydroxide
(d) Calcium bicarbonate
129. Elasticity of demand of luxury goods is:
(a) Less than one
(b) Infinite
(c) Zero
(d) More than one
130. A system program designed to aid the programmer in finding and
correcting errors or bugs is know as:
(a) Debugger
(b) Evaluator
(c) Currector
(d) Quarantiner
131. Both the Tropic of Cancer and the Tropic of Capricorn pass through
(a) South America
(b) Asia
(c) Africa
(d) Australia
132. Who among the following built the Golden Temple at Amritsar?
(a) Guru Gobind Singh
(b) Guru Teg Bahadur
(c) Guru Nanak
(d) Guru Arjun Dev
133. Who has been elected as the new Secretary General of the United
Nations?
(a) Antonio Guterras
(b) Ban-ki-moon
(c) Lewis Hamilton
(d) Juan Manuel Santos
134. After a shower of rain, a rainbow is seen:
(a) anywhere irrespective of the position of the sun
(b) even in the absence of the sun
(c) towards the sun
(d) opposite the sun
135. Who decides about the size and membership of the Council of
ministers?
(a) Lok Sabha
(b) Chairman of the Rajya Sabha
(c) Prime Minister
(d) President
136. Varun Singh Bharti who won bronze at 2016 Rio Paralympics Games is
associated with which sports?
(a) Badminton
(b) Shot Put
(c) High Jump
(d) Javelin Throw
137. The maximum number of Mongol invasions took place during the reign
of:
(a) Muhammad bin Tughluq
(b) Balban
(c) Firoz Tughluq
(d) Alauddin Khilji
138. Microsoft Excel is a
(a) Graphics Package
(b) MS Office Package
(c) Electronic Spreadsheet
(d) Financial Planning Package
139. In the Panchayat Raj System, "the Panchayat Samiti" is constituted at
the:
(a) State level
(b) District level
(c) Block level
(d) Village level
140. Participation is an important element of every:
(a) Aristocratic system
(b) Oligarchical system
(c) Democratic system
(d) Monarchical system
141. The Kailasanatha Temple (Siva Temple) at Kanchipuram containing
sculptures with painting in the inner wall, was built by?
(a) Narasimhavarman
(b) Ravivarman
(c) Mahendravarman
(d) Devavarman
142. Element that is not found in blood is:
(a) chromium
(b) copper
(c) magnesium
(d) iron
143. Govind Ballabh Pant Sagar reservoir is situate in:
(a) Jharkhand
(b) Uttar Pradesh
(c) Chhattisgarh
(d) Uttarakhand
144. The teeth lacking in Herbivorous animals is:
(a) Molar
(b) Canine
(c) Incissor
(d) premolar
145. Of the following which one is non-contractual earning?
(a) Profit
(b) Interest
(c) Rent
(d) Wage
146. Which one of the following gases released from bio-gas plant is used as
a fuel gas?
(a) Ethane
(b) Butane
(c) Propane
(d) Methane
147. The title of 'Gangai Kondan' was adopted by the following Chola King:
(a) Vijayalaya
(b) Karikala
(c) Rajaraja I
(d) Rajendra I
148. Volt×Second is the unit of:
(a) Inductance
(b) Resistance
(c) Capacitance
(d) Energy
149. Excretory organ of cockroach is:
(a) Nephridia
(b) Malpighian tubules
(c) Kidney
(d) Malpighian Corpuscles
150. The number of players in a Futsal team is
(a) 8
(b) 6
(c) 5
(d) 9
1. (d) RATE cannot be formed as letter 'A' is not present in the given
word.
2. (c) 9 + 8 ÷ 8 – 4 × 6 → 9 × 8 + 8 ÷ 4 – 6
= 72 + 2 – 6 = 68

3. (a)

Similarly
4. (a)

5. (b)

6. (c) He is finally facing in north diretion.

7. (b) There are 18 triangles in all


8. (b) 83 → 512
93 → 729
9. (c)

10. (c) Pen is used by an author.


Similarly, stethoscope is used by a doctor.

11. (b)

Letters are written in reverse order.


So, SINGLE : ELGNIS is the correct option.

12. (a)

Only conclusion I follows.


13. (b)

Similarly, 5 × 4 × 1 = 145
14. (d) Option (d) is the correct figure.
15. (a) Option (a) is the mirror image of the given figure.
16. (a) Kilograms, tonnes and quintals are units of mass. Kilometer is a
unit of distance.
17. (a)

So, JKNQ is the correct option.


18. (a) 154. All other numbers are multiples of 17.
19. (a) Option (a) is correct.
20. (c) Correct order is :

21. (d) Let age of elder son be x years.


Let age of younger son be y years.
Then, x – y = 15 ⇒ y = x – 15
Age of father = 2x
2x + 10 = 3 (y + 10)
⇒ 2x + 10 = 3 (x – 5) ⇒ x = 25
Hence, age of father will be 2x i.e. 50 years.

22. (c)

23. (d) Mango and Banana are two different types of fruits.
24. (b) From the given matrices, we can find that

So, correct set is 98, 13, 30, 10


25. (d) The option (d) is the missing figure.
26. (b) Amount invested by P = `9100 × 3 = `27300
Amount invested by Q = `6825 × 2 = `13650
Amount invested by R = `8190 × 5 = `40950
Total amount invested by P + Q + R = `27300 + `13650 + `40950
= `81900
Q's share of profit =

= ` 693
27. (a) As 20 persons complete of work in 10 days.

Let N persons complete the remaining of work in 30 days.

Then from,

⇒ N = 20
28. (a) Greatest number = 16
Smallest number = =8
Total sum of three numbers = 12 × 3 = 36
Third number = 36 – 16 – 8 = 12
29. (d) Distance covered = 6 × 40 = 240 km.
To cover the same distance in 4 hours
speed = = 60 km/hour

30. (c) CP = = = ` 600

31. (d) Let the cost of superior quality be ` x/kg.


Let the cost of inferior quality be ` y/kg.
Let a kg of superior and b kg of inferior quality are mixed.
Then = x – 0.5

⇒ ax + by = ax + bx – 0.5 (a + b)
⇒ b(x – y) = 0.5 (a + b). ...(1)Also,
= y + 0.75

⇒ ax + by = ay + by + 0.75 (a + b)
⇒ a (x – y) = 0.75 (a + b) ...(2)
Dividing (2) by (1), we get
or a : b = 3 : 2

32. (a) P's one day work =

Q's one day work =

R's one day work =

Ratio of their efficiency

or, P : Q : R = 20 : 12 : 15
∴ P's share of profit = ` 960

33. (d) Let the ratio of their values be x

Then, the values of coins are :

Number of coins:

⇒ 13x + 22x + 28x = 378


⇒ 63x = 378 ⇒ x = 6
Hence the number of 25 P coins = 28x = 28 × 6 = 168.
34. (b) M.P. – 15% of M.P. = ` 255
⇒ M.P. = `255

⇒ M.P. × = `255

⇒ M.P. = 255 × = `300

35. (c) We know that for any power of 6, its unit digit always ends with 6.
For odd power of 4, unit's digit will ends with 4 and for even
power of 4, unit digit ends with 6
So, 6256 – 4256 units digit will be 6 – 6 = 0
36. (a) Let Shyam's income be ` x. Then Ram's income = (x + 100)
Their total income = 2 × ` 850 = ` 1700
⇒ x + x + 100 = 1700
⇒ 2x = 1600 ⇒ x = 800
Therefore, Ram's income = ` 800 + ` 100
= ` 900
37. (b) Out of 5 girls, 1 girl took part in the camp.
Also, out of 8 boys, 1 boy took part in the camp.
So, out of (5 + 8) = 13 students in the class (1 + 1) = 2 students
took part in the camp.
So, total number of students, who took part in the camp

38. (c) Let a be the side of the square. Then, Area = a2


New area = (a + 10% of a)2

Increase in area =

= 21%
39. (c) 20% = ` 5000
1% = ` = `250

Sum of expenditure incurred under head B and D


= 25% + 10% = 35% = 35 × ` 250 = ` 8750
40. (a) Angle formed by A = 20% of 360° = 72°
Angle formed by B = 25% of 360° = 90°
Angle formed by C = 30% of 360° = 108°
Angle formed by D = 10% of 360° = 36°
Angle formed by E = 15% of 360° = 54°
So, A and D together form an angle of 108°
41. (c) Let n years be the required time. Then,

36864 = 90000

⇒ ⇒n=4
42. (b) Here, I = P, R = = ,T=?

I= ⇒P= ⇒ T = 8 years

43. (b) Price after Ist discount = 1000 – 20% of 1000 = ` 800
Price after 2nd discount = ` 800 – 10% of ` 800 = ` 720
Final price after 3rd discount = ` 720 – 5% of `720
= ` 684
CP =

= = ` 600

44. (b) Let ` 100 be the cost of electricity and x be the consumption.
Then,
expenditure = 100x
New cost of electricity = ` 125
Let y be the new consumption. Then
100 x = 125 y
⇒y= ⇒y=

or y = % of x ⇒ y = 80% of x

Hence, reduction in consumption = 20%


45. (b) Let CP be ` x. Then,
M.P. = x +

Let R be the rate of discount. Then


S.P. = =

Profit % = 8% of x =
C.P. =

⇒x=

⇒ ⇒

⇒ ⇒ R = 10

46. (d) Surface area of a sphere of radius r = 4πr2


Surface area of a sphere of radius 2r = 4π(2r)2 = 16πr2
Increase in area = 16πr2 – 4πr2 = 12πr2
% increase = = 300%

47. (c) Let a be the side of the Ist square


Diagonal = =
Area = a2
Let a' be the side of 2nd square
Diagonal =
⇒ a' = 2a
Area = (2a)2 = 4a2
Required ratio = 4a2 : a2 = 4 : 1
48. (d) t1 = hours = horus

t2 = = 20 hours

t3 = = hours
t4 = = 5 hours

Total time taken = hours

= hours

Total distance travelled = (600 + 600 + 500 + 300) km


= 2000 km
Average speed = =

km/h

49. (c) Period of service = = 36 years

Joining age = 60 years – 36 years = 24 years


50. (a) Total cost of 80 reams of paper
= 120 × 80 + 280 +

= 9600 + 280 + 48 + 72 = ` 10000


Gain % = 8%
S.P. of 120 reams = ` 10000 + 8% of ` 10000
= ` 10800
S.P. of 120 reams = ` 10800
S.P. of 1 ream = ` = `90

51. (d) 'absorbing' should be replaced with 'absorb' to make the sentence
grammatically correct.
52. (a) 'am thinking' should be replaced with 'think' to make the sentence
grammatically correct.
53. (d) 'much' should be removed as it is superfluous in the context of the
sentence.
54. (c) 'asks' should be replace with 'asked' to make the sentence
grammatically correct.
55. (b) No error
56. (b) No error
57. (a) 'a' should be replaced with 'the' to make the sentence
grammatically correct.
58. (b) 'ours' should be replaced with 'our' to make the sentence
grammatically correct.
59. (c) The question tag 'doesn't he?' should be replaced with 'is he?' to
make the sentence grammatically correct.
60. (c) 'for' should be replaced with 'since' to make the sentence
grammatically correct.
61. (b) bombast 62. (c) few
63. (a) out of 64. (b) Even though
65. (d) executing 66. (c) was gaining
67. (c) across 68. (b) by
69. (b) repay 70. (b) hanged
71. (a) fake 72. (c) random
73. (a) rude 74. (b) savage
75. (a) annually 76. (c) theist
77. (b) disarranged 78. (b) audience
79. (d) No improvement
80. (d) she contributed to the education of girls
81. (d) is not fit for 82. (d) No improvement
83. (c) party deserves 84. (d) Fixed
85. (b) Repugnant 86. (d) Guiltless
87. (d) 88. (a)
89. (a)
90. (b)
91. (c)
92. (a)
93. (b)
94. (b)
95. (d)
96. (a) It can easily be inferred from the passage.
97. (c) It can easily be inferred from the passage.
98. (d)
99. (a) generous
100. (d) It is given in the last paragraph of the passage.
101. (c) Atmospheric pressure decreases with increasing height. Since most
of the atmosphere's molecules are held close to the earth's surface
by the force of gravity, air pressure decreases rapidly at first, then
more slowly at higher levels.
102. (b) Gujarat is strategically located with largest share in India's
coastline, followed by Andhra Pradesh and Tamil Nadu.
103. (c) Cotton, Jute and Sugarcane are amongst the main cash crops of
India and are covered by the Crop Development Programme of the
Government of India.
104. (b) The constitution declares that the sovereignty lies with the people
and the constitution is the embodiment and solemn expression of
the will of the people.
105. (c) Hemisphere refers to the half of the sphere and it does not belong
to physical environment of the earth.
106. (a) Some of the examples of liquid in solid colloid are Butter, Cheese,
Jelly, Paint, etc.
107. (c) The principal economic mineral deposits of Koraput district are
Bauxite and Limestone.
108. (b) Ustad Bismillah Khan (born as Qamaruddin Khan) was an Indian
musician credited with popularizing the shehnai, a sub continental
wind instrument of the oboe class. He was awarded India's highest
civilian honour, the Bharat Ratna, in 2001.
109. (d) The process by which plants loose water in liquid form is called
Guttation; on the other hand, if the water is lost from the plant
body in the form of water vapours, then the phenomenon is called
Transpiration.
110. (c) Read-only memory (ROM) is a type of non-volatile memory used
in computers and other electronic devices. Data stored in ROM can
only be modified slowly, with difficulty, or not at all, so it is
mainly used to store firmware or application software in plug-in
cartridges.
111. (b) Union Government established NITI Aayog (National Institution
for Transforming India) on January 1, 2015, as a replacement for
the Planning Commission.
112. (b) A common method used to estimate poverty in India is the income
or consumption method. If the income or consumption falls below
a given minimum level, then the family is said to be below the
poverty line. A minimum level of food requirement, clothing, fuel,
medical requirement is used to determine subsistence.
113. (c) The Attorney General for India is appointed by the President of
India under Article 76(1) of the Constitution of India and holds
office during the pleasure of the President.
114. (a) The most common bleaching agents generally fall into two
categories: chlorine and its related compounds (such as sodium
hypochlorite) and the peroxygen bleaching agents, such as
hydrogen peroxide and sodium perborate. Sulphur Dioxide
bleaches by reduction and is temporary. Nitrous Oxide is not a
bleaching agent.
115. (a) Open-File Table. When two or more processes open a file for
reading, there's an entry in the open file table per open. There is
even an entry per open if one process opens the file multiple times.
116. (d) Antyodaya Programme is a Government of India sponsored
scheme which is aimed at the upliftment of the poorest of the poor
families.
117. (d) Rajasthan Government has launched Annapurna Rasoi Scheme to
provide quality meal at cheap price to poor and needy on lines with
Tamil Nadu Government's Amma canteens.
118. (c) Priority Sector Lending is an important role given by the Reserve
Bank of India (RBI) to the banks for providing a specified portion
of the bank lending to few specific sectors like agriculture and
allied activities, micro and small enterprises, poor people for
housing, students for education and other low income groups and
weaker sections.
119. (c) Minamata disease was first discovered in Minamata city in
Kumamoto prefecture, Japan, in 1956. It was caused by the release
of methylmercury in the industrial wastewater from the Chisso
Corporation's chemical factory.
120. (d) Gambusia fish is a species of freshwater fish, also known as
western mosquitofish and a part of its diet consists of mosquito
larvae. Releasing Gambusia fish into ponds and wells help in
controlling of mosquitoes.
121. (d) Sun Yat Sen , the founding father of the Republic of China, was
the leader of the Kuomintang (KMT) Party.
122. (c) A biome is a community of plants and animals that have common
characteristics for the environment they exist in. They can be found
over a range of continents. Biomes are distinct biological
communities that have formed in response to a shared physical
climate.
123. (a) The lens is composed of transparent, flexible tissue and is located
directly behind the iris and the pupil. It is the second part of your
eye, after the cornea that helps to focus light and images on your
retina.
124. (d)
125. (a) Kerala has been a major spice exporter since 3000 BCE, according
to Sumerian records and it is still referred to as the "Garden of
Spices" or as the "Spice Garden of India". Kerala's spices attracted
ancient Babylonians, Assyrians and Egyptians to the Malabar
Coast in the 3rd and 2nd millennia BCE.
126. (a) A Chaitya is a pillared Buddhist shrine or prayer hall with a stupa
at one end.
127. (d) Materials that have the resistance levels between those of a
conductor and an insulator are referred to as semiconductors. They
are quite common, found in almost all electronic devices. Good
examples of semiconductor materials are germanium, selenium,
and silicon.
128. (a) When CO2 is passed through lime-water, it turns milky due to the
formation of an insoluble precipitate of Calcium Carbonate.
Reaction goes like this:
Ca (OH)2 + CO2(g) → CaCO3(s) + H2O(l)
129. (d) If income elasticity of demand of a commodity is less than 1, it is a
necessity good. If the elasticity of demand is greater than 1, it is a
luxury good or a superior good.
130. (a) Debugger is a computer program that assists in the detection and
correction of errors in other computer programs.
131. (c) Africa is the only country through which both the Tropic of Cancer
and Tropic of Capricorn passes through.
132. (d) Guru Arjan Sahib got Golden Temple's foundation laid by a
muslim saint Hazrat Mian Mir ji of Lahore. The construction work
was directly supervised by Guru Arjan Sahib himself and he was
assisted by the prominent Sikh personalities like Baba Budha ji,
Bhai Gurdas ji, Bhai Sahlo ji and many other devoted Sikhs.
133. (a) Antonio Guterres, former Prime Minister of Portugal and United
Nations High Commissioner for Refugees is the incumbent
secretary general of United Nations General Assembly.
134. (d) Rainbows caused by sunlight always appear in the section of sky
directly opposite the sun.
135. (c) The Prime Minister determines the size and membership of the
Council of Ministers. However, it cannot exceed 15% of the total
membership of Lok Sabha.
136. (c) Varun Singh Bhati is a Para High Jumper from India. He has won
bronze at 2016 Summer Paralympic Games and 2017 World Para
Athletics Championships. He received Arjuna Award in 2017 for
Para Athletics.
137. (d) The maximum number of Mongol invasions were faced by
Alauddin Khilji. He built Siri his capital mainly to deter the
Mongols.
138. (c) Microsoft Excel is an electronic spreadsheet developed by
Microsoft for Windows, macOS, Android and iOS. It features
calculation, graphing tools, pivot tables, and a macro programming
language called Visual Basic for Applications.
139. (c) Panchayati Raj System is divided into three levels: 1) Gram
Panchayat at village level; 2) Mandal Parishad or Panchayat Samiti
at Block level, and 3) Zila Parishad at District level.
140. (c) Etymological roots of democracy imply that the people are in
power and thus that all democracies are participatory. Participatory
democracy emphasizes the broad participation of constituents in
the direction and operation of political systems.
141. (a) Located in Tamil Nadu, India, Kailasanath Temple is a Hindu
temple in the Dravidian architectural style. It is dedicated to Lord
Shiva, and is known for its historical importance. The temple was
built from 685-705AD by a Rajasimha (Narasimhavarman II) ruler
of the Pallava Dynasty.
142. (a) Chromium is the element which is not found in the blood.
143. (b) Rihand Dam, also known as Govind Ballabh Pant Sagar, is the
largest dam of India by volume. The reservoir of Rihand Dam is
India's largest artificial lake. Rihand Dam is a concrete gravity dam
located at Pipri in Sonbhadra District in Uttar Pradesh, India.
144. (b) All animals have teeth that are adapted to eating certain types of
food. For instance, herbivores, because they are plant eaters, have
strong and flat molars that are made for grinding leaves and non-
existent canine teeth.
145. (a) Contractual income is income paid on a contractual basis. The
income is intended to cover a specific period of time. Rent, Interest
and Wage, all are intended to cover a specific period of time. Same
does not apply to Profit.
146. (d) When organic matter, such as food scraps and animal waste, break
down in an anaerobic environment, they release a blend of gases,
primarily methane and carbon dioxide. Methane is important for
electricity generation by burning it as a fuel in a gas turbine or
steam generator. In many cities, methane is piped into homes for
domestic heating and cooking.
147. (d) After his successful campaign to Ganges river in North India,
Rajendra Chola I adopted the title of 'Gangaikonda Cholapuram',
literally meaning, the Chola who took the Ganges river.
148. (d)
149. (b) The excretory organ of cockroach is the malpighian tubules. It is
found at the junction of the midgut and hind gut and is about 150 in
number. They are fine, syellow coloured and branched threads
present in bundles. They lie freely in the haemolymph.
150. (c) Futsal is played between two teams of five players each, one of
whom is the goalkeeper. Unlike some other forms of indoor
football, the game is played on a hard court surface delimited by
lines; walls or boards are not used.
PART-A : GENERAL INTELLIGENCE AND REASONING
DIRECTIONS: In question no. 1 and 2, a series is given, with one term
missing. Choose the correct alternative from the given ones that will
complete the series.
1. 7, 14, 23, 34, ?
(a) 46
(b) 47
(c) 44
(d) 45
2. AE, FJ, KO, ? UY
(a) QN
(b) TQ
(c) NP
(d) PT
3. From the given alternatives select the word which cannot be formed
using the letters of the given word.
DEPARTMENT
(a) PEN
(b) PARK
(c) PART
(d) DEAR
4. In a coded language, MANAGER is written as REGANAM. How will
ASSISTANT be written in that code ?
(a) TNATSISSA
(b) TNATISSSA
(c) TNATSSIA
(d) TNATSISAS
5. A boy introduced a girl as the daughter of the son of the father of his
uncle. How is the girl related to the boy ?
(a) Aunt
(b) Grand-daughter
(c) Niece
(d) Sister
6. Which one of the given responses would be meaningful order of the
following in ascending order?
1. Phrase
2. Alphabet
3. Sentence
4. Word
(a) 2, 1, 4, 3
(b) 1, 2, 3, 4
(c) 2, 4, 1, 3
(d) 2, 4, 3, 1
DIRECTIONS : In questions no. 7 to 10, select the related
word/letter/number from the given alternatives.
7. 3 : 7 : : 15 : ?
(a) 30
(b) 35
(c) 45
(d) 49
8. Kalidas : Meghdoot : : Kautilya : ?
(a) Ramayana
(b) Arthashastra
(c) Kamayani
(d) Kadambari
9. Water : Ocean : : Sand : ?
(a) Island
(b) Waves
(c) River
(d) Desert
10. ABC : XYZ : ; CDE : ?
(a) UVW
(b) WXY
(c) AVW
(d) VWX
DIRECTIONS: In questions no. 11 to 13, select the one which is different
from the other three alternatives.
11. (a) 42 – 49
(b) 35 – 62
(c) 63 – 70
(d) 28 – 21
12. (a) Engineer
(b) School
(c) Lawyer
(d) Doctor
13. (a) SRPQ
(b) YWTV
(c) IHFG
(d) NMKL
14. Six persons are sitting in a circle. 'J' is between 'N' and 'O'; 'N' is
opposite 'M'; and 'L' is not in either of the neighbouring seats of 'N'.
Who is opposite to 'K'?
(a) M
(b) O
(c) J
(d) L
15. Select the correct combination of mathematical signs to replace * signs
and to balance the following equation :
35 * 7 * 25 * 15 * 2
(a) + = ×
(b) +=×
(c) × = +
(d) = ×
16. Some equations are solved on the basis of a certain system. Find the
correct answer for the unsolved equation on that basis.
If 7 × 9 × 6 × 5 = 5 × 7 × 4 × 3,
then 8 × 4 × 14 × 12 = ?
(a) 5 × 3 × 7 × 10
(b) 6 × 3 × 9 × 11
(c) 6 × 2 × 12 × 10
(d) 6 × 4 × 8 × 9
17. Select the missing number from the given responses.
121 156 105
145 187 126
115 190 ?
(a) 231
(b) 225
(c) 255
(d) 305
18. Deepak is standing facing South. She goes 20 metres ahead and turns
right and goes 30 metres. Now she turns left and goes for 40 metres and
turns right. In which direction is she headed now ?
(a) North
(b) South
(c) East
(d) West
19. Two statements are given followed by two conclusions I and II. You
have to consider the statements to be true, even if they seem to be at
variance from commonly known facts. You are to decide which of the
given conclusions definitely follows from the given statements. Indicate
your answer.
Statements : All pencils are rails.
All rails are stations.
Conclusions:
I. All stations are pencils.
II. Some stations are pencils.
(a) Both Conclusions I and II follow
(b) Neither Conclusion I nor II follows
(c) Only Conclusion I follows
(d) Only Conclusion II follows
20. Identify the diagram that best represents the relationship among the
classes given below :
Doctors, Engineers, Lawyers
21. If a mirror is placed on the line MN, then which of the answer figures is
the right image of the given figure ?
Question figure :

Answer figures :

22. Which answer figure will complete the pattern in the question figure ?
Question figure :

Answer figures :
23. From the given answer figures, select the one in which the question
figure is hidden/embedded.
Question figure :

Answer figures :

24. A piece of paper is folded and cut as shown below in the question
figures. From the given answer figures, indicate how it will appear
when opened.
Question figure :

Answer figures :

25. A word is represented by only one set of numbers as given in any one
of the alternatives. The sets of numbers given in the alternatives are
represented by two classes of alphabets as in the two matrices given
below. The columns and rows of Matrix I are numbered from 0 to 4
and that of Matrix II are numbered from 5 to 9. A letter from these
matrices can be represented first by its row and next by its column, e.g.
'P' can be represented by 04, 11, 30, etc. and 'T' can be represented by
58, 75, 99, etc. Similarly, you have to identify the set for the word given
below :
BASE
Matrix-I

Matrix-II

(a) 65, 01, 77, 13


(b) 78, 02, 98, 33
(c) 97, 02, 87, 14
(d) 89, 12, 76, 13
FOR VISUALLY HANDICAPPED CANDIDATES ONLY
20. Given below are capital letters in the first line and numbers in the
second line. Numbers and letters are codes for each other. Choose the
correct code for given letters.
CKWJMYQH LT
2 7 0 41 8 36 9 5
Given letters : J K C L W Q
(a) 461803
(b) 469035
(c) 472903
(d) 472035
21. Which one set of letters when sequentially placed at the gaps in the
given letter series shall complete it ?
_b c d a _ c d a b _ d a b c _
(a) d, c, a, b
(b) c, d, a, b
(c) a, b, c, d
(d) c, c, d, a
22. Arrange the following words as per the order in the dictionary:
1. Dairy
2. Dainty
3. Daimyo
4. Daisy
(a) 3, 2, 1, 4
(b) 4, 3, 2, 1
(c) 3, 2, 4, 1
(d) 3, 1, 4, 2
23. Aniruddh is heavier than Anand but not as heavy as Laxman. Anand is
heavier than Vijay. Raman is heavier than Aniruddh but lighter than
Shyam. Who of these is the lightest ?
(a) Anand
(b) Aniruddh
(c) Laxman
(d) Vijay
24. There are altogether 30 students in a class. If in a particular day 90%
students attend, how many students are absent ?
(a) 4
(b) 10
(c) 2
(d) 3
25. Find out the set among the four sets which is like the given set of
numbers, 500, 50, 5
(a) 608, 60, 6
(b) 619, 81, 9
(c) 280, 40, 5
(d) 392, 56, 8
PART-B : NUMERICAL APTITUDE
26. A sales representative will receive a 15% commission on a sale of `
2,800. If he has already received an advance of ` 150 on that
commission, the remaining amount of commission is
(a) ` 320
(b) ` 420
(c) ` 120
(d) ` 270
27. A railway train 100 metres long is running at the speed of 30 km/hr. In
what time does it pass a man standing near a line ?
(a) 10 seconds
(b) 13 seconds
(c) 12 seconds
(d) 15 seconds
28. Raju has to cover a distance of 240 km in 4 hours. If he covers one-third
of the journey in time, what is his speed at the beginning of the

journey ?
(a) 70 km/hr
(b) 75 km/hr
(c) 60 km/hr
(d) 65 km/hr
29. X borrowed some money from a source at 8% simple interest and lent it
to Y at 12% simple interest on the same day and gained ` 4,800 after 3
years. The amount X borrowed, in `, is
(a) 42,000
(b) 60,000
(c) 1,20,000
(d) 40,000
30. Ram borrows a certain sum of money at 8% per annum simple interest
and Rahim borrows ` 2,000 at 5% per annum simple interest. If the
interest at the end of 3 years is equal, then the amount borrowed by
Ram is
(a) ` 1,250
(b) ` 1,500
(c) ` 2,000
(d) ` 1,000
31. A bookseller sells a book at a profit of 10%. If he had bought it at 4%
less and sold it for ` 6 more, he would have gained . The cost

price of the book is


(a) ` 160
(b) ` 170
(c) ` 150
(d) ` 155
32. The length and breadth of a rectangle are 20 m and 15 m respectively. If
length is increased by 20% and the breadth by 30%, the percentage
increase in its area is
(a) 54%
(b) 56%
(c) 50%
(d) 52%
33. The average height of 8 students is 152 cm. Two more students of
heights 144 cm and 155 cm join the group. What is the new average
height ?
(a) 151.5 cm
(b) 152.5 cm
(c) 151 cm
(d) 150.5 cm
34. The batting average of a cricket player for 30 innings is 40 runs. His
highest score exceeds his lowest score by 100 runs. If these two innings
are not included, the average of the remaining 28 innings is 38 runs.
The lowest score of the player is
(a) 18
(b) 20
(c) 12
(d) 15
35. Aman sells two watches at ` 99 each. On one he gets 10% profit and on
the other he loses 10%. His net gain or loss percent is
(a) loss of 1%
(b) no profit no loss
(c) profit of 10%
(d) loss of 10%
36. If a person lost 8% by selling an article for ` 1,035, he bought the
article for
(a) ` 1,135
(b) ` 1,152
(c) ` 1,105
(d) ` 1,125
37. A cycle merchant allows 25% discount on the marked price of the
cycles and still makes a profit of 20%. If he gains ` 360 over the sale of
one cycle, find the marked price of the cycle.
(a) ` 2,920
(b) ` 2,800
(c) ` 2,880
(d) ` 2,900
38. If 2x = 3y = 4z, find x : y : z.
(a) 3 : 4 : 6
(b) 6 : 4 : 3
(c) 4 : 3 : 2
(d) 2 : 3 : 4
39. The ratio of the ages of A, B and C is 5 : 8 : 9. If the sum of the ages of
A and C is 56 years, the age of B will be
(a) 12 years
(b) 23 years
(c) 21 years
(d) 32 years
40. A box contain 280 coins of one rupee, 50 paise and 25 paise. The values
of each kind of coin are in the ratio of 8 : 4 : 3. The number of one
rupee coins will be
(a) 52
(b) 81
(c) 60
(d) 80
41. Length of each equal side of an isosceles triangle is 10 cm and the
included angle between those two sides is 45°. Find the area of the
triangle.
(a)
(b)
(c)
(d)
42. Rita purchased a car with a marked price of ` 2,10,000 at a discount of
5%. If the sales tax charged is 10%, find the amount she has to pay.
(a) ` 2,19,500
(b) ` 2,19,000
(c) ` 2,19,450
(d) ` 2,20,000
43. A shopkeeper sold an item for ` 1,800 at a discount of 10% and gained
` 200. Had he not given the discount, his gain would be
(a) ` 300
(b) ` 400
(c) ` 180
(d) ` 200
44. The sum of the squares of the digits of the largest prime number in two
digits is
(a) 148
(b) 130
(c) 97
(d) 118
45. Find the number lying between 900 and 1000 which when divided by
38 and 57 leaves in each case a remainder 23.
(a) 912
(b) 926
(c) 935
(d) 962
46. Raju can do a piece of work in 20 days, while Ram can do it in 30 days.
If both of them work at it together, then the number of days in which
they will be able to finish the work is
(a) 12 days
(b) 10 days
(c) 50 days
(d) 25 days
47. A is twice as good a workman as B and together they finish a piece of
work in 20 days. In how many days will A alone finish the work?
(a) 30 days
(b) 25 days
(c) 40 days
(d) 35 days
48. A spherical ball of lead of radius 14 cm is melted and recast into
spheres of radius 2 cm. The number of the small spheres is
(a) 300
(b) 525
(c) 343
(d) 450
49. Using the pie-chart answer the following ;

If the annual income of the family is ` 60,000, then the savings is


(a) ` 7,500
(b) ` 9,000
(c) ` 3,000
(d) ` 6,000
50. The equation of the graph shown here is

(a) when x ≤ 0, y = + 1
when x > 0, y = – 1
(b) when x < 0, y = + 1
when x ≥ 0, y = – 1
(c) when x < 0, y = + 1
when x > 0, y = – 1
(d) when x < 0, y = – 1
when x > 0, y = + 1
FOR VISUALLY HANDICAPPED CANDIDATES ONLY
49. A piece of work can be completed in 18 days if A and B work together,
in 24 days if B and C work together and in 36 days if A and C work
together. The number of days required if all the three work together is
(a) 14
(b) 12
(c) 10
(d) 16
50. Three different containers contain 496 litres, 403 litres, 713 litres of
mixtures of milk and water respectively. What biggest measurer can
measure all the different quantities exactly?
(a) 31 litres
(b) 41 litres
(c) 11 litres
(d) 17 litres
PART-C : ENGLISH LANGUAGE
DIRECTIONS: In question no 51 and 53, four alaternatives are given for
the Idiom/Phrase underlined in the sentence. Choose the alternative that best
expresses the meaning of the Idiom/Phrase and mark it in the Answer Sheet.
51. The son wants to purchase a new car, but his father is dragging his feet.
(a) acting in a slow and hesitant manner
(b) acting methodically
(c) acting quickly with firm conviction
(d) acting courageously
52. The student passed out in the lab during the practical exam.
(a) fell down
(b) became anxious
(c) rushed out
(d) fainted
53. My friend Rahim is fair and square in all his dealings.
(a) dishondest and complex
(b) cruel
(c) rough and complex
(d) honest and simple
DIRECTIONS: In question no. 54 to 58, a part of the sentence is
underlined. Below are given alternatives to the underlined part at (a), (b) and
(c) which may improve the sentence. Choose the correct alternative. In case
no improvement is needed your answer is (d). Mark ayour answer in the
Answer Sheet.
54. I knew that if the animal was at home in that moment it would probably
be sound asleep.
(a) within that moment
(b) about that moment
(c) at that moment
(d) No improvement
55. What would you have done if you are attacked by a bandit?
(a) if you have been attacked by a bandit
(b) if you would have been attacked by a bandit
(c) if you were attacked by a bandit
(d) No improvement
56. She works so that she may not fail.
(a) or that she may not fail
(b) therefore she may fail
(c) lest she should fail
(d) No improvement
57. Looking out of the window the little boy saw a kite entangled in the
branches of the gulmohar tree.
(a) Looking at the window
(b) Peeping in at the window
(c) Gazing out into the window
(d) No improvement
58. If you do not mend your behaviour, you will suffer
(a) bend
(b) repair
(c) tend
(d) No improvement
DIRECTIONS: In question no 59 and 63, out of the four alternatives,
choose the one which can be substituted for the given words/ sentence and
indicate it by blackening the appropriate oval ( ) in the Answer Sheet.
59. A place where money is coined.
(a) Press
(b) Mint
(c) Lair
(d) Archive
60. A series of lectures or lessrons.
(a) Catalogue
(b) Panel
(c) Course
(d) Syllabus
61. A false name adopted by an author for writing.
(a) Nomenclature
(b) Title
(c) Nickname
(d) Pseudonym
62. One who possesses many talents
(a) Gifted
(b) Talented
(c) Versatile
(d) Exceptional
63. A very accurate form of clock.
(a) Galvanometer
(b) Calorimeter
(c) Voltameter
(d) Chronometer
DIRECTIONS: In questions no. 64 to 69, four words are given in each
question, out of which only one word is correctly spell. Find the correct spell
word and mark your answer in the Answer Sheet.
64. (a) Millinar
(b) Millenar
(c) Miliner
(d) Milliner
65. (a) Privillage
(b) Privilage
(c) Priviledge
(d) Privilege
66. (a) Questionnaire
(b) Questionnare
(c) Questionnair
(d) Questionnaire
67. (a) Amoeba
(b) Ameoba
(c) Amieba
(d) Ameboa
68. (a) Annihilate
(b) Annhillate
(c) Anihilate
(d) Annihilet
69. (a) Committee
(b) Commitea
(c) Committee
(d) Comittee
DIRECTIONS : In question no. 70 to 74, you have a brief passage with 5
questions. Read the passage carefully and choose the best answer to each
questions out of the four alternatives and mark it by blackening the
appropriate oval ( ) in the Answer Sheet.
"Something is very wrong. "says the detective. "I know!" says Ms. Gervis.
"It is wrong that someone has stolen from me!" The detective looks around
Ms. Gervis' apartment. "That is not what I am talking about, ma'am. What is
wrong is that I do not understand how the robber got in and out." Ms. Gervis
and the detective stand in silence. Ms. Gervis' eyes are full of tears. Her
hands are shaking. "The robber did not come through the window," says the
detective. "These windows have not been opened or shut in months." The
detective looks at the fireplace. "The robber did not squeeze down here."
The detective walks to the front door. He examines the latch. "And
since there are no marks or scratches, the robber definitely did not try to or
scratches, the robber definitely did not try to break the lock." " I have no idea
how he did it." says a bothered Ms. Gervis. "It is a big mystery." "And you
say the robber stole nothing else?" asks the detective. "No money, no
jewellery, no crystal?" That's right, detective. He took only what was
important to me," Ms. Gervis says with a sigh. "There is only one thing I can
do now." And what is that?" the detective asks with surprise. "I will stop
baking cakes," Ms. Gervis says. "They are mine to give away. They are not
for someone to steal." "You can't do that!" says the detective with alarm.
"Who will bake those delicious cakes?" "I am sorry. I do not know," says Ms.
Gervis, "I must solve this case immediately!" says the detective.
70. What does Ms. Gervis say is a big mystery?
(a) How the robber got in
(b) How the robber got in and out
(c) How the robber got out
(d) How the robber stole
71. What was stolen?
(a) Crystal
(b) Money
(c) Cakes
(d) Jewellery
72. Why does the detective say, "I must solve this case immediately?"
(a) Because Ms. Gervis is scared
(b) Becuase Ms. Gervis is crying
(c) Because Ms. Gervis is worried about who stole from her house
(d) Because Ms. Gervis says she won't bake cakes again
73. What does the expression 'her hands are shaking' mean here?
(a) Ms. Gervis is shivering with fever
(b) Ms. Gervis is shivering with wonder
(c) Ms. Gervis is shivering with cold
(d) Ms. Gervis is shivering with fear
74. Why does the detective say that the robber did not come through the
front door?
(a) The latch was not opened
(b) There was no doorbell
(c) There was no lock
(d) There were no scratches
DIRECTIONS : In questions no. 75 to 84, some parts of the sentence have
errors and some are correct. Find out which part of a sentence has an error
and blacken the oval ( ) corresponding to the appropriate letter (a, b, c). If
a sentence is free from error, blacken the oval corresponding to (d) in the
Answer Sheet.

75. / / /

76. / / /

77. / / /
78. / /

79. / /

80. / / /

81. / / /

82. / / /

83. / /

84. / /

DIRECTIONS: In question no. 85 to 94, senetences are given with blanks


to be filled in with an appropriate word(s). Four alternatives are suggested
for each questions. Choose the correct alternative out of the four and indicate
it by blackening the appripriate oval ( ) in the Answer Sheet.
85. It is cool today, _____________?
(a) aren't it
(b) didn't it
(c) wasn't it
(d) isn't it
86. The book ultimately reached the person _____________ it belonged.
(a) who
(b) where
(c) to whom
(d) to who
87. Besides being reputed, he is famous for his _____________.
(a) humanities
(b) hostility
(c) humility
(d) humiliation
88. There was a queue of people in the rain, patiently waiting to get
_____________ the coach.
(a) at
(b) on
(c) onto
(d) in
89. I _____________ a terriable dream last night.
(a) had
(b) got
(c) saw
(d) have
90. Mr. Ahamed went _____________ a very tough time, when he
incurred a loss in his business.
(a) about
(b) off
(c) through
(d) over
91. The lawyer advised the convict not to _____________ anything.
(a) hold up
(b) hold over
(c) hold back
(d) hold in
92. Hospital services across the capital _____________ for three hours due
to the strike.
(a) were disrupted
(b) had been disrupted
(c) are disrupted
(d) have disrupted
93. The School Annaual Sports Day is postponed and it would now
_____________ on the 14th of next month.
(a) to be held
(b) been held
(c) being held
(d) be held
94. The coach leaves at 5.20, so get to the station ___________ that.
(a) before
(b) after
(c) until
(d) from
DIRECTIONS : In question no. 95 to 97, out of the four alternative, choose
the one which best expresses the meaning of the given word and mark it in
the Answer Sheet.
95. Disgrace
(a) Disrespect
(b) Jealousy
(c) Disregard
(d) Shame
96. Striking
(a) Attractive
(b) Violent
(c) Funny
(d) Hateful
97. Fiasco
(a) Festival
(b) Failure
(c) Fortune
(d) Feast
DIRECTIONS: In questions no. 98 to 100, choose the word opposite in
meaning to the given word and mark it in the Answer Sheet.
98. Gloomy
(a) Heavy
(b) Gay
(c) Sad
(d) Forlorn
99. Start
(a) Continue
(b) Break
(c) Begin
(d) Resume
100. Elation
(a) Animation
(b) Bliss
(c) Depression
(d) Pride
PART-D : GENERAL AWARENESS
101. Image files can be sent along with the e-mail documents using
(a) Attachments
(b) Subject
(c) Signature
(d) CC & BCC
102. The invention of _____________ led to the third generation of
computers.
(a) Vacuum tubes
(b) Very Large Scale Integration (VLSI)
(c) Transistors
(d) Integrated chips
103. Hydrogen bomb is based on the principle of
(a) Double decomposition
(b) Artificial radioactivity
(c) Nuclear fission
(d) Nuclear fusion
104. The commonly used safety fuse-wire is made of
(a) an alloy of Nickel and Lead
(b) an alloy of Tin and Lead
(c) an alloy of Tin and Nickel
(d) an alloy of Lead and Iron
105. At what temperature is the density of water the maximum?
(a) 2°C
(b) 4°C
(c) 0°C
(d) 1°C
106. The linear expansion of a solid rod is independent of its
(a) increase in temperature
(b) time of heat flow
(c) initial length
(d) material
107. Cathode rays when obstructed by metal cause emission of
(a) γ- rays
(b) X-rays
(c) α-rays
(d) β-rays
108. In a _____________ network, all devices are connected to a device
called a hub and they communicate through it.
(a) Ming
(b) Mesh
(c) Bus
(d) Star
109. Who is the father of biology?
(a) Lamarck
(b) Robert Hooke
(c) Aristotle
(d) Pasteur
110. The smallest unit of classification is
(a) Species
(b) Genus
(c) Family
(d) Order
111. Arenchyma is present in
(a) Banana stem
(b) Palm stem
(c) Aquatic plants
(d) Xerophytic plants
112. The deficiency of vitamin A causes
(a) Scurvy
(b) Night blindness
(c) Beri-Beri
(d) Dermatitis
113. Clove is a
(a) Dried flower bud
(b) Flower
(c) Fruit
(d) Seed
114. The recently discovered field with oil potential in Krishna-Godavari
Basin is called
(a) Ravva Offshore Block
(b) Golkunda Block
(c) Bombay High
(d) Telangana Block
115. Which of the following tribal groups are found in Manipur?
(a) Kuki
(b) Mundas
(c) Bhils
(d) Khonds
116. Consider the following sea-ports:
1. Chennai
2. Machilipatnam
3. Nagapattinam
4. Tuticorin
The correct sequence of these ports from north to south is
(a) 1, 3, 2, 4
(b) 2, 1, 4, 3
(c) 1, 2, 4, 3
(d) 2, 1, 3, 4
117. Santa Cruz is
(a) an International airport in Chennai
(b) a Domestic airport in Chennai
(c) an International airport in Mumbai
(d) a Domestic airport in Mumbai
118. What is the name given to Moon Mission in India?
(a) Vikram I
(b) Chandrayaan I
(c) Kalpana II
(d) Astrosat
119. Lira was the currency of which country?
(a) China
(b) Australia
(c) Japan
(d) Italy
120. On heating, Gypsum loses certain percentage of its water content and
becomes
(a) Chalk
(b) Calcium sulphate
(c) Plaster of Paris
(d) a pearl
121. Which one of the follwong is not an organ of the Government?
(a) Executive
(b) Legislature
(c) Sovereignty
(d) Judiciary
122. The Almatti Dam is constructed on the river
(a) Tungabhadra
(b) Krishna
(c) Kaveri
(d) Sileru
123. The name of the scientist who discovered neutron is
(a) Fermi
(b) Rutherford
(c) Chadwick
(d) Bohr
124. The bubbles in Champagne and Soda are
(a) Nitrogen
(b) Oxygen
(c) Carbon dioxide
(d) Hydrogen
125. Nobel Peace Prize - 2013 was awarded to
(a) Food and Agriculture Organization of the United Nations
(b) Organization for Economic Cooperation and Development
(c) Organization for the Prohibition of Chemical Weapons
(d) World Health Organization of the United Nations.
126. The State not ruled by the Congress political party during December
2013 is
(a) Mizoram
(b) Chhattisgarh
(c) Haryana
(d) Uttarakhand
127. Gobar gas contains mainly
(a) Butane
(b) Carbon monoxide
(c) Methane
(d) Carbon dioxide
128. "Carbon Credit" is a term associated with the
(a) Global deforestation
(b) Offshore banking
(c) Protection of environment
(d) Deforestation in India
129. The green colour of plant leaves is due to
(a) Protein
(b) Chlorophyll
(c) Cellulose
(d) Starch
130. Temperature is measured by the instrument called
(a) Voltmeter
(b) Calorimeter
(c) Thermometer
(d) Ammeter
131. As on December 2013, which of the following is the youngest political
party?
(a) BSP
(b) RJD
(c) JD(U)
(d) AAP
132. The pH of pure water is
(a) Seven
(b) Foruteen
(c) Zero
(d) One
133. The process of separation of pure water from impurities is called
(a) Fractional crystallisation
(b) Decantation
(c) Distillation
(d) Sublimation
134. H2SO4 cannot be used as
(a) Disinfectant
(b) Food preservative
(c) Drying agent
(d) Dehydrating agent
135. Who is the Supreme Commander-in-Chief of the armed forces?
(a) Prime Minister
(b) Defence Minister
(c) President
(d) Vice-President
136. To make tools and weapons, the earliest inhabitants of India used
(a) clay
(b) wood
(c) stones
(d) bronze
137. Name an Indian Saint (priest) who, attended the "World Congress of
Religions" held at Chicago (U.S.) in 1893.
(a) Basaveshwara
(b) Swami Vivekananda
(c) Ramanuja
(d) Madhavacharya
138. Chhatrapati Shivaji was a follower of
(a) Madhava
(b) Kabir
(c) Basava
(d) Ramadasa
139. Who was the greatest Kushan ruler?
(a) Vashiska
(b) Vasudeva
(c) Huvishka
(d) Kanishka
140. Who was the founder of Arya Samaj?
(a) Kabir Das
(b) Shankaracharya
(c) Ranade
(d) Dayanand Saraswati
141. Which one of the following is an abiotic resource?
(a) Mineral oil
(b) Water
(c) Forest
(d) Coal
142. The concept of Judicial Review has been borrowed from the
Consitution of
(a) France
(b) Great Britain
(c) U.S.A.
(d) U.S.S.R.
143. In the Constitution of India, the budget is known as
(a) Annual Financial Statement
(b) Annual Revenue Statement
(c) Annual Budget Statement
(d) Annual Expenditure Statement
144. A portion of an individual's total income is spent on consumption. The
remaining part is called
(a) Savings
(b) Deposits
(c) Surplus
(d) Excess
145. Phase of increasing returns is otherwise called
(a) increasing cost
(b) decreasing cost
(c) increasing revenue
(d) decreasing revenue
146. Monopoly refers to
(a) competition among monopolists
(b) absence of competition
(c) a firm charging different prices to different customers
(d) a market situation in which there is only one buyer of a
commodity and one seller
147. Which is The second most populous State of India as per Census 2011?
(a) West Bengal
(b) Rajasthan
(c) Maharashtra
(d) Bihar
148. Which of the following countries has a Parliamentary form of
Government?
(a) New Zealand
(b) Cuba
(c) United States of America
(d) France
149. Primary sector of an economy includes
(a) Service sector rendering services like banking, transport, etc.
which one of the primary importance
(b) Important enterprises of the manufacturinig sector
(c) Indsutrial sector which is of primary importance for the economy
(d) Agriculture and allied activities
150. Liquidity preference theory of interest is propounded by
(a) J.S. Mill
(b) A. Marshall
(c) I. Fisher
(d) J.M. Keynes
1. (b)

2. (b)

3. (b) There is no 'K' in the word DEPARTMENT.


4. (a) Reverse order
ASSISTANT
Reverse order → TNATSISSA

5. (c)

There is no option of cousin sister.


6. (c) Alphabet → Word → Phrase → Sentence
(2) (3) (4) (2)
7. (b)

3x = 15 × 7

8. (b) Meghdoot has been written by Kalidas.


Similarly,
Arthashastra has been written by kautitya.
9. (c) Ocean is the mass of water
Similarly,
Desert is the mass of sand.

10. (d)

11. (b) Except (b) all others are divisible by 7.


12. (c) Except (b) others are connected with a job that needs special skill,
while school is an organisation.

13. (b)

14. (b)

Hence, O is the opposite of K.


15. (b) 30 ÷ 7 + 25 = 15 × 2
5 + 25 = 30
30 = 30

16. (c)
17. (b) 156 – 121 = 35 × 3 = 105
187 – 145 = 42 ×3 = 126
190 – 115 = 75 × 3 =

18. (d)

19. (d)

Conclusions:
I. False II. True

20. (d)

21. (b)
22. (b)
23. (a)
24. (c)
25. (d)

For visually handicapped only

20. (c)

21. (c) a b c d/
a b c d/
a b c d/
abcd
22. (a) Order in the dictionary:
Daimya → Dainty → Dairy → Dasiy
(3) (2) (1) (4)
23. (d) Laxman > Aniruddh > Anand > Vijay
Shyam > Raman> Aniruddh
Combining (i), (ii) & (iii)
Laxam, Shyam > Raman > Aniruddh > Anand > Vijay
OR
Shyam > Raman, Laxman > Aniruddh > Anand> Vijay
Hence, Vijay is the lightest.
24. (c) Number of students present in class =

Number of students absent in a class = 30 – 27 = 3


25. (d) 500/10 = 50; 50/10 = 5
Similarly,
392/7 = 56; 56/7 = 8
26. (d) Sales representative will receive total amount

remaining amount = 420 – 150 = 270


27. (c) Time taken by train = S = 12 seconds

28. (a) Distance = km = 80 km

Speed = =70 km/h

29. (d) Let X borrow ` P


He has to return total amount,

Total amount x get from y, A′ =

According to question
A′ – A = 4800

30. (a) Let Ram borrowed ` P

= `1,250

31. (c) Let C be the cost price of book


Selling price, S = C +
If cost price is 6% less, C′ = C –

S′ = 1.1C + 6

14C + 600 = 18C


4C = 600
C = `150
32. (b) Area of rectangle, A = 20 m × 15 m = 300m2.

increased area,

= 24 × 19.5 = 468 m2.


% increase in area = = 56%

33. (a) Total height of 8 students = 8 × 152 cm = 1216 cm


Total height of 10 students = 1216 cm + 144 cm + 155 cm = 1515
cm

new average =

34. (c) Total runs in 30 innings = 30× 40 = 1200


total runs in 28 innings = 28 ×38 = 1064
Sum of height and lowest score
H + L = 1200 – 1064
H + L = 136 ...(1)
Also, H – L = 100 ...(2)
adding (1) and (2)
2H = 236
H = 118
Lowest run, L = 118 – 100 = 18
35. (a) Cost price of watch on which he get 10% Profit,

Cost Price of watch on which he losses 10%,

Net loss%

36. (d) Person bought the article for

= ` 1,125
37. (c) Let M be the market price and C be the cost price of the cycle.
Selling price,

Also,
M = 360 × 4 × 2 = ` 2,880
38. (b) 2x = 3y = 4z
x = 2z, y =

x : y : z = 2z : :z=6:4:3

39. (d) Let ages of A, B and C are 5x, 8x and 9x respectively.


5x + 9x = 56
x=4
Age of B = 8 × 4 = 32 years
40. (d) Ratio of number of coins
= 8 : 4 × 2 : 3 × 4 = 8 : 8 : 12
=2:2:3
Number of one rupee coin

41. (a) Area of triangle

42. (c) Selling price of car; S.P. = 2, 10,000 –

= 1,99,500
Sales tax charged is 10%
Total cost for Rita = 1,99,500 +

= ` 2, 19, 450
43. (b) Let M be the marked price.
Cost Price, C = 1800 – 200 = 1600
If no discount is given
Profit ` 2000 – ` 1600 = ` 400
44. (b) Largest two digit prime number is 97
92 + 72 = 81 + 49 = 130
45. (c) L.C.M of (38, 57) = 114
Multiple of 114 between 900 and 1000 = 912
number which leaves 23 = 912 + 23 = 935
46. (a) Raju and Ram together can finish the work in

47. (a) If A can finish the work in x days, B finish the same work in 2x
days.
(A + B) together finish work in 20 days

x = 30 days
48. (c) Numebr of small spheres,

49. (d) Angle for saving = 360° – (72° + 72° + 72° + 180°)
= 360° + 324° = 36°
Savings = = ` 6,000

50. (a)
For visually handicapped candidates only
49. (d) ...(1)

...(2)

...(3)

Adding (1), (2) and (3)

If all three work together work finish in 16 days


50 (a) Required measurer= H.C.F (496, 403, 713)
= 31 Litres.
50. (a) Required measurer = H.C.F (496, 403, 713) = 31 litres.
51. (a) The idiom 'dragging his feet' means to deal with something slowly
because you do not really want to do it. Here, the son wants to buy
a new car, but his dad is acting in a slow and hesitant manner.
52. (d) The idiom 'pass out' means to become unconscious and faint. Here
the student fainted in the lab during the practical exam.
53. (d) The idiom 'fair and square' means completely fair; justly; within
the rules. Here 'My friend Rahim is honest and simple in all his
dealings'.
54. (c) I knew that if the animal was at home at that moment, it would
probably be sound asleep. The preposition 'at' is used to signify the
point in time.
55. (c) What would you have done if you were attacked by a bandit? If,
will, would and were are used in case of condition and/or
imagination.
56. (d) The sentence is correct and doesn't need any improvement.
57. (d) The sentence is correct and doesn't need any improvement.
58. (d) The sentence is correct and doesn't need any improvement. The
idiom ' mend your behaviour' means to improve your behavior and
stop doing things that cause trouble.
59. (b) Mint is a place where money is made, or an unlimited supply of
wealth. In other words, it is a place where the coins of a country
are manufactured by authority of the government.
60. (a) Course means a list of contents at a university, systematically
arranged and often including descriptive material such as lessons
and lectures.
61. (d) A pseudonym is a name that a person or group assumes for a
particular purpose, which differs from his or her original or true
name. Pseudonyms include stage names, screen names, ring names,
pen names, nicknames, aliases, superhero identities and code
names, gamer identifications.
62. (c) A versatile person is one who is able to adapt or be adapted to
many different functions or activities and is multitalented and
flexible.
63. (d) A chronometer is an instrument for measuring time accurately in
spite of motion or variations in temperature, humidity and air
pressure. It is s timepiece with a special mechanism for ensuring
and adjusting its accuracy.
64. (d) Milliner is the correctly spelt word. Milliner is person who makes
or sells women's hats.
65. (d) Privilege is the correctly spelt word. It means a special right,
advantage, or immunity granted or available only to a particular
person or group.
66. (a) Questionnaire is the correctly spelt word. A questionnaire is a
research instrument consisting of a series of questions and other
prompts for the purpose of gathering information from
respondents.
67. (a) Amoeba is the correctly spelt word. It is a type of cell or organism
which has the ability to alter its shape, primarily by extending and
retracting pseudopods.
68. (a) Annihilate is the correctly spelt word. It means to destroy
completely.
69. (c) Committee is the correctly spelt word. It is a group of person
appointed for a specific function by a larger group and typically
consisting of members of that group.
70. (b) Ms. Gervis is unable to understand the mystery about how the
robber got in and out of the house.
71. (c) The robber stole the cakes that Ms. Grevis baked for giving.
72. (d) When Mr. Grevis got to know that the cakes she baked for giving
are stolen; she was devastated and decided to quit baking. The
detective was worried for who will bake the delicious cakes if she
quits baking, thus he decides to solve the case quickly.
73. (d) Ms. Grevis was shivering with fear that she had robbers at her
home.
74. (d) the detective checked on the front door and because the latch did
not had any scratch or marks.
75. (c) When the weather is pleasant it is nice to go on a picnic.
76. (a) Many passersby stopped at the place of accident. 'passer-bys' is an
incorrect word. Because the people are plural, not the action or
time. i.e., "by" can indicate action and/or time, and that cannot be
plural.
77. (c) The batsman completed his century in minimum number of balls.
78. (c) The accident was fatal as the truck had a head on collision with a
van. With is used to indicate being together or being involved.
79. (e) A real brave person is not one who does not feel afraid and one
who conquers that fear. Here pronoun that should be used instead
of he is one. As a pronoun, one can also function in an impersonal,
objective manner, standing for the writer or for all people who are
like the writer or for the average person or for all people who
belong to a class.
80. (a)
81. (b) I have known her for three years now. We use for when we
measure the duration - when we say how long something lasts.
82. (b) I am happy because I have paid off my bank loan. The sentence is
present participle and have is used instead of had because the
action is completed before 'now' the present.
83. (b) They blamed their teachers for their poor performance in the Board
examination. For is used to mean because of.
84. (b)
85. (d) It is cool today, isn't it?
86. (c) The book ultimately reached the person to whom it belonged.
87. (c) Besides being reputed, he is famous for his humility.
88. (c) There was a queue of people in the rain, patiently waiting to get
onto the coach. Onto has the word to in it, which reminds us that its
meaning includes the sense of movement towards something. The
preposition on does not have this sense of movement, and it tells
you only about location.
89. (a) I had a terrible dream last night. Had is used because the action
was completed before another action in the past.
90. (c) Mr. Ahamad went through a very tough time, when he incurred a
loss in his business.
91. (c) The lawyer advised the convict not to hold back anything.
92. (a) Hospital services across the capital were disrupted for three hours
due to the strike.
93. (d) The school Annual Sports Day is postponed and it would now be
held on the 14th of next month. The sentence is in passive form of
past participle.
94. (a) The coach leaves at 5:20, so get to the station before that.
95. (d) disgrace means a state of shame.
96. (a) Striking means extraordinary, attractive.
97. (b) fiasco means a complete failure.
98. (b) gloomy means sad and dark. Whereas it's opposite is gay which
means happy.
99. (b) start means to begin. Whereas it's opposite is break.
100. (c) elation means extreme happiness. Whereas it's opposite is
depression.
101. (a) Image files can be sent along with the e-mail using attachments.An
e-mail attachment is a file that is attached to an e-mail message.
For example, you may attach a graphic, a spreadsheet, or a word
processing document.
102. (d) The invention of integrated chips led to the third generation of
computers. During the period of 1964 to 1971 third generation
computers were developed. The third generation computers
emerged with the development of IC (Integrated Circuits). IC is a
single component containing a number of transistors. It made the
computers more fast and reliable.
103. (d) Hydrogen bomb is based on the principle of nuclear fusion.
Hydrogen bomb or H-bomb, weapon deriving a large portion of its
energy from the nuclear fusion of hydrogen isotopes. The hydrogen
bomb functions by the fusion, or joining together, of lighter
elements into heavier elements. The end product again weighs less
than its components, the difference once more appearing as energy.
Because extremely high temperatures are required in order to
initiate fusion reactions, the hydrogen bomb is also known as a
thermonuclear bomb.
104. (b) The commonly used safety fuse wire is made up of an alloy of tin
and lead. Fuse is a safety device used in any electrical installation,
which forms the weakest link between the supply and the load. It is
a short length of wire made of lead / tin /alloy of lead and tin/ zinc
having a low melting point and low ohmic losses. A fuse wire
should always have a high resistance and low melting point. High
resistance to heat up soon and low melting point to melt away due
to the heat produced by high resistance so that the circuit is not
damaged
105. (b) When cooled from room temperature liquid water becomes
increasingly dense, as with other substances, but at approximately
4 °C (39 °F), pure water reaches its maximum density. As it is
cooled further, it expands to become less dense.
106. (b) The linear expansion of a solid rod is independent of the time of
heat flow. It is directly dependent on the temperature, length of the
rod and material of the rod.
107 (b) X-rays are produced as a result of obstruction of cathode rays with
metal.
108. (d) Star networks are one of the most common computer network
topologies. In its simplest form, a star network consists of one
central switch, hub or computer, which acts as a conduit to transmit
messages. This consists of a central node, to which all other nodes
are connected; this central node provides a common connection
point for all nodes through a hub.
109. (c) Aristotle is the father of Biology and Zoology.
110. (a) the smallest unit of classification is species. The hierarchy of
biological classification's major taxonomic ranks is as follows:
species< genus< family < order< class< phylum < kingdom <
domain<life.
111. (c) Aerenchyma refers to spaces or air channels in the leaves, stems
and roots of some plants, which allows exchange of gases between
the shoot and the root. Aerenchyma is widespread in aquatic and
wetland plants which must grow in hypoxic soils.
112. (b) Night blindness is the first sign of vitamin A deficiency. Night
blindness and its worsened condition, xerophthalmia, are markers
of vitamin A deficiency, as it can also lead to impaired immune
function, cancer, and birth defects.
113. (a) Cloves are the aromatic flower buds of a tree in the family
Myrtaceae, Syzygium aromaticum. They are native to the Maluku
Islands in Indonesia, and are commonly used as a spice.
114. (a) The Ravva oil and gas field in the Krishna- Godavari Basin was
developed in partnership with Cairn India, ONGC, Videocon and
Ravva Oil, under a production sharing contract (PSC) that runs
until 2019.
115. (a) Kuki tribes are found in Manipur. Kuki Tribes of Manipur are one
of the most dominant tribes who inhabit the hilly regions. While
the valley of Manipur is largely inhabited by the Meiteis and
Meitei pangal, the hilly regions which surround the capital of
Imphal are inhabited by the Kuki and Naga tribes.
116. (d) Starting from north, Machallipatnam is located in Andhra Pradesh.
Chennai, Nagapatnam and Tuticorin are present in Tamil Nadu.
117. (d) Santa Cruz is a domestic airport in Mumbai.
118. (b) Chandrayaan-1 was India's first lunar probe. It was launched by
the Indian Space Research Organisation in October 2008, and
operated until August 2009.
119. (d) Lira was the currency of Italy between 1861 and 2002. Between
1999 and 2002, the Italian lira was officially a national subunit of
the euro.
120. (c) Heating gypsum partially dehydrates the mineral by driving off
approximately 75% of water contained in its chemical structure.
The partially dehydrated mineral is called calcium sulfate
hemihydrate or calcined gypsum commonly known to be Plaster of
Paris.
121. (c) Political theory envisages a Separation of Powers between the
three main organs of government i.e. the legislature, the executive
and the judiciary, in the interest of prevention of concentration of
power.
122. (b) The Almatti Dam is a dam project on the Krishna River in North
Karnataka, India which was completed in July 2005. The Almatti
Dam is the main reservoir of the Upper Krishna Irrigation Project.
123. (c) Neutron was discovered by a British Physicist named Sir James
Chadwick. In1932, Chadwick showed that the radiation from the
element beryllium, caused by the bombardment of alpha particles
is actually a stream of electrically neutral particles. He called these
particles neutrons. He also studied some other properties of these
particles. Neutrons directly emitted from atomic nuclei are termed
as fast neutron.
124. (c) Carbonated beverages like soda get their name from the carbon
dioxide dissolved in the beverage. Most gases will dissolve in
water (which is the main component of soda) but carbon dioxide is
particularly good at dissolving and it adds a nice slightly sour taste
to the beverage. It may also be added to make it reminiscent of the
carbonation found in fermented beverages like champagne.
125. (c) Organisation for the prohibition of chemical weapons won the
noble peace prize 2013 for its extensive efforts to eliminate
chemical weapons.
126. (b) Chhatisgarh state was ruled by Bhartiye Janta Party during
December 2013. Since Chhattisgarh was created in 2000, when it
was carved out of the tribal-dominated southern districts of
Madhya Pradesh, two people have served as the state's chief
minister. The first was the Indian National Congress party's Ajit
Jogi, who served for three years from 2000 to 2003. After Jogi's
departure, since 7 December 2003, Raman Singh of the Bharatiya
Janata Party has been the incumbent Chief Minister of
Chhattisgarh.
127. (c) Bio gas is a clean unpolluted and cheap source of energy in rural
areas. It consists of 55-70% methane which is inflammable. Bio
gas is produced from cattle dung in a bio gas plant commonly
known as gobar gas plant through a process called digestion.
128. (c) Carbon credit is a permit which allows a country or organization to
produce a certain amount of carbon emissions and which can be
traded if the full allowance is not used. It is associated with
protection of environment.
129. (b) A green leaf is green because of the presence of a pigment known
as chlorophyll, which is inside an organelle called a chloroplast.
When they are abundant in the leaf's cells, as they are during the
growing season, the chlorophylls' green color dominates and masks
out the colors of any other pigments that may be present in the leaf.
Thus the leaves of summer are characteristically green
130. (c) Temperature is measured by a thermometer. One of the most
common devices for measuring temperature is the glass
thermometer. This consists of a glass tube filled with mercury or
some other liquid, which acts as the working fluid. Temperature
increase causes the fluid to expand, so the temperature can be
determined by measuring the volume of the fluid. Such
thermometers are usually calibrated so that one can read the
temperature simply by observing the level of the fluid in the
thermometer.
131. (d) Aam Aadmi Party is the youngest party as on December 2013. It
was launched on 6 November 2012 that came into existence
following differences between the activists Arvind Kejriwal and
Anna Hazare regarding whether or not to politicise the popular
India Against Corruption movement that had been demanding a
Jan Lokpal Bill since 2011. The party's first electoral test was in
the 2013 Delhi legislative assembly election, from which it
emerged as the second-largest party, winning 28 of the 70 seats.
132. (a) At 25° C the pH of pure water is very close to 7. Acids have a pH
less than 7 while bases have a pH greater than 7. Because it has a
pH of 7, water is considered to be neutral. It is neither an acid nor a
base, but is the reference point for acids and bases.
133. (c) Distillation is a process of separating the component substances
(impurities) from a liquid mixture by selective evaporation and
condensation. The end result is pure water.
134. (b) H2SO4 is the sulphuric acid which is a highly corrosive strong
mineral acid . It cannot be used in food industry as a preservative.

135. (c) The President of India is the Supreme Commander of the Indian
Armed Forces. The Indian Armed Forces are under the
management of the Ministry of Defence (MoD), which is led by
the Union Cabinet Minister of Defence.
136. (c) the earliest inhabitants of India used stones to make tools and
weapons.
137. (b) Swami Vivekanand was the Indian saint to attend ' World
Congress of Religion' held at Chicago in 1893. He is perhaps best
known for his inspiring speech beginning with "Sisters and
Brothers of America," through which he introduced Hinduism at
the opening session of the Parliament on 11 September. Thereafter
he conducted hundreds of public and private lectures and classes,
disseminating tenets of Hindu philosophy in America, England and
Europe. In America Vivekananda became India's spiritual
ambassador.
138. (d) Chhatrapati Shivaji was a follower of Ramayana and Mahabharta
from his childhood. Shivaji Maharaj did everything possible to
promote Hinduism in every aspect of the fight for independence
and establishing his own kingdom. Shivaji Maharaj's assembly of
eight ministers was formed based on Hindu ideals. One comes
across this concept of eight ministers in the Ramayan and the
Mahabharat.
139. (d) Kanishka was the greatest ruler of the Kushan Empire, a realm that
covered much of present-day India, Pakistan, Iran and other parts
of central Asia and China during the first and second centuries.
140. (d) Arya Samaj is a Hindu reform movement founded by Swami
Dayananda on 7 April 1875. He was a sannyasi who promoted the
Vedas. Dayananda emphasised the ideals of brahmacharya. The
group found most of its support in Punjab.
141. (b) Abiotic resources are those that come from non-living, non-
organic material. Examples of abiotic resources include land, fresh
water, air and heavy metals including ores such as gold, iron,
copper, silver, etc. Biotic resources are obtained from the biosphere
(living and organic material), such as forests and animals, and the
materials that can be obtained from them. Fossil fuels such as coal
and petroleum are also included in this category because they are
formed from decayed organic matter.
142. (c) The concept of Judicial Review in the Constitution of India is
borrowed from USA. In the Indian constitution, Judicial review is
dealt with under Article 13. Judicial Review refers that the
Constitution is the supreme power of the nation and all laws are
under its supremacy.
143. (a) In the Constitution of India, the budget is known as Annual
Financial Statement. The Union Budget of India, referred to as the
Annual Financial Statement[1] in Article 112 of the Constitution of
India, is the annual budget of the Republic of India, presented each
year on the last working day of February by the Finance Minister
of India in Parliament. The budget, which is presented by means of
the Financial Bill and the Appropriation bill has to be passed by the
House before it can come into effect on April 1, the start of India's
financial year.
144. (a) If a portion of individuals income is spent on consumption, the
remaining portion will be saving. Since whatever is not consumed
must be saved, as soon as we specify a consumption function we
have necessarily specified a savings function. "Function" just
means that one thing depends on another thing or things.
145. (d) Phase of increasing returns is otherwise called decreasing revenue.
If output increases by more than that proportional change in inputs,
there are increasing returns to scale (IRS).
146. (b) A monopoly exists when a specific person or enterprise is the only
supplier of a particular commodity. Monopolies are thus
characterized by a lack of economic competition to produce the
good or service and a lack of viable substitute goods.
147. (c) Maharashtra is the second most populous state as per census 2011
after Uttar Pradesh.
148. (a) New Zealand has parliamentary form of government. A
parliamentary system of government means that the executive
branch of government must have the direct or indirect support of
the parliament. This support is usually shown by a vote of
confidence. The relationship between the executive and the
legislature in a parliamentary system is called responsible
government. Parliamentary systems usually have a head of
government and a head of state. The head of government is the
prime minister, who has the real power. The head of state often is
an elected (either popularly or through parliament) president or, in
the case of a constitutional monarchy, hereditary.
149. (d) The primary sector of the economy is the sector of an economy
making direct use of natural resources. This includes agriculture,
forestry, fishing and mining. This is contrasted with the secondary
sector, producing manufactured goods, and the tertiary sector,
producing services.
150. (d) J.M. Keynes propounded what has come to be known as the
liquidity preference theory of interest. According to this theory, the
rate of interest is determined by the demand for and supply of
money.
PART-A :
GENERAL INTELLIGENCE & REASONING
DIRECTIONS (Qs. 1 & 2) : Select the related letter/word/number from the
given alternatives.
1. Astronomy : Stars : : Geology :?
(a) Sky
(b) Geometry
(c) Science
(d) Earth
2. Up : Down : : Back : ?
(a) Left
(b) Deep
(c) Front
(d) Right
3. If 4 × 2 × 6 = 1626, 3 × 7 × 4 = 974, then 5 × 6 × 8 = ?
(a) 3658
(b) 2568
(c) 5664
(d) 6456
DIRECTIONS (Qs. 4 - 7): Find the odd number/word from the given
alternatives.
4. (a) Madam
(b) Animal
(c) Othello
(d) Noun
5. (a) 5 : 25
(b) 8 : 64
(c) 7 : 49
(d) 6 : 30
6. (a) 245
(b) 443
(c) 633
(d) 821
7. (a) Green gram
(b) Millet
(c) Barley
(d) Wheat
8. Which one set of letters when sequentially placed at the gaps in the
given letter series shall complete it?
a _ b _ c _ a _ bc _ b _ cb
(a) acbcab
(b) ccbcca
(c) ccaccb
(d) cacabc
9. Arrange the following words as per order in the dictionary.
1. Obscure
2. Objective
3. Objection
4. Obligation
5. Oblivion
(a) 3, 2, 4, 5, 1
(b) 3, 2, 5, 4, 1
(c) 3, 2, 5, 1, 4
(d) 5, 2, 1, 3, 4
DIRECTIONS (Qs. 10 - 12): In the following questions, a series is given
with one term missing. Choose the correct alternative from the given ones
that will complete the series.
10. 15, 23, 31, 39, ? , 54, 61
(a) 45
(b) 47
(c) 46
(d) 44
11. 2, 3.5, 5, 6.5, 8, ? .
(a) 9.0
(b) 9.5
(c) 10.5
(d) 11.0
12. 32, 58, 92, 134, ? .
(a) 169
(b) 184
(c) 194
(d) 156
13. From the given alternative words, select the word which cannot be
formed using the letters of the given word:
DISINTEGRATION
(a) INTERROGATE
(b) SIGNATURE
(c) INTERN
(d) SINGER
14. If PAINT is coded as 74128 and EXCEL is coded as 93596, how is
ACCEPT coded?
(a) 459578
(b) 457958
(c) 459758
(d) 455978
15. If ‘+’ means ‘÷’, ‘×’means ‘+’, ‘–’ means ‘×’ and ‘÷’ means ‘–’, then
which of the following equations is correct?
(a) 36 + 6 – 3 × 2 = 20
(b) 36 × 6 + 3 – 2 < 20
(c) 36 × 6 + 3 × 2 > 20
(d) 36 + 6 × 3 + 2 = 20
16. A father is 5 times as old as his son. His son is 6 years old. After how
many years, will the father be 4 times as old as his son?
(a) 2 years
(b) 5 years
(c) 6 years
(d) 4 years
DIRECTIONS: In questions no. 17 and 18, among the four answer figures,
which figure can be formed from the cut-pieces given below in the question
figure?
17. Question Figure :
Answer Figures :

18. Question Figure :

Answer Figures :

DIRECTIONS: In questions no. 19 and 20, a mirror is placed on line MN.


Then which of the answer figures is the correct image of the given figure?
19. Question Figure :
Answer Figures :

20. Question Figure :

Answer Figures :

21. Find the missing figure of the series from the given responses.
Question Figures :

Answer Figures :
22. Select the missing number from the given responses.

(a) 56
(b) 49
(c) 45
(d) 64
23. In the given diagram, Circle represents strong men, Square represents
short men and Triangle represents military officers. Which region
represents military officers who are short but not strong?

(a) 2
(b) 3
(c) 4
(d) 1
24. Select the figure which is different from the rest.
25. A triangular piece of paper is folded and cut as shown below. Find out
from the answer figures how it will appear when opened.
Question Figures :

Answer Figures :

FOR VISUALLY HANDICAPPED


CANDIDATES ONLY
DIRECTIONS (Qs. 17 - 20) : Find the odd number/word/number pair from
the given alternatives.
17. (a) 11
(b) 18
(c) 33
(d) 5
18. (a) Weight
(b) Eyesight
(c) Intelligence
(d) Height
19. (a) C. V. Raman
(b) Amartya Sen
(c) Abdul Kalam
(b) Rabindra Nath Tagore
20. (a) 125 – 5
(b) 100 – 10
(c) 325 – 9
(d) 625 – 25
21. From the given alternative words, select the word which cannot be
formed using the letters of the given word.
ESTABLISHMENT
(a) TIMETABLE
(b) HESITATE
(c) ABSENCE
(d) BLEMISH
DIRECTIONS (Qs. 22 & 23) : Select the related word/letters from the given
alternatives.
22. Hands : Mittens : : Legs : ?
(a) Slippers
(b) Shoes
(c) Socks
(d) Boots
23. YWUS : QOMK : : ZXVT : ?
(a) YWUS
(b) RPNL
(c) RPNM
(d) XWUT
24. Select the missing number from the given responses.
1, 3, 7, 15, 31, 63, 127, ? .
(a) 260
(b) 275
(c) 350
(d) 255
25. Arrange the following words as per order in the dictionary.
1. Advert
2. Advertise
3. Adverse
4. Adversity
5. Adversary
(a) 5, 1, 4, 3, 2
(b) 5, 4, 3, 2, 1
(c) 5, 3, 4, 1, 2
(d) 5, 4, 1, 3, 2
PART-B : NUMERICAL APTITUDE
26. A circle is inscribed in an equilateral triangle and a square is inscribed in
that circle. The ratio of the areas of the triangle and the square is
(a)
(b)
(c)
(d)

27. The simplified value of is

(a) 4
(b) 3
(c) 2
(d) 6
28. equals

(a) 5
(b) 50
(c) 500
(d) 0.05
29. Two pipes A and B can fill a tank in 6 hours and 4 hours respectively. If
they are opened on alternate hours and if pipe A is opened first, then the
tank shall be full in
(a)

(b) 5 hrs
(c)

(d) 6 hrs
30. A, B and C can do a piece of work in 10, 12 and 15 days respectively. A
leaves 5 days before the completion of the work and B leaves 2 days
after A. The whole work lasts for
(a) 7 days
(b) 6 days
(c) 12 days
(d) 13 days
31. A merchant purchases a wrist watch for ` 450 and fixes its list price in
such a way that after allowing a discount of 10%, he earns a profit of
20%. Then the list price of the watch is
(a) ` 600
(b) ` 650
(c) ` 700
(d) ` 550
32. The ratio in which a man must mix rice at ` 10.20 per kg and ` 14.40
per kg so as to make a mixture worth ` 12.60 per kg, is
(a) 3 : 4
(b) 4 : 3
(c) 2 : 5
(d) 18 : 24
33. If the sum of the length, breadth and height of a rectangular
parallelopiped is 24 cm and the length of its diagonal is 15 cm, then its
total surface area is
(a) 351 cm2
(b) 256 cm2
(c) 265 cm2
(d) 315 cm2
34. Two successive discounts of 70% and 30% are equivalent to a single
discount of
(a) 89%
(b) 75%
(c) 79%
(d) 100%
35. A merchant allows a discount of 10% on marked price for the cash
payment. To make a profit of 17%, he must mark his goods higher than
their cost price by
(a) 30%
(b) 33%
(c) 40%
(d) 27%
36. The present ages of two persons are 36 and 50 years respectively, if after
n years the ratio of their ages will be 3 : 4, then the value of n is
(a) 3
(b) 4
(c) 7
(b) 6
37. Out of 20 boys, 6 are each of 1 m 15 cm height, 8 are of 1 m 10 cm and
rest of 1 m 12 cm. The average height of all of them is
(a) 1 m 12 cm
(b) 1 m 12.1 cm
(c) 1 m 21.1 cm
(d) 1 m 21 cm
38. Average of first five prime numbers is
(a) 3.6
(b) 5.3
(c) 5.6
(d) 5
39. A dishonest grocer sells rice at a profit of 10% and also uses weights
which are 20% less than the marked weight. The total gain earned by
him will be
(a) 35%
(b) 37.5%
(c) 40%
(d) 30.5%
40. The cost price of a radio is ` 600. 5% of the cost price is charged
towards transportation. After adding that, if the net profit to be made is
15%, then the selling price of the radio must be
(a) ` 684.50
(b) ` 704.50
(c) ` 724.50
(d) ` 664.50
41. A certain amount of money is divided among x, y and z. If x receives
25% more than y and y receives 25% less than z, then x : y : z is equal to
(a) 12 : 10 : 11
(b) 14 : 12 : 13
(c) 15 : 12 : 16
(d) 10 : 9 : 12
42. Walking at a speed of 5 km/hr, a man reaches his office 6 minutes late.
Walking at 6 km/hr, he reaches there 2 minutes early. The distance of his
office is
(a) 2 km
(b) 3 km
(c) 4 km
(d) 3.5 km
43. By selling a fan for ` 600, a man loses 10%. To make a gain of 20%, the
selling price of the fan should be
(a) ` 800
(b) ` 900
(c) ` 1000
(d) ` 700
44. Two persons contested an election of Parliament. The winning candidate
secured 57% of the total votes polled and won by a majority of 42,000
votes. The number of total votes polled is
(a) 4,00,000
(b) 5,00,000
(c) 6,00,000
(d) 3,00,000
45. A number when reduced by 10% gives 30. The number is
(a) 35
(b)

(c)
(d) 40
46. Two trains 108 m and 112 m in length are running towards each other on
the parallel lines at a speed of 45 km/hr and 54 km/hr respectively. To
cross each other after they meet, it will take
(a) 10 sec
(b) 12 sec
(c) 9 sec
(d) 8 sec
47. A sum of money becomes 1.331 times in 3 years as compound interest.
The rate of interest is
(a) 50%
(b) 8%
(c) 7.5%
(d) 10%
48. A person deposited ` 500 for 4 years and ` 600 for 3 years at the same
rate of simple interest in a bank. Altogether he received ` 190 as interest.
The rate of simple interest per annum was
(a) 3%
(b) 4%
(c) 5%
(d) 2%
49. The following graph represents the maximum and minimum temperature
recorded every day in a certain week. The day on which the difference
between the maximum and minimum temperature was maximum is
(a) Monday
(b) Wednesday
(c) Saturday
(d) Sunday
50. Different choices made by a group of 200 students are given below in
percentage. The number of students who have taken neither Science nor
Commerce is

(a) 40
(b) 80
(c) 120
(d) 60
FOR VISUALLY HANDICAPPED
CANDIDATES ONLY
49. In an examination 70% of the total examinee passed. If the number of
failures is 420, the total number of examinees is
(a) 1400
(b) 1200
(c) 1000
(d) 693
50. If 10% of x is the same as 20% of y, then x : y is
(a) 5 : 1
(b) 1 : 2
(c) 3 : 1
(d) 2 : 1
PART-C : GENERAL ENGLISH
DIRECTIONS (Qs. 51 - 60) : In the following questions, some parts of the
sentences have errors and some are correct. Find out which part of a
sentence has an error and blacken the oval ( ) corresponding to the
appropriate letter (a, b, c). If a sentence is free from error, blacken the oval
corresponding to (d) in the Answer Sheet.
51. I and him / are / very good friends. / No error.
(a) (b) (c) (d)
52. One should / look after / their parents. / No error.
(a) (b) (c) (d)
53. She placed / the offering / to God in the altar. / No error.
(a) (b) (c) (d)
54. Teachers were instructed / to follow an uniform method / of
(a) (b)
evaluation. / No error.
(c) (d)
55. The newspapers they admit that / advertising sometimes /
(a) (b)
influences their editorial policy. / No error.
(c) (d)
56. No sooner did I finish / my speech, I was subjected / to a
(a) (b)
barrage of questions. / No error.
(c) (d)
57. I saw him / coming out of the hotel / on 10 o’clock. / No error.
(a) (b) (c) (d)
58. One of my friend / is returning / to India from the U.S.A. /
(a) (b) (c)
No error.
(d)
59. He knows / that your muscles / are not same as his. /
(a) (b) (c)
No error.
(d)
60. We shall wait / till you / will finish your lunch. / No error.
(a) (b) (c) (d)
DIRECTIONS (Qs. 61 - 70) : In the following questions, sentences are
given with blanks to be filled with an appropriate word(s). Four alternatives
are suggested for each question. Choose the correct alternative out of the
four.
61. The new government took __________ last year.
(a) out
(b) after
(c) over
(d) upon
62. Mohan’s career has taken some __________ twists and turns.
(a) incentive
(b) interesting
(c) interactive
(d) intuitive
63. The bus __________ fifty passengers fell __________ the river.
(a) with; into
(b) for; upon
(c) over; on
(d) of; at
64. It is raining __________ . Do not go out.
(a) heavily
(b) fast
(c) soundly
(d) strongly
65. She tries to adjust __________ her relations.
(a) for
(b) at
(c) so
(d) with
66. She was remarkably __________ in singing and dancing.
(a) accomplished
(b) conducive
(c) fluctuating
(d) cooperative
67. Sheila gained an advantage __________ me.
(a) upon
(b) from
(c) on
(d) over
68. Take this medicine regularly and you will get rid __________ this
disease.
(a) at
(b) from
(c) of
(d) over
69. Statistics __________ always my worst subject.
(a) are
(b) were
(c) is
(d) have
70. When she retired, she handed __________ the charge to the Vice-
President.
(a) over
(b) out
(c) across
(d) off
DIRECTIONS: In questions no. 71 to 73, out of the four alternatives,
choose the one which best expresses the meaning of the given word and mark
it in the Answer Sheet.
71. Barbaric
(a) Thorny
(b) Uncivilized
(c) Premeditated
(d) Barber’s
72. Hurdle
(a) Suspicion
(b) Throw
(c) Opposition
(d) Obstacle
73. Deter
(a) To hinder
(b) To neglect
(c) To disapprove
(d) To differ
DIRECTIONS (Qs. 74 - 76) : Choose the word opposite in meaning to the
given word and mark it in the Answer Sheet.
74. Indolent
(a) Solvent
(b) Diligent
(c) Malovolent
(d) Brilliant
75. Coherent
(a) Distorted
(b) Disorganized
(c) Inept
(d) Carefree
76. Brutal
(a) Adamant
(b) Humane
(c) Fearless
(d) Criminal
DIRECTIONS (Qs. 77 - 79) : In the following questions, four alternatives
are given for the idiom / phrase underlined in the sentence. Choose the
alternative which best expresses the meaning of the idiom / phrase and mark
it in the Answer Sheet.
77. The man changed colours when I questioned him on the allocation of
funds.
(a) turned pale
(b) got numbed
(c) turned happy
(d) get motivated
78. We cannot depend on him for this assignment as it needs careful
handling and he is like a bull in a china shop.
(a) a felicitous person
(b) a clumsy person
(c) a tactful person
(d) a no-nonsense person
79. The mother always insists on keeping the house spick and span.
(a) open
(b) locked
(c) safe
(d) tidy
DIRECTIONS (Qs. 80 - 84) : In the following questions, a part of the
sentence is underlined. Below are given alternatives to the underlined part at
(a), (b) and (c) which may improve the sentence. Choose the correct
alternative. In case no improvement is needed, your answer is (d)
80. On receiving his appointment letter, Ravi treated us with a sumptuous
meal.
(a) treated us to
(b) treated us for
(c) treated us by
(d) No improvement
81. She is scrutinising hard for the final examination.
(a) recollecting
(b) recapitulating
(c) revising
(d) No improvement
82. Since she directing the play for quite some time, she knows the actors
really well.
(a) Since she has directed
(b) Since she has been directing
(c) Since she was directing
(d) No improvement
83. You can borrow my laptop as long as you promise not to misuse it.
(a) only long as
(b) too long as
(c) so long as
(d) No improvement
84. This is the late edition of the Shakespearean play which was originally
published in 1603.
(a) later
(b) latest
(c) latter
(d) No improvement
DIRECTIONS (Qs. 85 - 89) : In the following questions, out of the four
alternatives choose the one which can be substituted for the given words /
sentence.
85. One who is unable to pay one’s debt
(a) Borrower
(b) Bankrupt
(c) Bank-roll
(d) Extravagant
86. Instrument that magnifies objects
(a) Microscope
(b) Periscope
(c) Stethoscope
(d) Telescope
87. Animals which live in water
(a) Barren
(b) Wild
(c) Domestic
(d) Aquatic
88. The study of plant life
(a) Geology
(b) Zoology
(c) Botany
(d) Geography
89. Exclusive possession or control of anything
(a) Mono-mania
(b) Monotheism
(c) Monopoly
(d) Monoism
DIRECTIONS (Qs. 90 - 95) : In the following questions, there are four
different words out of which one is correctly spelt. Find the correctly spelt
word.
90. (a) exellence
(b) excellence
(c) excellencce
(d) exillance
91. (a) grammar
(b) grammer
(c) gramer
(d) gramar
92. (a) ommited
(b) ommitted
(c) omitted
(d) omited
93. (a) calender
(b) calandar
(c) colendar
(d) calendar
94. (a) objectionable
(b) objectioneble
(c) objecktionable
(d) objectionablle
95. (a) apollogy
(b) appology
(c) apalogy
(d) apology
DIRECTIONS (Qs. 96 - 100) : Read the passage carefully and choose the
best answer to each question, out of the four alternatives.
Long ago in Mongolia there lived an emperor who feared growing old. One
day, he saw an old man in the street. Upset at being reminded that someday,
he too, would age, he ordered all the old people to leave his land.
One day, a violent storm swept the kingdom. Nothing was safe from its fury.
It roared into the palace and blew away the emperor’s belongings, including
his priceless golden pitcher. When the storm ended, the emperor ordered that
the pitcher be found and brought back to him.
People went in search of the pitcher. They saw it in a lake nearby. But no
matter who tried, no one could get a grip on the pitcher. All they got was a
handful of water. Yet it could be plainly seen, glittering and just below the
water’s surface.
96. The people saw the golden pitcher
(a) in a river nearby
(b) in a lake nearby
(c) in a pit nearby
(d) inside the palace
97. The emperor’s orders were that all the
(a) children should leave his land
(b) old men should leave his land
(c) old men should live in his land
(d) young men should stay in his land
98. What did the people who went to bring the pitcher get?
(a) Nothing at all
(c) A handful of water
(c) A handful of air
(d) The pitcher’s handle
99. The emperor feared
(a) getting old
(b) getting young
(c) getting weak
(d) getting ill
100. The emperor was upset to see the old man because
(a) it reminded him of his grandfather
(b) it reminded him that he might fall ill
(c) it reminded him that he would grow old too.
(d) it reminded him that he had to colour his hair.
PART-D : GENERAL AWARENESS
101. Name the Commander of the Arab army who conquered the Sindh.
(a) Muhammad bin Qasim
(b) Al Hazzaz
(c) Qutbuddin Aibak
(d) Allauddin Khilji
102. Who was the chairperson of the Chinese Communist Party at the time of
liberation of China?
(a) Liu Shaoqi
(b) Zhou Enlai
(c) Deng Xiaoping
(d) Mao Zedong
103. Who was regarded by Gandhiji as his political Guru?
(a) Gopal Krishna Gokhale
(b) Lala Lajpat Rai
(c) Bipin Chandra Pal
(d) Bal Gangadhar Tilak
104. Select the High Yielding Varieties of seed-crops developed under Green
Revolution in India.
(a) Wheat, Rice, Sugarcane, Pulses and Maize.
(b) Rice, Wheat, Pulses, Oil seeds and Sugarcane
(c) Maize, Black-gram, Jowar, Coffee and Tea.
(d) Rice, Wheat, Jowar, Bajra and Maize.
105. In which year did Amartya Kumar Sen receive the Nobel Prize in
Economics?
(a) 1990
(b) 1998
(c) 1995
(d) 2000
106. The worldwide Great Depression took place in
(a) 1930
(b) 1936
(c) 1929
(d) 1928
107. India is the largest producer and consumer of
(a) Sugar
(b) Paddy
(c) Tea
(d) Coffee
108. A camera in the hands of a professional photographer is a ___________
good.
(a) Capital
(b) Free
(c) Intermediary
(d) Consumer
109. Which among the following movements was not led by Mahatma
Gandhi?
(a) Civil Disobedience Movement
(b) Quit India Movement
(c) Swadeshi-Movement
(d) Non-Cooperation Movement
110. Constitution of India came into force in
(a) 1949
(b) 1951
(c) 1956
(d) 1950
111. Cold War refers to
(a) tension between East and West
(b) ideological rivalry between Capitalist and Communist world
(c) tension between Superpowers
(d) All of the above
112. Which one of the following is not an All India Service?
(a) Indian Police Service
(b) Indian Foreign Service
(c) Indian Forest Service
(d) Indian Administrative Service
113. In which session of the Indian National Congress was the “Poorna
Swaraj” resolution adopted?
(a) Karachi Session in 1931
(b) Lucknow Session in 1916
(c) Belgaum Session in 1924
(d) Lahore Session in 1929
114. The school of arts developed during the Kushan Period with the mixture
of Indian and Greek style is known as
(a) Mughal art
(b) Kushan art
(c) Persian art
(d) Gandhara art
115. Where was Christopher Columbus from?
(a) Portugal
(b) Venice
(c) Genoa
(d) Spain
116. Speed of processor chip is measured in
(a) Bytes/second
(b) Mbps
(c) MHz
(d) Bits/second
117. Which command is not used to switch off the computer?
(a) Hibernate
(b) Turn off
(c) Log off
(d) Shut down
118. Which one out of the following helps in burning
(a) Carbon dioxide
(b) Oxygen
(c) Carbon monoxide
(b) Nitrogen
119. In organic compounds, nitrogen is estimated by
(a) Dumas’ method
(b) Carius method
(c) Victor-Meyer’s method
(d) Liebig’s method
120. The busiest rail section in respect to goods transportation is
(a) Mumbai – Chennai section
(b) Delhi – Kolkata section
(c) Kolkata – Chennai section
(d) Delhi – Mumbai section
121. The Tropic of Cancer does not pass through
(a) China
(b) Myanmar
(c) Nepal
(d) Bangladesh
122. The country in East Asia which is most conspicuous for its decreasing
population growth since 1981 is
(a) Japan
(b) South Korea
(c) Thailand
(d) China
123. The Sethusamudram Ship Canal Project (SSCP) is supposed to reduce
the distance between Chennai and Tuticorin by _________ nautical
miles.
(a) 305
(b) 361
(c) 434
(d) 243
124. Cities with population from one to five million are called
(a) Cosmopolitan
(b) Conurbation
(c) Million City
(d) Metropolitan
125. Master copy of genetic information is
(a) DNA
(b) Nucleus
(c) r-RNA
(d) m-RNA
126. Contraceptive pills in the market contain
(a) Steroid-hormones
(b) Inorganic compounds
(c) Herbicides
(d) Antibiotics
127. Nematocysts are present in
(a) Sea anemone
(b) Starfish
(c) Ascaris
(d) Centipede
128. Which of the following micro-organisms is used in milk curdling?
(a) Lactobacillus
(b) Acctobacter
(c) Leuconostoc
(d) Bacillus
129. Which of the following is present in Chlorophyll molecule?
(a) K
(b) Mn
(c) Mn
(d) Fe
130. When a body falls from an aeroplane, there is increase in its
(a) Potential energy
(b) Kinetic energy
(c) Mass
(d) Acceleration
131. What does ‘Ozone Layer’ absorb?
(a) γ-rays
(b) Infrared rays
(c) Ultraviolet rays
(d) X-rays
132. In a water lifting electric pump, we convert
(a) Electrical energy into Kinetic energy
(b) Electrical energy into Potential energy
(c) Kinetic energy into Electrical energy
(d) Kinetic energy into Potential energy
133. The type of mirror used in automobiles to see the traffic on the rear side
is
(a) Convex
(b) Concave
(c) Plano-Convex
(d) Plane
134. In MS-Excel, _________ provide a visual representation of the values in
a worksheet.
(a) Views
(b) Charts
(c) Formulae
(d) Templates
135. Why was Justice Dalveer Bhandari in the news recently?
(a) He was elected to the International Court of Justice
(b) He became Director-General of World Trade Organization
(c) He became Secretary-General of SAARC
(d) He became a member of U.N. Human Rights Council
136. The hottest part of the gas flame is known as
(a) dark zone
(b) blue zone
(c) non-luminous zone
(d) luminous zone
137. Which of the following radiations has the least wavelength?
(a) β-rays
(b) X-rays
(c) α-rays
(b) γ-rays
138. The earth is a
(a) bad absorber and bad radiator of heat
(b) good reflector of heat
(c) non-absorber of heat
(d) good absorber and good radiator of heat
139. BCG vaccination is given at the age of
(a) Within 15 days
(b) 2 – 3 years
(c) 10 years
(d) Newborn
140. The International Year of Biodiversity was
(a) 2010
(b) 1996
(c) 1999
(d) 2006
141. Which of the following atmospheric gases constitute greenhouse gases?
1. Carbon dioxide
2. Nitrogen
3. Nitrous oxide
4. Water vapour
Select the correct answer using the codes given below.
(a) 1, 2 and 4
(b) 1, 3 and 4
(c) 1 and 4
(d) 1 and 3
142. ‘Kyoto Protocol’, an agreement signed by various countries, is
associated with
(a) Deep Sea Oil and Mineral Exploration
(b) Clean Environment and Climate Change
(c) Building common food stock to save human beings from any
natural disaster
(d) International Trade
143. Sanjukta Panigrahi was famous for the dance
(a) Odissi
(b) Bharatnatyam
(c) Kathak
(d) Manipuri
144. Which Indian batsman announced his retirement from International
Cricket on August 18, 2012?
(a) V. V. S. Laxman
(b) Virat Kohli
(c) Saurav Ganguly
(d) Sachin Tendulkar
145. Which sportsperson announced recently that he will not contest charges
of doping against UN Anti-Doping Agency?
(a) Lance Armstrong
(b) George Hincapie
(c) Bradley Wiggins
(d) Carl Lewis
146. Who is the winner of the Rajiv Gandhi Khel Ratna Award (2011)?
(a) M. S. Dhoni
(b) Gagan Narang
(c) Saina Nehwal
(d) Sachin Tendulkar
147. ‘White Revolution’ is related to
(a) Fish production
(b) Wheat production
(c) Milk production
(d) Flood control
148. Who proposed the adage “Survival of the fittest”?
(a) Lamarck
(b) Darwin
(c) William
(d) Huxley
149. Who is the author of the book ‘Beyond the Lines: An Autobiography’?
(a) Ray Bradbury
(b) Kushwant Singh
(c) General J. J. Singh
(d) Kuldip Nayar
150. Which of the following Indian States has the largest concentration of
Scheduled Tribes population?
(a) Orissa
(b) Madhya Pradesh
(c) Assam
(d) Bihar
PART A : GENERAL INTELLIGENCE
1. (d) Astronomy is the branch of science which deals with celestial
objects such as moons, planets, stars, galaxies etc. While geology is
the science which deals with the physical and substance of the
earth, their history, and the processes which act on them.
2. (c) Second is the opposite of first.
3. (b) 4 × 2 × 6 = 1626 = (42)26 = 1626
3 × 7 × 4 = 974 ⇒ (32)74 = 974
∴ 5 × 6 × 8 = (52)68 = 2568
4. (b)
5. (d) Second number is a square of first number. i.e., 5:(5)2, 8:(8)2, 7:(7)2.
Therefore, 6:30 is odd one out.
6. (c) Sum of all digits of given numbers are 11. But option (c) shows the
sum ‘12’. Therefore, it is odd one out.
7. (a)
8. (b)
9. (a) The correct order is:
Objection > Objective > Obligation > Oblivion > Obscure

10. (b)

11. (b)

12. (b)

13. (a) SIGNATURE cannot be formed as reference word doesnot have


‘U’ alphabet.
14. (d)
15. (a) By checking options
36 ÷ 6 × 3 + 2 = 6 × 3 + 2 ⇒ 20 = 20
16. (a) Son’s age = 6 yrs.
Father’s age = 30 yrs.
Let ‘x’ be the yr. after which father will be 4 times as old as his son.
According to question
30 + x = 4(6 + x) = 30 + x = 24 + 4x ⇒ 6 = 3x.
x = 2.
Hence, require year is 2 yrs.
17. (c)
18. (a)
19. (d)
20. (d)

21. (c)

22. (d) 23 = 8; 33 = 27
∴ 43 = 64

23. (a)

Hence, darken portion in above diagram represents that there are 2


military officers who are short but not strong.
24. (d)
25. (a)
For visually handicapped candidates only
17. (b) All other numbers except 18 are odd numbers.
18. (c) Weight, eyesight and height are medical standards.
19. (c) All others Abdul kalam are/were nobel prize winners.
20. (c) 125 – (5)3 – 5; 100 – (10)2 – 10; 625 – (25)2 – 25
Hence, (c) is odd one out.
21. (c) ABSENCE as ‘c’ letter is not in the reference word.
22. (c) Mitten is a type of protective clothing used to cover the hand.
Hence, socks is used to cover the legs.

23. (b)

24. (d)

25. (c) The correct order is -


Adversary > Adverse > Adversity > Advert > Advertise
PART B : NUMERICAL ABILITY

26. (d)

In the given figure ABC is an equilateral ∆ of a side with a circle


inscribed in it and a square inscribed in the circle.
AD, BO and CO are the angle bisectors of ∠A, ∠B and ∠C and O is
the centre of the circle.
We know that the angle bisector from the vertex of an equilateral
triangle is the perpendicular bisector of the opposite side.
AD is the perpendicular bisector of BC.
⇒ BD = and ∠DOB =

Now in ∆BOD

tan 30° =

⇒ Radius of circle =

Now in right ∆EDG


EG2 + GD2 = ED2 (Pythagoras theorem)

2(EG)2 = 2 (OD)2 =

Side of the square =

Now ar (∆ABC) : ar (∆EFG)

30. (a) Suppose, the work was finished in x days. Then,


A’s (x – 5) day’s work + B’s (x – 3) day’s work
+ C’s x day’s work = 1.
⇒ ⇒ 6(x – 5) + 5(x – 3) + 4x = 60.
⇒ 6x – 30 + 5x – 15 + 4x = 60
⇒ 15x = 60 + 30 + 15
⇒ 15x = 105 ⇒ x = 7 days.
For visually handicapped candidates only

27. (c)

28. (b)

⇒ = 50

29. (b) A’s work in 1 hour =

B’s work in 1 hour =

(A + B)’s 2 hour’s work when opened alternately


=

(A + B)’s 4 hour’s work when opened alternately


=

Remaining part =

Now, it is A’s turn and part is filled by A in 1 hour.


∴ Total time taken to fill the tank = (4 + 1) hrs. = 5 hrs.
31. (a) C.P. = ` 450; profit = 20%
∴ S.P. =

Let the list price of the wrist watch be `x.


Then discount @ 10% = `

∴ S.P. =

According to quesiton,
x=

32. (a) By the rule of alligation:


Cost of 1 kg rice of 1st kind Cost of 1 kg rice of 2nd kind

∴ Required ratio = 1.80 : 2.40 = 3 : 4.


33. (a) Let length = l, breadth = b, height = h.
l + b + h = 24 (given) ... (i)
Diagonal of parallellopiped = 15 cm
= 15 or l2 + b2 + h2 = 225
Squaring eqn. (i) on both sides
l2 + b2 + h2 + 2 lb + 2bh + 2hl = 576
2(lb + bh + hl) = 576 – 225 = 351
[ Surface area of parallellopiped = 2(lb + bh + hl)]
34. (c) Single discount =

= – 100 + 21 = –79%
‘–’ denotes discount. Hence, single discount equivalent to 79%
35. (a) Solving this type of question by short cut.
Net profit% =

17% = [∴ ‘–’ for dicount]


27 =

27 × 10 = 9y
y = 30%
Hence, He must mark his goods 30% higher than their cost price.
36. (d) According to question

36 × 4 + 4n = 50 × 3 + 3n
4n – 3n = 150 – 144
n=6
37. (b) Average height =

38. (c) First five prime numbers are 2, 3, 5, 7, 11


Average =

39. (b) Let us consider a packet or rice marked 1kg. It’s


actual weight is 80% of 1000 gm = 800 gm
Let C.P. of each gm be `1.
Then, C.P. of this packet = ` 800
S.P. of this packet = 110% of C.P. of 1kg =

Gain % =

40. (c) C.P. of a radio = 600


New C.P. after adding transportation charges
=

S.P. =
=

41. (c) or or

or or

Then, x: y: z is equal to 15: 12: 16


42. (c) Let the required distnace be x km.
Difference in the times taken at two speeds
= 8min =

Hence, the required distance is 4km.


43. (a)

To make a gian of 20%, the S.P. of Fan should be


S.P = =

Hence, S.P. should be 800.


44. (d)
45. (c) Let the number is x.
According to question
x – 10% of x = 30

Hence, the number is

46. (d) Relative speed =

Distance covered in crossing each other


= (108+112) = 220m
Required time =

47. (d)

R = 10%
48. (c) S.I (Simple Interest)
=
190 =

190 = 20R + 18R 38R = 190 R = 5%


49. (c) Saturday
50. (b) Students having both Science and commerce
= of

Students who have taken neither science nor commerce


= Total students – Students having both scicence and commerce
= 200 –120 = 80
For visually handicapped candidates only
49. (a)

Total examines passed = 70% of x


According to question
x – 70% of x = 420

50. (d) 10% of x = 20% of y

x = 2y
or

or x:y=2:1
PART C : GENERAL ENGLISH
51. (a) if pronouns of different persons are to be used together in a
sentence to provide good and normal sense then the serial order
should be second, third and first so here it should “He and I are very
good friends.”
52. (c) when a sentence starts with ‘one’ then to in accordance with the
singularity of the subject it should be one’s parents.
53. (c) ‘God’ is a universal noun so it should be preceded by ‘the’.
54. (b) according to the rules of the ‘articles’ the word ‘uniform’ should be
preceded by ‘a’ because here the vowel sound of ‘u’ is different
55. (a) here the use of ‘they’ is not needed as ‘the newspaper’ itself is the
subject
56. (b) according to the rules of conjunctions the word ‘no sooner...’
should be followed by ‘than...’
57. (c) the preposition ‘on ‘ is used to indicate a date or place should be
‘by’
58. (a) here it means one among many so it should be ‘one of my
friends...’
59. (c) here due to the sense of comparison it should be ‘the same...’
60. (c) here the word ‘will’ is not needed because ‘shall’ is already used in
the initial part of the sentence that itself signifies the tense of the
sentence.
61. (c) over, here it means came into force or effect
62. (b) interesting, exciting or fortunate
63. (a) with; into, here it means consisting of and plunged
64. (a) heavily or massively
65. (d) with
66. (a) accomplished, proficient or skilful
67. (d) over
68. (c) of, here it means cured of
69. (c) is, as it is a subject so it is a singular word
70. (a) over or give the responsibility to other person
71. (b) means uncivilised
72. (d) means obstacle
73. (a) means to hinder
74. (b) malevolent means malicious, spiteful, wicked
75. (b) disorganised means muddled, jumbled, confused
76. (b) humane means gentle, benevolent, civilised
77. (a) turned pale means colourless
78. (b) a clumsy person means plump
79. (d) tidy means clean
80. (a) treated us to which also means offered us
81. (c) scrutinising means revising
82. (b) since she has been directing
83. (c) so long as will be the exact phrase
84. (b) latest will be the correct word here
85. (b) bankrupt means insolvent or penniless
86. (a) microscope
87. (d) aquatic
88. (c) botany
89. (c) monopoly can also mean domination
90. (b) the correct spelling is excellence
91. (a) the correct spelling is grammar
92. (c) the correct spelling is omitted
93. (d) the correct spelling is calendar
94. (a) the correct spelling is objectionable
95. (d) the correct spelling is apology
96. (b) the people saw the golden pitcher in a lake nearby
97. (b) the emperor’s orders were that all the old men should leave his land
98. (b) a handful of water
99. (a) the emperor feared getting old
100. (c) the emperor was upset to see the old man because it reminded him
that he would grow old too
PART D : GENERAL AWARENESS
101. (a) Muhammad bin Qasim Al-Thaqafi (c. 31 December 695-18 July
715) was an Umayyad general who conquered the Sindh and Punjab
regions along the Indus River (now a part of Pakistan) for the
Umayyad Caliphate. He was born and raised in the city of Taif (in
modern day Saudi Arabia). Qasim’s conquest of Sindh and Punjab
enabled further Islamic expansion into India.
102. (d) Mao Zedong, commonly referred to as Chairman Mao (December
26, 1893 - September 9, 1976), was a Chinese communist
revolutionary, politician and socio-political theorist. The founding
father of the People’s Republic of China from its establishment in
1949, he governed the country as Chairman of the Communist Party
of China until his death.
103. (a) Gandhi calls Gokhale his mentor and guide. Gandhi also recognised
Gokhale as an admirable leader and master politician, describing
him as ‘pure as crystal, gentle as a lamb, brave as a lion and
chivalrous to a fault and the most perfect man in the political field’.
104. (d) High Yielding Variety (HYV) seeds played vital role in the
progress of agriculture. They are considered as `miracle seeds’. The
high yielding programme was launched in the Kharif season of
1966 in selected areas having assured rainfall. Seed is the basic and
crucial input for attaining sustained growth in agricultural
production.
105. (b) Amartya Kumar Sen, (born 3 November 1933) is an Indian
economist and a Nobel laureate. He has made contributions to
welfare economics, social choice theory, economic and social
justice, economic theories of famines, and indexes of the measure
of well-being of citizens of developing countries. He was awarded
the Nobel Memorial Prize in Economic Sciences in 1998 for his
work in welfare economics.
106. (a) The Great Depression was a severe worldwide economic
depression in the decade preceding World War II. The timing of the
Great Depression varied across nations, but in most countries it
started in 1930 and lasted until the late 1930s or middle 1940s. It
was the longest, most widespread, and deepest depression of the
20th century.
107. (c) Tea is an aromatic beverage commonly prepared by pouring hot or
boiling water over cured leaves of the tea plant, Camellia sinensis.
After water, tea is the most widely consumed beverage in the world.
108. (a) Camera is a capital for a photographer because he earns his
livelihood as it is his occupation
109. (c) The Swadeshi movement started with the partition of Bengal by the
Viceroy of India, Lord Curzon, 1905 and continued up to 1911. It
was the most successful of the pre-Gandhian movements. Its chief
architects were Aurobindo Ghosh, Lokmanya Bal Gangadhar Tilak,
Bipin Chandra Pal and Lala Lajpat Rai.
110. (d) The constitution of India came into effect from 26th January 1950
111. (d) The Cold War, often dated from 1947 to 1991, was a sustained
state of political and military tension between powers in the
Western Bloc, dominated by the United States with NATO among
its allies, and powers in the Eastern Bloc, dominated by the Soviet
Union along with the Warsaw Pact. This began after the success of
their temporary wartime alliance against Nazi Germany, leaving the
USSR and the US as two superpowers with profound economic and
political differences.
112. (b) The Indian Foreign Service is the foreign service under Group A
and Group B of the Central Civil Services of the executive branch
of the Government of India. It is the body of career diplomats of
India. The Indian Foreign Service is part of the Central Civil
Services of the Government of India. The Foreign Secretary of
India is the administrative head of the Indian Foreign Service.
113. (d) The Purna Swaraj declaration, or Declaration of the Independence
of India was promulgated by the Indian National Congress on
January 26, 1930, resolving the Congress and Indian nationalists to
fight for Purna Swaraj, or complete self-rule independent of the
British Empire. The flag of India had been hoisted by Congress
President Jawaharlal Nehru on December 31, 1929, on the banks of
the Ravi river in Lahore, modern-day Pakistan.
114. (d) Gandhra is noted for the distinctive Gandhra style of Buddhist art,
which developed out of a merger of Greek, Syrian, Persian, and
Indian artistic influence. This development began during the
Parthian Period (50 BC - AD 75). Gandhran style flourished and
achieved its peak during the Kushan period, from the 1st to the 5th
centuries. It declined and suffered destruction after invasion of the
White Huns in the 5th century.
115. (c) Christopher Columbus was an Italian explorer, navigator, and
colonizer, born in the Republic of Genoa, in what is today
northwestern Italy.
116. (c) integrated circuit semiconductor chip that performs the bulk of the
processing and controls the parts of a system; a microprocessor
functions as the central processing unit of a microcomputer; a disk
drive contains a microprocessor to handle the internal functions of
the drive
117. (c) Hibernate is the word that is never used to switch off the computer
118. (b) Highly concentrated sources of oxygen promote rapid combustion.
Fire and explosion hazards exist when concentrated oxidants and
fuels are brought into close proximity; however, an ignition event,
such as heat or a spark, is needed to trigger combustion. Oxygen
itself is not the fuel, but the oxidant.
119. (a) The Dumas method in analytical chemistry is a method for the
quantitative determination of nitrogen in chemical substances based
on a method first described by Jean-Baptiste Dumas in 1826.
120. (b) Delhi-Kolkata section
121. (c) The Tropic of Cancer, also referred to as the Northern tropic, is the
circle of latitude on the Earth that marks the most northerly position
at which the Sun may appear directly overhead at its zenith.
122. (d) The demographics of the People’s Republic of China are identified
by a large population with a relatively small youth division, which
is partially a result of China’s one-child policy. Chinese population
reached the billion mark in 1982.
123. (c) Sethusamudram Shipping Canal Project is a proposed project that
would link Palk Bay and the Gulf of Mannar between India and Sri
Lanka by creating a shipping channel through the shallow sea called
Sethusamudram and through a chain of islands collectively called
Adam’s Bridge, Ramar Palam , Ram Sethu and similar names. This
would provide a continuously navigable sea route in and around the
Indian Peninsula.
124. (d) A metropolitan area, sometimes referred to as a metro area or
metro, is a region consisting of a densely populated urban core and
its less-populated surrounding territories, sharing industry,
infrastructure, and housing. A metropolitan area usually comprises
multiple jurisdictions and municipalities: neighborhoods,
townships, cities, exurbs, countries, and even states.
125. (a) Deoxyribonucleic acid (DNA) is a molecule that encodes the
genetic instructions used in the development and functioning of all
known living organisms and many viruses. DNA is a nucleic acid;
alongside proteins and carbohydrates, nucleic acids compose the
three major macromolecules essential for all known forms of life.
126. (a) A steroid hormone (abbreviated as sterone) is a steroid that acts as a
hormone. Steroid hormones can be grouped into five groups by the
receptors to which they bind: glucocorticoids, mineralocorticoids,
androgens, estrogens, and progestogens. Vitamin D derivatives are
a sixth closely related hormone system with homologous receptors,
though they are technically sterols rather than steroids. Steroid
hormones help control metabolism, inflammation, immune
functions, salt and water balance, development of sexual
characteristics, and the ability to withstand illness and injury.
127. (a) nematocyst, minute, elongated, or spherical capsule produced
exclusively by members of the phylum Cnidaria (e.g., jellyfish,
corals, sea anemones). Several such capsules occur on the body
surface. Each is produced by a special cell called a cnidoblast and
contains a coiled, hollow, usually barbed thread, which quickly
turns outward (i.e., is everted) from the capsule upon proper
stimulation. The purpose of the thread, which often contains poison,
is to ward off enemies or to capture prey.
128. (a) Lactobacillus, also called Döderlein’s bacillus, is a genus of Gram-
positive facultative anaerobic or microaerophilic rod-shaped
bacteria.
129. (c) Chlorophyll is a green pigment found in cyanobacteria and the
chloroplasts of algae and plants. Its name is derived from the Greek
words chloros (“green”) and phyllon (“leaf ”). Chlorophyll is an
extremely important biomolecule, critical in photosynthesis, which
allows plants to absorb energy from light.
130. (b) In physics, the kinetic energy of an object is the energy which it
possesses due to its motion. It is defined as the work needed to
accelerate a body of a given mass from rest to its stated velocity.
Having gained this energy during its acceleration, the body
maintains this kinetic energy unless its speed changes.
131. (c) The ozone layer is a layer in Earth’s atmosphere that absorbs most
of the Sun’s UV radiation. It contains relatively high concentrations
of ozone (O3), although it is still very small with regard to ordinary
oxygen, and is less than ten parts per million, the average ozone
concentration in Earth’s atmosphere being only about 0.6 parts per
million. The ozone layer is mainly found in the lower portion of the
stratosphere from approximately 20 to 30 kilometres (12 to 19 mi)
above Earth, though the thickness varies seasonally and
geographically.
132. (a) Airlift pumps are often used in deep dirty wells where sand would
quickly abrade mechanical parts. (The compressor is on the surface
and no mechanical parts are needed in the well). However airlift
wells must be much deeper than the water table to allow for
submergence. Air is generally pumped at least as deep under the
water as the water is to be lifted. (If the water table is 50 ft below,
the air should be pumped 100 feet deep).
133. (a) The passenger-side mirror on a car is typically a convex mirror. In
some countries, these are labeled with the safety warning “Objects
in mirror are closer than they appear”, to warn the driver of the
convex mirror’s distorting effects on distance perception. Convex
mirrors are preferred in vehicles because they give an upright,
though diminished, image. Also they provide a wider field of view
as they are curved outwards.
134. (b) charts
135. (a) Dalveer Bhandari (born 1 October 1947) is a member of the
International Court of Justice and was a justice of the Supreme
Court of India. Bhandari is notable for his interest in
computerization and intellectual property law. Also he has a history
of promoting legal education, both to professionals and to the
general public who might be litigants. He has established mediation
and conciliation centers in Maharastra and an information centre for
litigants in the Bombay High Court.
136. (c) non-luminous zone
137. (d) Gamma radiation, also known as gamma rays, and denoted by the
Greek letter, refers to electromagnetic radiation of extremely high
frequency and therefore high energy per photon. Gamma rays are
ionizing radiation, and are thus biologically hazardous.
138. (a) Gives us atmospheric pressure - Ozone layer shields earth from
UV radiation - Ionosphere layer allows radio waves to be bounced
off and radios to work - Prevents water from vaporising off the
earth thanks to atmospheric pressure.
139. (d) Bacillus Calmette-Guérin is a vaccine against tuberculosis that is
prepared from a strain of the attenuated live bovine tuberculosis
bacillus, Mycobacterium bovis, that has lost its virulence in
humans.
140. (a) The United Nations declared 2010 to be the International Year of
Biodiversity. It is a celebration of life on earth and of the value of
biodiversity for our lives. The world is invited to take action in
2010 to safeguard the variety of life on earth: biodiversity.
141. (b) A greenhouse gas (sometimes abbreviated GHG) is a gas in an
atmosphere that absorbs and emits radiation within the thermal
infrared range. This process is the fundamental cause of the
greenhouse effect. The primary greenhouse gases in the Earth’s
atmosphere are water vapour, carbon dioxide, methane, nitrous
oxide, and ozone.
142. (b) The Kyoto Protocol to the United Nations Framework Convention
on Climate Change (UNFCCC) is an international treaty that sets
binding obligations on industrialised countries to reduce emissions
of greenhouse gases. The UNFCCC is an environmental treaty with
the goal of preventing “dangerous” anthropogenic (i.e., human-
induced) interference of the climate system.
143. (a) Sanjukta Panigrahi (24 August 1944 - 24 June 1997) [1] was a
dancer of India, who was the foremost exponent of Indian classical
dance Odissi. Sanjukta was the first Oriya girl to embrace this
ancient classical dance at an early age and ensure its grand revival.
144. (a) Vangipurapu Venkata Sai Laxman, commonly known as V.V.S.
Laxman, is a former Indian cricketer. Laxman represents
Hyderabad in domestic cricket and has played for Lancashire in
English county cricket.
145. (a) For much of the second phase of his career, cyclist Lance
Armstrong faced persistent allegations of doping. Armstrong
consistently denied allegations of doping until a partial confession
during a January 14, 2013, interview with Oprah Winfrey (which
then was broadcast in two parts on the Oprah Winfrey Network on
January 17 and 18, 2013).
146. (b) Gagan Narang is an Indian shooter, in Air rifle shooting, supported
by the Olympic Gold Quest. He was the first Indian to qualify for
the London Olympics.
147. (c) The father of the White Revolution in India was Verghese Kurien.
The white mentioned is milk production. He was able to implement
programs that took India from a country with very little dairy
production to the world’s largest producer.
148. (b) Charles Robert Darwin, FRS (12 February 1809 - 19 April 1882)
was an English naturalist and geologist, best known for his
contributions to evolutionary theory. He established that all species
of life have descended over time from common ancestors, and in a
joint publication with Alfred Russel Wallace introduced his
scientific theory that this branching pattern of evolution resulted
from a process that he called natural selection, in which the struggle
for existence has a similar effect to the artificial selection involved
in selective breeding.
149. (d) Kuldip Nayar (born 14 August 1923) is a veteran Indian journalist,
syndicated columnist, human right activist and author, noted for his
long career as a left-wing political commentator. He was also
nominated as a Member of the upper house of the Indian Parliament
in 1997. Nayar is also the author of 15 books, including “Beyond
the Lines”, “Distant Neighbours: A Tale of the Subcontinent”,
“India after Nehru”, “Wall at Wagah, India-Pakistan Relationship”,
“The Judgement”, “The Martyr”, “Scoop” and “India House”.
150. (b) According to GOI report, the total population of Bihar is
8,29,98,509 of which Schedule Tribes constitute 7,58,351. The
decennial growth of ST population in Bihar has been 32.4 per cent
which is 3.8 per cent higher than the rate of general population.

You might also like